You are on page 1of 471

. ,.,. · .

' glan yeu


~

,. . cahoe .IYthuyet

.

X.M.Targ " ~-. i!:f


ry'

-.'~"""___~~,

..

,/Z

?:j
I

. ·... (Gllr

11 71418B11

71"'"

...'-;.0iII0

..

.
,

C. M. Tapr.

KPATKV1~ HYPC TEOPETlti4 ECKO~

MEXAHltiKlti

x. M. Targ

,l
~~
.....

,
J

ca hoc Iy thuldet
Dich lir bOn tiellg Nga

Giao trinh gian yeu

Nha xuat ban d9i hoc vatrung hoc chuyen nghi$p HaNoi

Nha xuat ban«Mir» Matxcava

Ha BbeTHaMCKOM

aasnce

Ml)C Ll)C
Lo i Uri Uri Mo' tua cho Ian xu:H ban thir nh1H t ira cho Ian xuat ban thir ba . tu a cho Ian xuiH ban thir sau . dau .
{'f1.J'"

II 12 13 15 rhlF "hat

TiNH HOC V~T RAN


Chiro-ng I. Cac khai ni~m va tien di! co- ban cUa tinh hoc . I. 2. 3. 4. 5. Doi Lire Cac Lien Tien nro-ng cua tinh hoc. _ tien tinh hQC . . ket va phan hrc lien k~t de lien k~t . . . . . . 19 J9 20 22 25 28 29 29 31 34 35 37 38 40 41 49 50 51 52 52 54 56 59 60 60· 61 63 65

§ § § § §
Chuo-ng

de

11. Ho-p hrc.

H~ lV'c d6ng quy .

§ 6. § 7. § 8. § 9. § .10. § I I. § 12. § 13. § 14.

§ §
§

HQ'p luc bang phuo ng phap hinh hoc, HQ'p Il!C ella h~ hrc dong quy . Phan lire. . . . . . . . . .. ..... Hinh chieu cua lire tren mot true va tren mat phang . Phuo-ng phap bieu then luc bang giai tich Phu-o-ng phap ho-p hrc bang giai tich. Can bang cua h~ hrc dong quy Cac h~ tinh dinh va sieu tinh . . Giai cac bai toan tinh hoc . . . Mornen elm lire doi voi mot tarn (hay mot diem). 15. Dinh Iy Varin hang mornen cua ho-p hrc . . . 16~ Phuong trinh mornen cua h~ hrc dong quy. . .

ve

Chiro ng 111. H~ IV'c song song va h~ ngiu tren ml)t m~t phAng .

§ § §
§ § § §

17. 18. 19. 20. 21. 22. 23. 24.

HQ'p va phan cac hrc song song. . Ngau lire. Mornen ngau hrc , . . . . . . . . . Sl! nrorig diro-ng cua cac ngau hrc . . . . . . HQ'p cac ngau cung nam tren mat mat phang. Dieu kien din bang cac ngau,

Chirong IV. H~ 'V'c phan bll bilt Dinh Thu Cac Dieu

ky

tren m~t phAng .

Iy ve chuyen dich song song cua hrc , . . . . gon h~ lire phang vi! mot tam. ...... truorig hop thu h~ hrc phang ve dang toi gian . kien din bang cua h~ hrc phang tuy y. Truong hop cac lire song song.

6 § 25. § 26. § 27" § 28~


Chuo-ng Cach Can bilng cua h~ V~l Xac dinh (rng lire trong Cac hrc phan bo. giai cac bili toan

MVC

L~JC 6S

~
thi, .

75

79
80 83 83 84 86 86 87 88 88 92 93

V. CO' sO- cOa tinh hoc

do

§ § §
§ §
Chuong

29. 30. 31. 32. 33.

Da giac hrc va da gi{lC day. Thu h¢ hrc phang ve hai hrc . Xac dinh ho-p lire bang d6 thi. . . . . . Xac dinh ngjiu tong bang d6 thi . . Dieu kien can-bang h¢ hrc phang bang d6 thi , Xac dinh cac phan hrc cua goi nra . . . . VI. Cach tinh cac gian .....

§ 34. Khai niem ve gian. Tinh gian phang bang phuo-ng phap giai tieh . § 35 ~ Tinh gian phang bang d6 thi § 36· Bieu d6 Macxoen - Cremon . Chuong VII. Ma sat . . . . . . . . . . § 37. Cac dinh luat ve rna sat tnrot. § 38. PUn hrc lien ket nharn. Goc rna sat. § 39. Can bAng khi co rna sat . . . . § 40· Ma sat cua day doi vo i rnij.t tru . . . § 41 ~ Ma sat Ian va rna sat xoay . . . . . Chuong VIII. Hf ngAu "re vii hf I.,e phlln bIi bat ky trong khOng gian .

95 95 97
98 101 102 104 104 106 109 110 III 114 116 118 119 120 128 130 130 131 132 133 136

§
§ § § § § § § §

42. 43. 44. 45. 46* 47. 48* 49.

§ §

Momen cua hrc doi voi tarn la mot vecto- . . . . . . . Mornen cua hrc doi vai true . . . . . . . . . . . . . H¢ thirc gifra momen cua hrc doi voi tam va doi voi true Mornen cua ng1iu hrc la mot vecto . .. H'lP cac ngiiu hrc trong khong gian. Dieu kien can bang cua cac ngjiu . Thu he lire khong gian ve mot tam Cac t nro-ng hop thu gon h~ hrc khong gian ve dang toi gian . . Dieu kien can bang cua h~ hrc khong gian b1it kyo Tru-o-ng ho-p cac lire song song . . . ... ....: .................. 50. Dinh Iy Varinhong ve mornen cua hop hrc doi voi mot true . 51. Nhirng bai toan ve can bang ella v~t the du-oi tac dung ella h~ lire khong gian. 52~ Dieu kien can bang ella v~t rin khong tl,f do. Khai niern ve iinh 6n dinh ella can bAng. . . . . . . . . .

Chuong IX. Trong tllm § 53. Tam cua cac hrc song song. § 54. Trong tarn ella v~t rin . . . § S5 Toa dQ trong tam ella cac v!it dong chlit. § 56 Cac phiro-ng phap xac dinh toa dQ trong tam ella cac v!it. § 57. Trong tam ella mQt so v!it dong chat . . . . . . . . . Phdn thir hai

DONG HQC mtM


Chiro-ng X. D(lng hqc di~m M& dau ve dong hoc

VA V~T

RAN
139 139

§ 58.

MVC

L~JC

7
141

§ § § § § § § § § § § § § § § § § § § § § § § §. § §

59.

Cac phiro-ng phap xac dinh chuyen dong cua mot diem. Quy dao .

§ 60~ Chuyen tir phtrong phap eho chuyen dQng bang tea dQ sang phuong phap t\l' nhien . . . . . . . .
61. 62. 63. 64. 65. 66. 67. 68. 69. 70. 71~ 72~ 73. 74. 75. 76. 77. Vecto v~n ti5e cua diem. . . . . . . . . . . Vecto gia toe cua diem. .......... Dinh Iy v~ hinh chien cua dao ham cua vecto Xac djnh v~n toe va gia toe cua diem theo phirong phap Giai cac bai toan dQng hoc diem . . . . . . . . . Xac dinh v~n toe cua diem thea plurong phap t\l' nhien Gia toe ti1:p tuyen va gia toe phap tuyen cua diem . MQt so tnro ng ho-p rieng v~ chuyen dQng cua diem. Cac bieu d6 chuyen dong, v~n toe va gia tOe cua diem. Caeh giai cac bai toan . . . . . . . . . . . V~n toe trong toa dQ e\l'e. . . . . . . . . . . . . Khao sat chuyen dQng cua diem bang d6 thi , . . . XI. Chuy~n dqng tinh ti~n va chuy~ Chuyen Chuyen Chuyen V~n toe XII. dQng tinh dong quay dQng quay va gia toe ti1:n . cua v~t deu va cua cac

144
145 147 149 150 151 155 156 159 162

toa dQ . . . . .

164
. . 167 168 171 171 173 176 177 181 181 183 184 186 187

Chtro'ng

dqng quay ct'.a v,t rin .

.............. ran, V~n tOe goc va gia toe goc . chuyen dQng quay bien dOi d~u . di~m thuoc v~t quay ..... v,t rin. ........

Chirong

Chuy~n d(lng song phAng da

Phu-ong trinh chuyen dQng song phang, Ph an tieh chuyen dQng thanh chuyen dQng tinh tien va chuyen dQng quay 78. Xae dinh quy dao cua cac diem cua v~t th~ . . . . . . . 79. Xac djnh v~n toe cua cac diem cua v~t the. . . . . . . . 80. Dinh Iy v~ hinh chieu cac v~n toe cua hai diem trong v~t the. 81. Xac dinh v~n toe cua cac di~m trong v~t the thea tam v~n tOe tire thai. Khai niern v~ duo-ng tam tieh. 82. Cach giai cac bai toan . . . . . . . . 83.* Toe d6 . . . . . . . . . . . . . . . 84. Xac dinh gia tOe cac diem cua v~t th~ . 85' Tam gia toe tire thai. ........ XIII. Chuy'~ dqng cda v,t rin quanh di~m cli djnh va chuy~ tyo do . . . . . . . . . . . . . . . . . . . . dqogct'.a v,t rin

191
195

197
204 206 206 209 211 213

Chirong

§ § § § § § §

86. 87~ 88. 89. 90. 91 ~ 92.

Chuyen dQng cua v~t ra.n eo mot diem eo dinh , V~n toe va gia toe cua cac diem thuoc vat . .. Truong ho-p tOng quat cua chuyen dQng cua v~t ran t\l' do XIV. Chuy~n dQng ph.rc hQ'p cda di~m . . . . . . . .

Chiro-ng

Chuyen dong nro-ng doi, chuyen dQng keo thea va chuyen dong tuyet d1>i. 213 HQ'p v~n toe. .... 215 HQ'p gia toe . . . . . . . . . . . . 218 Cach giai cac bai toan . .' . . . . . ':, 223 hQ'p c,\a v,t rAn 229 229 230 232 HQ'p cac chuyen dQng tinh ti1:n . . . HQ'p cac chuyen dQng quay quanh hai true song song. H~ truyen dQng bang banh rang tru . . . . . . . . .

Chiro ng XV. Chuy~n d(lng ph.rc

§ 93. § 94.

95.'

8 § § §

MVC LVC 96.' HQ'p cac chuyen dc;mg quay quanh cac true di5ng quy . 97 *. Cac phu o ng trinh dong h9C O'lc. . .. 98~ HQ'p chuyen d(mg tinh tien vCri chuyen dQng quay. Chuyen 236 23.7 238

dQng vit.

Phtin thtr ba

DQNG Lt,rC HQC DI£M


Chuong XVl. 99. § 100. § 101. § 102. § 103. Chirong Mil- dilu dqng lY'c hoc, Cac dinh lu,t dqng IY'c hoc. 242 242 244 246 247 247 250 750 253 258 261 262 265 265 266 267 268 272 275 276 283 285 287 287 289

(~~

ic6,
.....
l~:JChuO'ng

Cae khai niem va dinh nghla co ban, Cac dinh lu~t d(ing lire hoc . . . . . . . . . . . . . don vi. . . . . . . .. Cac bai toan dQng luc hoc doi voi ehlit diem nr do va khong t\l" do. Gill.i bai toan thir nhat cua dQng 1\l"ChQC (xac dinh 1\l"Ctheo chuyen dQng)

He

XVII. Cac phu-O'ng trinh vi phi» chuy~ chung . Chuyen dQng Cach giai cac V~t roi trong Chuyen dQng Chuyen d(ing XVlIl.

d(ing coa di~m va cac tich phlln coa

§ 104. § 105.

106~

§ 107. § 108. § 109. § 110.

thAng cua diem . . . . . . . . . bai toan . . . . . . . . . . . . rnoi mrong can (trong kh6ng khi) cong cua diem . ella diem trong truong trong Itrc d6ng

nhat . ....

Cae djnh It tAng quat ct'Ja dqng IY'c hec di~m.

'.

§ § § § § § § §

Ill.

§ 112. § I I3.
114. 115. 116. 117~ 118. 119.

..... Xung IUQ'ng cua hrc . . . . . . . . . . . . . Dinh Iy vi! su bien thien dQng hrong ella di~m . Cong ella hrc. Cong sulit . . . .. ... Cac thi du vi! phep tinh cong . . . . . . Dinh Iy vi! bien thien dQng nang cua diem. Cach giAi cac hai toan . . . . . . . .. Dinh Iy bi~ thien momen d(ing hrong ella diem (dinh Iy cac momen) , Chuyen dQng duiYi tac dung cua 1\l"Cxuyen tam. Dinh lu~t cac dien tich. XIX. Chuy~o d90g kh60g ty- do ct'ta di~m. ........... Phuong trinh chuyen d(ing cua diem tren diro-ng eong co dinh. Xac dinh cac phan hrc lien kilt .. ' . XX. Chul'~n dQng tu-O'ng dlii ct'Ja di~m ............. nrong doi cua diem . . . dlit doi voi can bang va chuyen .............. dirng do S\l"quay cua trill diU.

Dong IUQ'ng va d(ing nang cua diem.

Chirong

Chirong 120. § 121.

292
292 295 298

§ § §

Phuong trinh chuyen dong va dirng yen Anh huong cua chuyen dQng quay cua trai d(ing ella cac vat . . . . . . . . . . . 122 ~ D(i I~eh cua di~m roi so vo-i phuong thang XXI. Dao dqng thAng ct'Ja di~m

Chtro'ng 123. 124. § 125.

...................

Cac dao dong t\l" do khong co hrc can . Cac dao d(ing t\l" do co 1\l"Ccan tY I~ voi v~n tl\c (cac dao d(ing tat diin). Dao d(ing etr5'ng birc. Cong hiro-ng . . . . . . .. . . . . . . . . . ..... Chuyen d(ing cua chat diem duoc phong nghieng mot g6c so viYi duo-ng nAm ngang 6- trong tnro-ng I\l"C hut cua trai dlit . . . . . . . . . .

300 300
306 308 311< 318

Chiro'ng XXII ~ Chuy~n d(lng ct'Ja v~t trong tru-irng l.,c hUt ct'ta trai diU.

126.

Ml)C L~'('

9
322 326

§ 127. § 128.

V~ tinh nhan tao ella trai d1it. Quy dao clip KMi niern v~ khong trong hrong ...

Phiin Ihit' IIr

&)
&
C. =-=-,' W

DONG LlJC HC;>C

m: VA V!T

RAN
332 332 333 334 338 340 343 343 344 345

Chirong XXIII. ~b- diu va! ~.qn~ l1fc hQC h~. Moml'll quan tinh clla. v,t rAn. § 129. CO' h". L\I"e ngoal va hrc trong . . . . . . . .'. . . . . . ,§ 130. Khoi luong ella h~. Khoi tam. . . " .......... § 131. Momen quan tinh ella v~t th~ doi voi mQt true. Ban kinh quan tinh . § 132. Momen quan tinh ella v~t th~ doi voi cac true song song. Dinh Iy Huyghen § 133'! Mornen quan tinh Iy tam. True quan tinh ehinh ella v~t th~. Chirong 134. § 135. § 136. XXIV. Djnh Iy va! chuy~n dqng khlli tim clla ,h~.

Phuong trinh vi phan chuyen d.Qng ella he . . . Djnh Iy v~ chuyen dQng ella khoi tam. .... Dinh lu~t bao toan chuyen dQng ella khOi tam . Cach giai cac bai toan .......... Djnh Iy vl! bi~n tbi!!n dqng hrcyng clla h~

§ 137. § 138. § 139. § 140.


141. § 142'!

347
351 351 352 353

Chirong XXV.

Dong hrong ella h~. . . . . . . . '. . . Dinh Iy ve bien thien dong hro-ng cua h~. Djnh luat bao toan dong IUQ'ng . . Cach giai cac bai toan . . . . . . . . . . V~t th~ eo khoi IUQ'ng pien dOi. Chuyen dQng ella ten lira. Djnh Iy va!bi~n thil!n momen dl1ng hr9'1lg ct'ta h~ ... Mornen ehinh ella dQng hrong ella he . . . . . .'. ... Djnh Iyve bien thien mornen ehinh ella dQng luong ella h~ (dinh Iy cac momen) , . . . . . . . . . . . . Dinh lu~t bao toan rnomen ehinh ella dQng hrong Cach giai cac bai toan XXVII. . . . . . . . . .. Djnb JYva! bien'thi!!n dqng ning clla h~ .

\x..

354 357
360 360 362 364 367 369 369 373 375 378 383 386

Chu ong XXVI.

§ 143. § 144.

I."

§ 145. § 146.
147. 148. § 149. § 150.

\...\.~ Chuo-ng

§ §

Dong nang ella he. ......... MQt so tnro-ng hop tinh cong ..... Djnh Iy v~ bien thien dQng nang ella M . Cach giai cac bai toan . . . . Tru-ong lire co the va ham h.re The nang . .. ., ... Dinh luat bao toan co- nang (fnl( dl.mg cac djnh Iy tAng quat cho dQng l1fc hQc v,. rAn . .... quay) . va chuyen dQng ella ...... . Chuyen dong quay cua v~t rAn . . . Con lac vat IY Chuyen dong song phang cua v~t rAn . . . . Ly thuyet gltn dung v~ hien tirong gyroscop (eon Chuyen dQng cua v~t rAn quanh mot diem eo dinh vat rAn nr do . . . . . . . . . . . . . . . U~g d\lng cac djnh Iy tAng quat
VBO

151.

§ 152.

§ '153. §
§ § § §
154. 155. 156. 157'! 158'!

387
389 389 391 394 401 406 410

Chtrong XXVIII.

f:{;J
.~

Chirong XX~X.

Iy thuyet va cham

10
§ 159. § 160.

MVC LVC Phuong trinh co' ban cua Iy thuyet va cham . Nhirng dinh Iy tAng quat cua Iy thuyet va cham H~ sll phuc hili va cham . Va cham cua v~t vo-i v~t can cll dinh . . . . . Va cham xuyen tam thing cua hai v~t (va cham caa cac kroi diu) S\!, mlit d!}ng nangkhi hai v~t va cham khong dan hili. Djnh Iy Cacno Va cham vao v~t quay . . . . . . . . . . . . . . Nguy& Nguyen Iy Dalambe Vecto chinh va m6men Cach giai cite bai toan Phan hrc dQng cua v~t IUQ'ng . . . . . . . XXXI. Nguyen 410 411 413 414 416 418 420 424 424 426 428 433 438 438 440 441 446 450 450 453 457 458 461 469

!:i

161. 162. 163. 164. 165~ 166. 167. 168. 169~

§ § § § § § § §

'j..

Chirong

XXX.

Iy Dallmbe.

Ap IIrc Il!n true c&a v~t quay. hrc .. len ..

....... . . . . '.: .. khlli ...

chinh cua cac ...... quay tac dung ......

quan tinh cua v~t nin . .. ...... truc . Can bang dQng cac ...........

Chiro-ng

Iy dq

dO-IkhA di va phU'O'ng trinh tAng quat c&a dqng IIrc hqc

§ § § §

170. 171. 172. 173.

DQ doi kha dl cua h~. So b~c ttr do Nguyen Iy dQ doi kha di . . . . . . . . Cach giai cac bai toan . . . . . . . . . Phuo-ng trinh tAng quat cna dQng lire hoc

Chirong XXXII ~ Di&i klfn clln bAng va cac phU'O'IIg trinh chuy~n dqng c&a h~ trong cac tea dq suy rqng 174. Toa dQ suy rOng va v~n tllc suy rong . . . . . . . 175. Lire suy rong . . . . . . . . . . . . . . . . . 176. Dii!u kien can bAng cua h~ trong h~ toa dQ suy rong 177. Phuong trinh Lagrang . 178. Cach giai cac bai toan Bang tra ciru nQi dung . . . .

§ § § § §

11

L(TI TV A CHO LAN XUAT BAN THU NHAT


Gido trinh ca hoc If thuyet nay viet cho doi ttrtrng chinh la hoc sinh cac truarng dai hoc kji thudt (hr chinh quy va hr tai chU'c) theo chu-ang trinn thu gon. Vi chu-ang trinh hoc tijp rlit khac nhau, nen khi viet gido trinh nay, trU'o-c:het chung toi co gdng d~ no. du-o-c tron v(?n va trinh My du oc dtiy dU cac phtrong phap co: ban cUa ca hoc rna cac kji SU' din biet va cac linh V!I'Ctrng dung cUa n6. Ngoai ra gido trinn phai phuc VI:I dU'(J'G cac chu-ong trinh gian yeu nMt cling nhu: cac chu-ang trinn mo: rrng ho-n. ca Ve nri dung cubn sach, ban doc co th~ xem trong phdn ml:lc luc. M(ic du giao trinh la gian yeu, nhirng cY day cY mire toi thi~u co xet ca cac bdi todn chuyen drng trong tnrong hrc hUt (vr tinh nhdn t{IO va quji dao elip), chuy~n drng cUa cac vijt c6 khi5i IU'(J'ngbien ddi (chuy~n drng ten lira), If thuyet so gilm ve hirn ttrang con quay, v.v... Dliy la nhirng vlin de hiin dai cdn phai du-o-c trinh My trong ca ,cac giao trinh rut gon.

dr

Khi viet, dira vao kinh nghiem stiu slic cUa nhitu niim giang dijy, chung toi thay rJng giao trinh gian yeu ci1n dU'(J'Ctrinh bay theo phirong phap di tit- tnrimg hop rieng uri tririmg lurp chung. Do do, trong sdch. chung toi trinh biiy tinh hoc phang tnro:c phdn tinh hoc khong gian, nghien cu-u drng l!I'c hoc di~m tru-ce phdn drng l!I'c hoc hr, nghien cu-u chuyen: dong thang tnrtrc phlin chuyen drng cong, v.v... Phu-o-ng phdp nay giup cho hoc sinh hi~u va tiep thu van de' nhanh va stiu slic lurn, do do lam cho qua trinh hoc tijp. dtro:« hi~n nhien va co trinh /!I'. Trong giao trinh, ngoai phu-ong phap hinh hoc va giai tich, dii dung phu-ong phap vecur la phu o ng phdp hirn dang sir dung rrng rdi, co nhieu U'U di~m va xem nhu: la phirong phap co: ban. Nhumg cY day mtri sa- dung nhirng phep tinh vecur c6 tinh chat giOng nhu: cac phep tinh tren cac dai IU'(J'ngvo huurng va khiing doi hoi cY ban doc 'nhii'ng khai niem mtri, Gido trinh cling dann nhieu trang cho cac vi du va cac phtro ng phap gi.1i bai tijp. Khbi IU'(J'ngbai tijp chiem tren mrt phdn ba t6ng so trang sach. Khi chon cac bai tijp, chung toi tijp trung chU f sao cho chung lam sang to cac hien ttrong co: hoc, va chi xet nhirng dang bai tijp chlnh gi.1i diro:c bdng mot trong cac phu ong phap trinh bay trong giao trinh. Nhirng 1M chi dan cadi giai bai tijp rlit b6 ich cho cac hoc sinh tw hoc, do d6 sacn nay c6 tM phuc VI:I cho hoc sinh cua moi chuyen ngann va nMt ld doi v&i cac hoc sinh tai chirc va t!l' hoc.

12

utI

TVA cno LAN XUAT BAN THU BA

Khi doc sad: nay, cdn chu y la cac tiet,ml,lc danh dliu sao khi cdn co thd b6 qua (ma khong anh hU'o.ng den qua trinh hoc t(ip todn b9 phdn con l{li) ; tuy nhitn giira cac tiet, ml,lc nay van co tM co cac chu gitii lien quan. Cac cong thtrc trong m6i phOn du-oc danh so m9t each lien tl,lC,cac cong thtrc du-oc sU- dung trong cung m9t phi1n du-oc ghi Mng so thu- tl!' CUelchung, khi dung trong cac phi1n khdc nhau, ta viet kern theo chl so cua tiet hay de ml,lc co cac cong thirc do. Tac giti xin cam on cac ong F R. Ganmakhe, A. lu. lslinxki va G. N. Xvenhicop vi dii gop nhieu y kien b6 ich cho giao trinh. Tdc giti ciing xin cam on cac ban dO'ngnghi~p dii luu y to-i giao trinh va chl cho nhieu y kien thich dang. Tac gia ciing mong cac ban doc gu-i cho nha xulit ban y kien phe binh ho(ic nguyen v{mg.

• ••
LaI TV'A CHO LAN XUAT BAN THU BA
Ldn xuat ban thu- ba cuon Giao trinh gian yeu co: hoc Iy thuyet co khac vo-i sad: cu & ch6 co m9t so b6 sung, su-a chira va thay d6i trong htiu het cac chu-ang. Phdn b6 sung co: btin ki them cac ml,lc ve chuyen d9ng CUelv(it rtin xung quanh m9t diJm c/i dinh (cdc phirong trinh d9ng hoc va d9ng lire hoc O'le), va chuimg co: so' cua phirong phap toa d9 suy r9ng iphirong trinh Lagrdng), so. di co S!I' b6 sung tu: Ii~u nay,la vi giao trinh pha! phuc VI,I yeu cdu dao tao can b9 chuyen mon cho nhieu linh v!/'c khac nhau. Trong sdch van trinh-bdy m9t it Iy thuyet co: btin ve gyroscop ( con quay) va nhirng vlin de cap thiet hi¢n nay nhu: chuydn d9ng trong tnrirng trong hrc (cac quy dao e/ip va cac CU9C bay vii trl,l)va chuydn d9ng CUel ac v{it co kMi 1U'(J'ng ien d6i (chuydn c b d9ng CUel en IU-a). Trong giao trinh con b6 sung them m9t tiet ve khai ni¢m kh6ng trong t 1U'(J'ng.-nd co khdi ni¢m ve n9i dung todn b9 cusn sach ciing nhu: thu- tl!' trinh bay, ban d(JCco tM xem & phi1n ml,lc luc.

Cdng nhu: & cac ldn xuat ban tnrtrc, 1i1n ciing cM y den nhieu vi dI,I va cac phirong nay phdp giai bdi t(ip vo-i khOi 1U'(J'ngchihn to-i tren m9t phdn ba cubn sach. Khi chon vi du, chung ~oi xulit phat tir yeu ctiu lam sang to thu-c cMt CUelcac hi¢n ttrirng co: h(JC va hinrng dan each sU- dung cac phirtmg phap trinh bay trong gido trinh. Ngodi vi du, trong sdch con co to-i 176 bai t(ip va-i cac chl dJn each giai nhtlm giUp hoc sinh tl!' hoc. Do do cusn sdch co tM phl,lc V!I cho hoc sinh CUelmoi nganh chuyen mon, nhat la doi vo-i cac hoc sinh h¢ tai chu-c va nhirng ngtriri tl!' hoc. Sach du-o«: soan
nhu: cac chuimg

di phuc

VI,I

cho ctmg tac giang ~y theo chu-ang trinh rut gon ciing

trinh mo. r9ng. Do do, khi cdn co tM b6 qua todn b9 cac chtrimg, tiet va de ml,lc co ddnh dliu sao, nhsrng ciing ci1n chu y rang giira cac tiet va cac de ml,lc nay van co s!/' lien quan nMt dinh. M(ic du trong sach sU- dung r9ng riii cac ky hi¢U vecto , nhirng di doc cac tiet va de ml,lc kh6ng co dliu sao thi kh6ng doi hoi ban doc phd! co kien thtrc ve dt;li so vecto:

trn

TVA CHO LAN XUAT BAN THU

sAu

13

Cdc cong thu-c trong bOn pMn cna cusn sach' dlu dln;rc danh so thu- t!t' lien t!lc. Khi trich diIn cong thu-c trong cung m9t phiin, chi vi~t so thu- t!t' cna chung, ChI nhirng cong thu-c cna cac phiin khac mlfi w(J'c ghi chu them m!lc trlch d&n.

MATxcaVA
Thdng 5 ndm 1963

x.

TARO

L(lI TVA CHO LAN XUAT BAN THU SA U


Trong llin xullt Mn thu- 6 nay Giao trinn gi&n yeu co hoc Iy thuyet ding co m9t so thay ddi, bd sung va su-a chira nhO nhu-ng khong Imh hU'tYng d~n n9i dung co ban va t~l'nh t!l' cuo~ sach. ChI co cac ti~t § 91 va § 128 thutJc phiin thlr hai la co nhirng thay ct6i ddng

va su-a chira, chung toi dii cM Y toi cac y kien phe binh' va di nghi do cac ban doc gu-i uri. Tdc giti chan thann cam on cac ban d9C do. Khi

b& sung

Thdng 1 nam 1968

x.

TARO

Gin

esc

B~N

DOC VI~T

NAM

Cu6n sach nay la sacti gido khoa vi€t cho sinh vi en cdc tnrimg dai hoc, song no ciing b& ich cho ca nhirng can b(j chuy en msn nao thliy can phdi on va b& sung cho minli nhiing kisn tlurc vii' CO' hoc,

CO' hl)c la khoa hoc ve sll can bdng va chuyen dpng eua vgt th~. No lam phon phU them kien thirc cba chung ta ve hang loat quy lugt co: ban cua thien nhien, ve cdc phirong phap nghisn cli'u nhieu hi~n tzrl)'ng trong th€ gio-i xung quanh tao No giup ta biit cdc hi~n tuong do phuc V(l con ngtriri, tao cho ta khd nang pluit trien tir duy va xay dung the gi/ri quan dung dan.
Mon khoa hoc nay co y nghia dgc bi~t d6i vo-i ngzra-i ky str tirtrng lai. No la ctr sO- ly thuyst viing chiic cua hiiu het cdc llnh VIlC cua nen ky thiuit da dq.t uri nhimg quy mo to ltrn va con dang tiep, tuc pluit trien, va khiing chi ky thUlj.t dan dung, Ngay tirtht: ky XVI, tir budi ddu cua Sll pluit trMn hoc thuyst ve chuyin d(jng, nha btic hl)c n6i tieng thiri do, d8ng thiri la nha yeu nzro-c - Nikalo Tartalia, da kiuim plui ra v{jt the dzrl)'Cphong di (dq.n dq.i bdc ) chuyln d(jng· nhir the nao va trong diii'u ki~n naa thi ctr ly bay se ltrn nhat, Khi do T6 qu6c Venexi cua ong dang bj qtuin this de doa tlin cting, Ong da quyet djnh giiri thi~u phdt minh cua minli cho «m6i ngiriri csng dan chdn chlnh, lam cho m3i ngtriri dzrl)'c vii trang t6t hen de bUo v~ t» qu6c cua minh,» Trong nhiing the ky vira qua, co hoc da phat trien thanli mim khoa hoc co: sOmanti me va y nghia cua nhiing kien thtrc ma no dem lai cho ta da tang len khimg the ke her dzrl)'c. Nhirng trau gi8i blit ky kien thirc co sO-nao va bier sir dung cluing bao giir ciing dOi hoi su: lao d(jng mi~t mai va nhieu sli'c lire, Vi vgy nhiing ai quy€t tdm hl)c tq.p de tro- thdnh m(jt can b(j chuy en msn co ich cho ddt nzro-c minh dilu phdi san sang lao d(jng nlur vgy. Dirng tiec sli'c lire, hay trau gi8i nhimg kien thirc vi nhirng kien thirc do se giup cdc ban, nhirng can b(j chuy en mdn ttrong lai, cimg vo-i nhdn dan Vi~t Nam anh himg xay dtrng m(jt ttnrng lai xan lqn cho T6 qu6c tutri dl}p ciu: cdc ban I X. TARG

15 ,.

Su: phdt trien coo ky thudt hi~n dai dang d{it ra cho cac ky SIt' nhitu vlin di c6 lien quan den tinh toan cong trinn (nha cu-a, cdu c/Jng, kenh d~p v.v... ), thiet ke, san xulit va su- dung cac may rrz6c, thiet bi, dl)ng co, ke cd cac phirong ti~n v~n tai nhu: oto, xe lu-a, tlfu tMy, may bay, ten lu-a, cdc con tlfu vU tr1,lv. v... Cdc vlin de d6 tuy nhilu ve, nhirng lai c6 tM giai quyet btlng nhirng nguyen ly, nhirng co: sO-khoa h9C chung. Dliy la vi phdn lon cac bai todn nay diu nghien cu-u cac quy lu~t chuyen dl)ng va can bang rna cac v~t tM Khoa hoc nghien cu-u cac quy lu~t tOng quat coo chuyen dl)ng va can btlnf! coo cac 1'91va ve S!l"nrtrng tac giira chung v&i nhau goi la co: hoc ly thuyet (hay co: h9C da! c~O'ng). CO' hoc ly thuyet la mot trong cac co: sO- khoa hoc coo cac ngdnn kJi thu{it hiin dai. CO'hoc theo nghia rong, la khoa hoc nham giai quyet cdc bdi loan ve chuyln drng hay din bang clla cac VQIIhe va W IU'O'nglac coo chung. Co- hoc If Ihuyel fa phdn co: hoc nghien ciru ve cac quy /UQIchung coo chuyen drng va W IU'O'nguic coo cdc VQIIhd, IU-cld ve cac quy IUQI o Ihd ap dung diro c cho ca chuyen drng clla trai d1i1 xung quanh mfil troi cling. nhu: chuyen c drng czla len It'ra va cac qua dan v.v... Phdn khdc coo co: hoc bao giJm cac chuyen nganh kji IhuQIkhac nhau co nhiem VIIIhiel ke va tlnh loan cdc cong Irinh ciing nhu: cac may moc, cdc co: diu va chi liel C!I 1M Till ca cac chuyen nganh kji IhuQInay di!u dfil co: stY tren cac d;nh IUQIva phu-ong phap coo co: hoc If thuyet. Trang co: h9C. chuyen dl)n~ du oc xem la chuyen d9ng co: h9C. tu-c la S!l" thay dOi vi tri ttrtrng d/Ji coo cac v{it tM trong khong gian theo thiri gian. Tac dl)ng co: hoc giira cac v{it la S!l" ttrong tac rna ket qua tM hi~n 0- S!l" thay dOi trang thai chuyen dong ho{ic hinh dang coo cac v{it (S!l" bien dq.ng). Da! lu ong dinn lu ong S!l" ttrong tac co: h9C coo cdc v{it g9i ld I!l"C. Nhiem V¥ chinh coo co: h9C ly thuyet la nghien cu-u cac quy lu~t tOng quat caa chuyeri dl)ng va can bang coo cac v{it tM duiri tac dung CUa lire d{it len chung. Theo tinh chllt coo bdi todn, ngiriri ta phdn co: hoc ly thuyet thanh' tinn hoc, dl)ng hoc va dl)ng lire hoc. Trong tinh hoc nghien ctru cac lu:c va dilu kien can bang cua cac v~t tM duuri tac dung coo I!l"C.Trong dl)ng h9C nghien cu-u cac tinh chat hinn h9C tOng quat coo chuyen d()ng, con trong dl)ng lu» hoc se nghien cu-u cac quy lu{it chuyen dl)ng coo cac v~t tM dinri tac dung caa lu-c..

16

M& DAU

Theo tinh chiu cUa dbi ttrong nghien cu-u, ngiriri ta phdn co: hoc ly thuye: thann : a) co: hoc chat diem (v{it diem) va ar hoc h~ chiu diem, trong do chiu diem du-oc xem nhu: mi)t v{it thi co kich thtrirc khong dang ke khi nghien cu-u chuyen di)ng (hay can bling) cUa no; b) cO'hoc v{it rtin la cac v{it thi co bien dang khimgddng ke khi nghien cu-u chuyen di)ng ho(ic can Mng; c) co: hoc cac viJt tM co khOi IU'(J'ngbien d6i (cac v{it thi mil khOi IU'(J'ngcUa no thay ddi thea thiri gian do thdnh phdn diu tao cUa no thay ddi; d) co: hoc cac V{lt bien dang (ly thuyet dan hOi va deo); d) co hoc cMt long (tMy di)ng hrc h{JC) va e) co: hoc ch/it khi (khi dong hrc h{Jc). Trong giao trinh co: hoc ly thuyet thirirng chi nghien cu-u co: hoc chiu diem va v{it rtin va nhimg quy lu{it tdng quat vl chuyen d!)ng cUa h~ chat diem. CO' hoc ly thuyet la mi)t mon khoa hoc t!l' nhien du-a tren nhirng dinh lu{it rut ra tir kinh nghiem dii tich luy du-oc vl m!)t lop cac hien ttrong thien nhien co lien quan uri chuyen di)ng cUa cac v{it tM Blh v{iy, vai tro va y nghia cUa co: hoc ly thuyet khong nhirng chl 0- cM no !iJ co sO- khoa hoc cUa nhilu nganh ky thu{l( hifn dai, mil con 0cM co the dung cac dinh lu{it va phirong phap co hoc de nghien cu-u va giai thlcn nhilu hifn ttrtmg quan trong trong the guri xung quanh ta, gop phdn thuc My S!l' phdt trien cUa toan bi) ki~n thirc vl tI,t' nhien ciing nhu: xay du-ng the gkri quan.duy v{it dung dt'in. CO' h{JC 1) dii xulit hien va phtu trien cung v&i ljch sU- phat trien cUa cac hrc IU'(J'ng san xulit xa Mi, co quan h~ ch{it chi v&i trinh di) san xulit va ky thu{it trong m(5i giai doan phdt trien.

U thiri cd dai, yeu ciJu cUa san xulit co: ban !iJ nMm tMa man cho nhu cdu xay du-ng cong trinh, nen nuri phat trien ly thuyet v~' cac may don gian (nhu: rong roc, don bdy, uri, m(it pht'ing nghieng) va vl S!f' can bdng cUa cac v{it (tinh h{Jc). Co so cua tinh hoc dii du-trc trinn bay trong cac cong trinh cUa nhd bac h{JCvi da! thiri cd dat Acsimet (287 - 212 tnrtrc cong nguyen).
Di)ng hrc hoc thi mdi vl sau nay miridtro-c phat trien. S!I' xulit hifn va phdt trien cac quan h~ til' ban cM nghia 0- cac nU'&c Tdy va Trung Au trong cac the kj 15 - 16 dii thuc day cac nganh tM cong nghiep, thirong nghiep, hang hQi va kf thudt qudn S!f' phdt trien mann (nhu: dii xulit hi~n sUng), ciing nhu: thuc ddy nnimg phat kien quan trong trong thiin van hoc. Tlit cd nhirng cai do dii tao dilu ki~n tich luy, h~ thOng hoa va tdng hip cac kien thirc thirc te va dan den nhirng phdt minh cac dinh IIJq.tcua . di)ng hrc hoc vao the kj 17. Nhirng ngtrtr! co cong xdy du-ng cO'so- cho di)ng hrc hoc la cac nhd bac hoc thien tai Galileo Galile (1564 - 1642) va Ixac Niuton (16431727). Trong cimg trinh « CO' sO-todn hoc cUa triet hoc tI,t' nhien» xulit ban ndm 1687, Niuurn dii trinh bay mi)t each co h~ thOng nhimg dinh lu{it co: ban cua co: hoc cd dien (cdc dinh lu{it Niutoni. Cac dinh luq.t nay d!i du-oc tiep t~c kiem nghiem trong thu:c

Au

1) Danh til' « co hoc » Ian dau xuat hien trong cong trinh cua nha triet hoc cO d~i vi dai Aristoten (384 - 322 tnroc cong nguyen). No bat ngudn til' chir Hy lap fL"rx.av~,rna ngay nay co nghia « cong trinh », « may moe », « phat minh ».

la

i-

\II'~~ '\.v:::/r:
te va dii du-oc ~h

McTDAU'

17

hoat d(ing tlurc tiln cua san xuat xii hf)i cUo loai ngtrtri khting

dinh. meu nay cho phep chung ta khang dinn nhirng kien thu-c co: hoc xtiy du-ng tren co: sO- cac dinh ludt Niuurn Id nhirng kien thirc dung dan rna cac ky su: co the manh dan Sll- dung trong hoat df)ng thirc tiln da minh 1). Trong the ky 18, cac phirong phdp gilli tich trong co: hoc bilt ddu phat trien manh me. Cac phtrtrng phdp nay dira tren or sO- cac phep tinli vi phdn va tich phdn, Nha toan hoc va cO' hoc vi dai L. ale (1707 - 1783) dii vt:,lchra cdc phu-ong phdp giai cac bai todn df)ng lire hoc diem va vtjt rtin bang each thiet Itjp va tlch phdn cac phu-irng trinh vi phdn chuyen dong. Trong so nhirng cong trinh nghien cu-u khac ve' linh vwc nay, thi cdc cong trinn cuo hai nlld bac hoc 16i lac Phap co gia tri hO'n ca dbi v&i SW phat trien cUo cO' hoc : D. Daldmbe (1717 - 1783) dii de xulit nguyen If cO' boc mtri va G. Lagrdng (1736 - 1813) dii de ra phirong phdp gilli tich t6ng quat trong df)ng Iwc hoc dWa tren nguyen (y Dalambe va nguyen If' df) doi kha di. Ngay nay phu-trng phdp gilli tich la phtrong phdp co: ban trong d(ing lire hoc. Df)ng hoc la ph!in co: hoc moi du-o:c tach rieng vao nil-a dtl'u the ky 19 do yeu cdu cUo nganh che tao may dang phat trien, Hien nay df)ng hoc dii co vai tro df)c lr)p nrtrng xtrng trong cong tac nghien cu'u chuyen df)ng cUo cac cO' eliu va may moe. Cac ding trinh cUo nha bdc h9C, nha tu: urirng thien tai M. V. Lomonoxop (1711 - 1765) va cac tdc phdm CUo L. ale la ngu-iri dii song va lam vifC nhieu ndm 0Petecbua dii co anh hirirng lon den 5W phat trien co: hoc dtiu tien 0- nw&c Nga. Trong so cac nha bdc hoc cua Nga .co cong phat trien cac linh vwc co: hoc If thuyet, tnrtrc het phai ke den M. V. Ostrograixki (1801 - 1861) co nhieu cong trinh nghien cWu quan trong ve phuo ng phdp g;ai tich trong ca hoc ; P. L. Tsebusep (1821 - 1894) dii de ra phtro ng huurng mtri trong vitc nghien cu-u chuyen df)ng cUo cac co: diu; X. V. Covalepxkaia (1850 - 1891) gilli mot trong so cac bai toan kho nhlit cUo df)ng luc hoc vtjt rdn ; A. M. Liapunop (1857 - 1918) de ra cac phtro-ng phap moi nghien cWu tinn 6n dinh cUo chuyen dong ; 1. V. Meserxki (1859 - 1935) d(lt nen mong cho cO' hoc cac vtjt the co khOi hrong bien d6i; K. E. Xioncopxki (1857 - 1935) co nhieu phat minh co ban ve If thuykt chuyen drng phan hrc ; A. N. Cnrlop (1863 - 1945) de'ra If thuye: tdu tMy va phat trien Iy thuyet ve' cac thiet bi gyroscop (con quay). Nhicng cong trinh Clia « Ngu-iri khai sinh nganh. hang khong Ngo» N. E. Giucopxki (1847 - 1921) vd cua hoc tro geill gui cUo ong la X. A. Tsapltrgyn (18691942) dii co tac dung to 1&11 v&i SW pluu triell sau lIay cUo co- hoc. D{ic diem sallg doi

1) Nhirng phat rninh khoa hoc ve sau cho bi~t, chuyen de;>ng cua cac v~t co v~n t3c giln bAng t3c de;> anh sang, tuan thea quy luat co- h9C cua Iy thuyet nro-ng doi, con chuyen de;>ng cua cac hat vi m6 (hat di~n tir, ...) thi thea cac djnh lu~t co' h9C hrong tir. Cac phat minh nay chl lam sang t6 them pham vi irng dung cua co- h9C cO dien va khang dinh me;>tan nira I Sl! dung dAn cua cac dinh lu~t co- h9C cO dien doi voi cac chuyen de;>ng cua cac v~t kh6ng phai la vi mo va co v~n t3c kh6ng gan bAng v~n t3c anh sang. D1iy la cac chuyen de;>ng dii

va dang co y nghia thuc ti~n to 16-ntrong .~~.,t,~J:...v~" ... ~.r;~.~ .:~~::'h~~, . 2 --7:\1

.... t.~e.

II
,

j,'I;':i_"I'~'c" """.'"~~
".,
-'_4-~~"

..

_', " '._..

18
1(.10

Ma

Diu

cUa Giucopxki fa trng dung cdc phtrtrng phdp co: hoc d~ giai quyit cdc nhiem v~ clip thiet. Nhirng j kien cUa Giucopxki dii co aM huxrng krn ca trong vi¢c giang day co: h9C If thuyet tY cac trtrirng dai hoc ky thufit cUa Lien x6.

k~v thudt

Ngdy nay, khoa hoc va k_v thufit phlli gop phdn vao vifC phat tri~n va hotin thirn khiing ngu-ng nftn san xuat xii h9i cM nghia va tip dung ky thu(tt nuri vav san xulit. Mu/)n v(ty doi hOi phil; tiep t~c nang cao chllt IU'9'ng dao tao ctin b9 ky thudt va mtY rong them co: stY If thuyet trong kien thtrc cua ho. MQt trong nhirng n9i dung va phu ong phap d~ giai quye: nhiem v~ nay Id nghien crru co' hoc Ij thuyet - m9t trong nhii-ng co: stY khoa hoc cUa nen ky thufit hifn dai.

PHAN THU NHAT

Il)

TiNH HOC V~T RAN

CHUCTNG

NHUNG KHAI NI~M VA TIEN DE

cO' BAN CVA TiNH HOC

§ 1. Dlii ttrcrng cna tinh hoc, Tinh hoc lit phan CO" hQCtrinh bay Iy thuyet t6ng quat vi! cac hrc va nghien ciru cac dieu kien can bang cua cac v~t th~ duo; tac dung cua hrc. Can bang diroc hieu hi trang thai dung yen caa v~t the nay doi vo-i cac vi.itth~ khac. Neu co th~ bo qua diroc chuyen d9ng ella v~t th~ dung lam v~t chuan d~ xet can bang thi can bang diroc quy iroc goi lit can bang tuyet doi, con trong tnrong hop ngiroc lai ta co can bang nrong doi. Trong tlnh hQCchi khaosat cac tnrong hop can bang tuyet doi cua v~t lh~. Trong thuc te tinh toan ky thuat, ta C!\ the· xem din bang doi voi trai diit hoac doi voi cac v~t tren trai dilt hi C<III bang tuyet doi. Trong phan dong 11!choc ta se chirng minh S1!dung dan cua dii!u khang dinh nay, 6day ta se co khai niem chat chi! han ve can bang tuy~t doi, dong tho; cling khao sat ca bai toan vi! can bang nrong doi cua cac v~t. Dii!u kien can bang cua v~t th~ chu yeu phu thuoc vito viec chung lit v~t ran, chat long hay chat khi. Can bang cua chat long va chat khi lit dai ttro-ng nghien ciru trong cac giao trinh thuy va khi tinh 11!C hoc. Trong coohoc ly thuyet thtro-ng chi xet cac bai toan vi! din bang cua cac 'v~t ran.
Tat d. cac v~t ran trong t1! nhien di!u it nhieu bi bien dang khi chju cac tac d9ng ben ngoai. DQ bien dang cua chung phu thuQc vao v~t lieu, vao hinh dang va kich thtroc cua cac v~t va vito 11!C dung, Do do d~ tac tao d9 ben cho cac cong trinh ky thuat, nguoi ta phai chon v~t li~u va kich thiroc cho cac chi tiet sao cho dQ bien dang cua chung kha nho khi bi lire tac dung I l. Do do, khi nghien ciru dieu kien can bang, ta hoan toan co the bo qua cac bien dang nho cua cac v~t ran do, xem chung nhtr nhirng Y~lthe khong bien dang hay v~t ran tuyet doi. V~y v~t ran tuyet dOi lit vat
j

,) Thi du, ngu<'ri ta chon v~t lieu va kich thuoc cac thanh trong kel cau, sao cho khi chiu lire dQ dan hoac d<) co khong diroc Ian hon mot phan nghin chien dai ban dau. Doi voi cac bien dang uon va XO~II V.V ... cling quy dinh d() cho phep luang nr.

20

NH1JNG

KHAI

NI~M

v): TIION DE

co

BAN

cux

TTNH

HQC

Ch. I

rna khoang each giu-a hai diem bat ky nao do cua no luon luon khong d6i. Sau nay, khi giai cac bai toan tinh hoc, ta se coi tat ca cac v~t la v~t rAn tuyet doi rna nhi~u khi de noi gon ta chi goi chung la v~t ran 1). Chang han 0- cuoi § 3, ta co the ap dung di~u kien can b~ng doi voi v~t rAn tuyet doi cho ca cac v~t th~ it bien dang va' bat k~ cac 'v~t th~ bien hinh nao. Nhir v~y, linh VI!C irng dung tlnrc te cua tinh hQC v~t rAn kha rong rai. Khi tinh to an dQ b~n cua cac chi tiet cong trinh hoac may moe, chung ta phai tinh den Sl! bien dang cua cac vat. Nhirng van d~ nay duoc de c~p toi trong cac giao trinh sire ben v~t li~u va Iy thuyet dan hOi. D~ v~t rtm diroc can bang (hay du-ng yen) khi chiu hrc, thi cac hrc phai thoa man cac dieu kien nhat dinh ve can bang h~ hrc. Tim ra cac dieu kien nay la mot trong nhirng nhiem vu co ban cua tinh hQC. Nhirng de tim ra cac dieu kien din bang cua h~ luc ding nhir d~ giai cac bai toan co- hoc, ta c~n biet cong cac lire tac dung len'v~t the, biet thay the h~ luc nay bang h~ lire khac, biet dira h~ lire khao sat ve dang don gian nhat. B6i v~y trong tinh hoc vat ran, din phai xet hai van de CO' han sau day: 1) hQ'P hrc va dua cac luc tac dung len v~t rAn ve dang toi gian ; 2) xac dinh dieu kien can bang cua cache hrc tac dung len v~t rin Ta co th~ giai bai toan tinh hoc bang cachxay dirng cac do thi (phirong

phap hinh hoc hay d5 thj) hoac bang cac phep tinh bang so (phirong phap giai tich). Trong giao trinh nay se trinh bay cl hai phuong phap nay, nhirng c~n chu y ra.ng plnrong phap do thi bao gio cling dong vai tro d~c biet trong viec giai cac bai toan co hQC.

§ 2. L ....c. Trang thai can bang hay chuyen d<)ng cua v~t the khao sat phu thuoc vito tinh chat tirong tac co hQC giira v~t the nay v61 cac v~t the khac, nrc la phu thuoc vao ap hrc, sue hut hoac sue dAy rna v~t phai chiu khi co tirong taco Trong co hoc, dai luurng dinh ltrtrng cho s!" ttrtrng tac coo hoc giira cac wjt thJ goi id lire.
Ta co th~ phan cac dai 1U'Q'ng khao sat trong co hoc thanh cac dai lirong vo hirong (hoan toan xac dinh diroc bang tri so) va dai 1U'Q'ngvecto (cac dai hrong ngoai tri so con co ca chi~u trong khong gian). INC la dai IU'Q'ng vecto. Tac dung cua hrc len v~t th~ QU'Q'cxac dinh boi : 1) tri so hay modun cua lire, 2) hu-trng cua lire, 3) diem ddt lire Co the xac dinh tri so cua hrc bing each so sanh hrc nay vai mot hrc

(1 N) hay I kilogam

diroc dung lam don vi. Trong co hoc ngiro-i ta lay don vi hrc la I Niuton hrc (l kG), trong do I kG = 9,81 N (xem cu th~ trong

') Cling con goi hi co tht (N.D.).

§)

Life

21

§ 101).

De do lire, ta co the dung mot thiet bi goi la lire ke.

HU'D-ngva diem d~t cua hrc phu thuoc vao tinh chAt tirong tac gifra cac v~t va vao vi tri urong hc3 ella chung, Thi du, trong hrc tac dung len cac v~t bao gio cling co chieu ti_ang dirng til tren xuong d1.f6'i. Ap lire do hai qua diu nhjin ep vao nhau ,ao gio- cling hirong thea phap tuyen ella cac mat diu tai tiep diem va d~t tai ehinh tiep diem do v.v .. : . Tren do thi ngU'Cri ta bieu di~n luc bang . mot doan thang co hirong (co mfii ten). D(> dai ella doan thang (doan thang AB .tren hinh I) E bieu dien tri so ella hrc theo mot ty I~ xieh nao do, chieu ella doan thang la chieu ella hrc, g()e (la diem A tren hinh 1) thirong Uiy ngay D tai diem d~L luc, Doi khi ngiroi ta bieu di~n hrc sao eho diem d~t trung voi diem d~u mut Hinh 1. ella hrc, tirc 1:'1, trung voi d~u miii ten (nhir tren hinh 4c). Dirong thang DE rna tren do d~t lire goi la dtrong tdc dung ella hrc, Ciing nhtr cac dai hrong vecto' khac, chung ta quy iroc ky hieu hrc bang mot chir d~m net (chir F) hoac b!Jng hai chfr nhirng co gach ngang ben tren (A B). Trj s() cua Il,I'C hieu lit I F I hoac bang chir thanh net (ehfr F). (Trong ky cac cong thuc viet tay nguoi La hay thay chir d~m net bang chir thanh net co gach ngang ben tren), " Sau nay ta quy U'D-egoi h¢ hrc la t~p hop cac Il,I'Ctac dung len m(>t v~t rAn nao do. Ta con su- dung them cac dinh nghia sau day: 1. M(>t v~t khong bi rang buec voi cac v~t the khac va co the thirc hien moi dich chuyen trong khong gian (til met vi tri nao do) goi la vq.t tlf' do. 2. Neu co the thay h~ 11,1'e dung len v~t rAn t1,1' bang mot h~ 11,1'e tac do khac rna khong lam thay dbi trang thai dirng yen hoac chuyen d(>ng ban d~u ella v~t, thi hai h~ 11,1'e o goi lit ttrtrng dtrong nhau. d 3. H~ 11,1'ena diroi tac dung ella no v~t rAn t1,1' co thf & trang thai r do dirng yen goi la h~ hrc din bang hay h¢ hrc ttrong dtrong khiing. 4. Neu M 11,1'e hao sat tirong dirong voi m(>t hrc thi 11,1'e o goi la k d hop hrc ella h~ 11,1'e. do, hop hrc ia hrc rna no co th€ thay the cho ca h¢ Do hrc tac dung ien vq.t thl L1,1'e tri so bang tri s() ella hop 11,1'e, co ngiroc chifu voi hop luc va tac dung doc theo cung dirong thang goi la hrc can bang. 5. Co the phan cac 11,1'e dung lenV~Lthe thanh 11,1'e tac trong va 11,1'e ngoai. Lire ngodi la 11,1'eil cac v~t the khac tac dung len cac phan tu- ella v~t khao t

22

NHO'NG

KHAI

NleM

vA

TlEN

DE

co

BAN

eVA

TTNH

HQC

Ch. I

sat. Lire trong la luc ma cac phan til' cua v~t khao sat tac dung Hin nhau.

6. LI,l'Ctac dung len v~t the tai mot diem nao do goi la hrc tq.p trung . L1!c tac dung len Hit ca cac diem trong met the tich hay len mot phan be ml)t cua v~t the goi la lwc phdn bOo Khai niem ve lire t~p trung chi co tinh chat quy uoc, vi trong thirc t~ khong the dl)t II,l'Clen v~t the tai mot diem. L1!c t~p trung xet trong co' hoc thirc chat la hop 11!c cua cac h~ 11!c phan ho nao do. Thi du, trong co' hoc, trong 11!c tac dung len v~t the la hop II,l'Ccac trong 11!c cua cac phan til' cua v~t. Dirong tac dung cua hop 11!c nay di qua mQt diem goi la trong Him cua v~t 1).

§ 3. Cac tieD dt; tiob hoc, Tat ca cac djnh ly va phuong trinh tinh hoc deu nit ra tir mQt so khoi diem diroc cong nhan khong co chirng minh toan hoc va diroc goi la cac tien de hay nguyen Iy tinh hoc. Tien de tinh hoc la k~t qua tAng hop cua vo so thi nghiem va quan sat ve can bang va chuyen dQng cua C3.C v~t dfl diroc thuc ~ kiem nghiem nhieu Ian. Trong cac tien de do, co mot la M qua cua cac dinh Iuat co' han cua co' hoc rna sau nay se dU'<;>'Cioi thieu trong phan dQng 11!c hoc. g

sa

TieD dt; 1. Ntu tren. m{JI vq.t rtin luy¢t doi IW do co hai lire Me dung, thi vq.t chi co Iluf can bdng khi va chi khi cdc hrc nay bang nhau ve tri so (F1 = F2), cung htrirng thea m{Jt dtrang thang va ngtrtrc chiiu nhau thinh 2).

Tien de I cho ta met vi du ve h~ 11!c din bang don gian nhat. Thirc te cho thay rang neu v~t r~n t1! do chi chiu mot II,l'Ctac dung thi khong the din bang dire-c.
TieD dt; 2. Tdc dung clla h¢ lire len vdt rtin IUYfl doi khong bi IIzayd6i nh« them wio hoiic btrt di m{JI h¢ lwc can bang. Theo tien de nay, hai h~ luc neu chi khac nhau boi mot h~ 11!c can bang thi nrong dirong nhau.

of

Hinh 2.

H~ qua cua tieD de 1 va 2. Tdc dung lu-c len vq.t rtin tuye: doi khiing bi thay neu dem dich chuy€n di€m d{it clla hrc theo du-irng tac dung Clla no uri m{Jt d;em kj ndo khac clla vq.t.

clla d6i doc

bat

I) Van d~ xac dinh trong tam cua v~t the se dU'Q'c khao sat 0- chiro-ng rAng : neu v~t dong chat co tam dlli xirng (nhir thanh chir nhat, thanh v.v ... ) thi trong tam cua no nam 6- ngay tam doi ximg do.
Ihay

IX.
tru,

Ta so- b6 khoi cau

!3

cAe TIEN DE TINH HOC

23

Thirc v~y, gia thu- co. 1\fCF tac dung len v~t th~ tai diem A (hinh 3). Khi do neu tren dirong tac dung cua hrc ta Jay mot diem B nao do va d~t vao dO. hai lire din Mng Ia F 1 va F 2 sao cho F 1 = F, F 2 = -F, thi tac dung

cua hrc F van duoc gifr nguyen khong


dbi. Nhtrng theo tien d~ 1, cac lire F va F2 tao tMnh) mot h~ din Mng, nen co. th~ 10~i bo 1 • Ket qua la v~t chl chiu tac dung mot lire F 1 bang F nhirng tac dung tai di~m B. B6i vay, vecto' rno ta 1\l'CF co. th~ d~t tai bat ky diem 0<\0 tren dirong tac dung cua lire. (nhfrng vecto nhir the goi Ia vecur trtro t]. Ket qua tren chi dung doi voi cac hrc tac dung len v~t rAn tuyet doi. Ta chi co.

Ilinh 3.

th~ su- dung ket qua nay trong tinh toan ky thuat d~ xac dinh cac di~u kien can bAng cua ket diu chir khong de tinh cac hrc trong cua cac bQ phan ket diu. Thi du thanh AB ve tren hinh 4a se din bang khi F1 = 1<;. Neu doi di~m d~t cua ca hai hrc v~ mot di~m C nao do (hinh 4b) hoac.doi di~m d~t ella lire F 1 v~ diem B, lire F 2 v~ diem A (hinh 4c) thi thanh vlln din bang. Tuy nhien, luc trong cua thanh & m6i tnro-ng hQ'P se khac nhau, & tnrong hop thir nhat thanh bi keo, & tnrong hQ'P thir hai trong thanh khong xu at hien lire trong, & tnrong hQ'P thir ba thanh bi nen 2). Do do, de xac dinh cac hrc trong ta khong the di chuyen diem d~t lire d9C thea dirong tac dung ella chung, Tien dti 3 (tien d~ hinh binh hanh h.... ). Hai 111'('ac dung VaG mot diim e t tren vt;it thi co h9'P hrc dfit tai cung didm do va du-o:c bieu diln bang dtrirng cheo hinh binh hdnh rna cdc canh chinh Iii cdc 11,I'c o. d

Vecto R bang duorig cheo hinh binh hanh dirng tren cac vecto F 1 va
F2
(

hinh 5) goi hi tbng hinh h9C cua cac vecto F 1 va F 2

R=F1+F2•

1) Nhirng

II)'C bi loai b6 hoij.c bi di chuyen .tren cac hinh vi! co vi! them dliu gachcheo,

2) D~ thanh bi keo (hoij.c bi nen) bo-i luc Flo ta phai dij.t Il)'c do vao m(it d1!u thanh, con d1!u kia phai chot cirng hoac giir cMt b&i hrc F 2 = - F 1 nhir hinh 4. Trong ca hai triro-ng ho-p, hrc keo (hoac hrc nen) deu bang F 1 chtr khong phai bang 2F 1 nhir d6i khi ngiro-i ta Him nrong.

24
;f

NHO"NG KHAt Nt~M vA T1EN

DE

CO· BAN CVA TINH HQC

en.

'i

IE

,
(

~--.

..

-4
'2

c '2

F2

JI

.. ,
;f

'i
_-_

~
B

In

~
Hinh 4.

C)

Hinh5.

Do do, ta con co the dien dat tien de 3 nhir sau: hai l~c tac dung len cung m{jt diim cua V(lt,co htrp Iu-c bang
t6ng hinl: hoc (t6ng vecur) cua cac lu«: va d(it tai cung diem do.

Ta din phan bi~t khai niem ve t6ng


cac lire va hQ'P 1l!C cua chung, Thi du, xet hai 11,fCF 1 va F 2 tac dung len v~t tai cac diem A va B (hinh 6). Lire Q tren

hinh 6 hi t6ng hinh hQCcua cac 11,fCF 1 = F 1 + F 2) bang dirong cheo hinh binh hanh nrong irng. Nhirng Q khong phai la hQ'P hrc cua cac hrc do vi co the thay mqt hrc Q khong the tac Hinh 6. d ung thay cho cl hai I\!'C F1 va F2' Ngoai ra nhir ta thay, hai hrc nay noi chung khong co hQ'P hrc ( § 48, bai t~p 42).
va F 2 (Q

TieD de 4. Ung vo-; m6i uic dung cUo V(lt the nay len V(lt the khac bao gid ciing co phan tdc dung v6'i cung tri so nhtrng ngtrirc chieu. Dmh luat ve s\!, bang nhau giira tac dung va phan tac dung la mot trong nhirng dinh lu~t co' ban cua co hoc. Theo dinh luat nay, ta suy ra : neu v~t A tac dung len v~t B mot luc F, thi dong thoi v~t B cfing tac dung len v~t A mot lire F' = - F co cung tri so, cung huang thea mot dirong thAng nhirng ngtroc chieu nhau (hinh 7). Nlnrng cac hrc F va F' khong tao thanh mot h~ hrc din bang vi chung d~t vao cac v~t khac nhau. Tinh chilt c.'la cac Il!'c trong. Theo tien de 4, hai phan tn- bat ky cua v~t the se tac dung len nhau Hinh 7. nhirng lire co cung tri so, nhirng ngiroc chieu nhau.

§4

LIEN KET vA

PHAN

t.u'c LIEN KET

25

Vi khi nghien cuu cac dieu kien din bang, ta xem v~t th~ nhir mQt v~t rAn tuyet dai, nen (theo tien de 1) t1it cac lire trong khi do, se tao thanh met h~ hrc can bang rna (theo tien de 2) ta co th~ bo qua. B6i v~y, khi nghien

ca

ngodi tac dung len v~t th~ hoac ket c1iu do. Do do sau nay, khi noi dtln cac II!C tac dung, neu khong co chu y gi d~c bi~t thi ta hieu ngam d1iy la cdc hrc ngodi, Tien d~ 5 (nguyen ly boa rAn). SI!' cdn bang cua v{it tM bien hinh (v{it bien dlJ,ng) dsrtri tdc dung cua h~ f!J:c vdn du-o c giii' nguyen sau khi v{it tM du o:c hoa rtin ithanh rtin tuy~t doi). Dieu khang dinh 6- tien de nay la hien nhien. Thi du,: ro rang la neu day xich can bang thi khi han cac mat xich lai day xich vAn can bang v.v ... Vi tnroc va sau hie hoa rAn van chi co cung mot h~ hrc tac dung len v~t can bang, nen ta con co th~ phat bieu tien de 5 nhir sau: cac hrc tdcdung len moi v{it bien hinh (v{it bien dlJ,ng) khi can bang ding thoa miin nhirng dieu ki~n nhu: doi vui cdc v{it rtin tuyrt doi: nhtrng doi vui cac v{it bien hinh thi cac dilu kirn din nay co tM khong fa dilu kirn duo Thi du, dai voi can bang cua mot day mem duo; tac dung cua hai hrc d~t 6- hai d~u, dieu kien c~n vAn la cacdieu kien nhir doi voi thanh cirng (cac II!C nay phai co tri so bang nhau, nhimg ngiroc chieu dQC then day). Tuy nhien day khong phai la dieu kien duo D~ day can bang ta con phai yeu c!u them la cac lire nay phai la 1\l'Ckeo, tire la phai co chieu nhir ve tren hinh 4a. Nguyen ly hoa rAn diroc sir dung rong rai trong cac tinh toan ky thuat, Nguyen ly nay cho phep ta khi thist l~p cac dieu kien can bang co th~ xem moi v~t bIen hinh (day curoa, chao, xich v.v ... ) hay moi c1iu true bien hinh nhir nhfrng v~t rAn tuyet doi va do do co the ap dung cac phirong phap tinh hQC v~t rAn de thiet l~p cac dieu kien can bang, Neu bang each do sa phirong trinh thu diroc van chua du d~ giai bai toan thi phai thiet l~p them cac phirong trinh thea dieu kien can bang cua tirng bQ phan ktlt cau hoac thea Sl! bien dang cua chung (hID toan co ke dtln Sl! bien dang se diroc xet trong giao trinh sire ben v~t lieu).

~~~~~~~$~~~~~~.~~

§ 4. Lien ket va pban hre lien keto Theo dinh nghia, mot v~t khong lien quan gi voi cac v~t, khac va tir vi tri dang xet co th~ thuc hien moi dQ do; trong khong gian goi la v{it tl!' do (thi du, qua bong bay trong khong khi). V~t rna dich chuyen cua no trong khong gian bi cac v~t khac co lien quan hoac tiep xuc khong chtl - goi la cac v~t khong tl!' do. Tat co nhirng doi tuurng co tac dung khOng cM dich chuyen cUo v{it khao sat trong khong gian du-oc goi fa lien keto

26

NHtJ"NG

KHAI

NI~M

vAnE:N

DE C(J BAN CVA TINH

HQC

('h. I

Thi du v~ cac v~t khong ur do nhir : v~t d~t tren m~t ban, canh cira treo tren ban l~ V.v... Lien ket trong cac tnrong hQ'P nay la : mdt ban khong cho phep vat di chuyen theo phiro-ng th~ng dirng lnrong xuong diroi, blm Ie' - khong cho canh cira roi khoi khung. V~t co lien ket khi chuyen dQng duoi tac dung cua lire se tac dung len lien ket nhtrng I\1'Cnao do goi la ap hrc len lien ket. Dong thoi theo tien de 4 khi d6 lien ket cling tac dung len v~t mot 1\l'Ccung tri so nhtrng ngtroc chifu. Lire rna lien ket tac dung len vd: .co tac dung can tre)' chuyen d{Jng cua vdt, goi la phan lu«: lien ket (l{l'C phan tac dung). Sau nay, ta goi cac lire khong phai la phan 11!clien ket (thi du nhir trong 11!C)la cac lire cM drng. D~c di~m cua cac 11!cchu dQng la tri so va chieu cua chung khong phu thuoc true tiep vao cac hrc khac cung tac dung len v~t do. Phan h,re lien ket khac cac 11!c chu dQng 0- ch6 tri so cua no bao gio- 'cling phu thuoc vao cac 11!c chu dQng va la 11!Cchira diroc xac dinh (neu khong co 11!cchu dQng nao tac dung thi phan 11!clien ket bang khong), De xac dinh phan 11!c lien ket ta phai giai cac bai toan tinh hoc nro-ng irng. Phan lire lien ket hirtrng ngtrtrc vtri chitu rna lien Ht can tre)' vgt di chuyen. Khi lien ket can tro v~t dich chuyen theo nhieu chieu thi chien cua phan 11!c lien ket cling chira biet va chi diroc xac dinh sau khi giai bai toano Doi voi cac bai toan tinh hoc, viec xac dinh dung chien cua phan 11!c lien ket dit quan trong, Do d6 ta se xet ty my hon chieu phan 11!ccua mot so dang lien ket co ba-i (xem them cac thi du 0- §25) .

1) M.t phiog (mit coog) hay glH t"a ohio. Mat diroc goi la nhdn neu 0- gan dung thir nhat co the xem nhir khong co rna sat doi voi v~t. Ml)t nay chi khong cho vat di chuyen doc theo phiro-ng phap tuyen chung cua mat tiep xuc cua cac v~t tai tiep diem (hinh 8a) 1), Do do phan I{I'C N cua mdt nhdn hay goi ura nhan sc hutrng theo phdp tuyen chung cua cac mdt tiep xuc tai tiep ditm va d{it ngay tai tiep ditrn do. Khi m:Qt trong cac mat tiep xuc nao do la mot diem (hinh 8b) thi phan lire htrirng thea phdp tuyen cua mdt kia.
2) Day treo. Lien ket diroc tlnrc hi~n 'bo-i day rnern khong dan (hinh 9). khong cho v~t M roi khoi diem treo theo hiro-ng AM. B&i v~y phan lire T cua OOy hirtrng doc theo day vi?" hia didrn treo. p

1) Tren cac hlnh 8 - II kh6ng vi: cac lire chu d(ing. Trong cac truong hQ'P ve tren hinh 8 va 9, phan hrc lien ket e6 chieu nhu tren hinh vc doi voi moi hrc chu d(ing, kh6ng phu thuoc vao v~t chuyen d(ing hay dung yen. .

LIEN

KET vA PH AN U)T)JLN

KET

27

a: Hinh 8. Hinh 9.

3) KhcYp tru (A true), Neu hai v~t lien ket voi nhau bing bul6ng bat qua cac II) khoan cua cac v~t, thi lien ket loai nay goi la lien ket khe-p hay la khop. Du-ong true cua bul6ng goi la true kho-p. V~t AB diroc lien ket khop voi goi nra D (hinh lOa) eo the quay tl! do xung quanh true khop (tren m~t phang hinh ve). Trong tnro-ng hop nay, diiu A kh6ng the dich chuyen vuong g6c voi true khop thea bat ky mot hiro-ng nao. Vi vay phan lire R cua khirp tru co th€ co chiiu biu ky tren m(it phdng vuong goc vtri true khtrp, tire la tren mat phang Axy. Trong tnrong hop nay, ea tri so I~n hirong (goc IX) cua lire R deu chira biet.
z

Hinh 10.

4) KhcYp cliu va khcYp" Loai lien ket nay eo dinh mot diem nao do coi. cua v~t, kh6ng eho n6 dich chuyen trong khong gian. Thi du ve loai lien ket nay : khop diu o chan may anh (hinh lOb) hay khop eoi (khop tru eo day) (hinh IOc). Ph/In lire R cUa khop ctiu hay khtrp coi co chisu tuy y trong khong gian. Doi voi phan hrc nay ta kh6ng biet tnroc eli ve tri so R Ian cac g6e do no I~p voi cac true x, y, z.

28

NHUNG

KHAI Nli;:M vA TlEN DE co

BAN CVA TJNH HQC

Ch.

5) Thanh. Gia thir trong met cau true nao do lien ket la thanh AB co khop 6hai dflu (hinh 11). Gia thu- thanh co trong hro-ng be kh6ng dang ke so voi cac hrc rna thanh phai chiu, Khi do tren thanh chi co hai hrc d~t tai cac khop A va B. Noi chung, cac hrc nay co chien tuy y, nhimg neu thanh AB din bang thi thea tien de 1, cac lire dij.t tai A va B phai huo-ng doc thea cung mot diro'ng thAng, tuc la doc thea true thanh (hinh 4a, c). Hinh 11. Dieu do chirng to, tharih chiu lire 6hai dau, neu trong ltrtrng cua no khOn~ dang ke so v&i cdc lire uu: dung, thi thanh chl bi keo hoac bi nen. N~u dung cac thanh do lam lien ket thi phan l,!,c N cua thanh se hsrtrng d9C
theo true thanh.

§ 5. Tien de lien keto Trong tinh hoc, ngiro-i ta khao sat can bang cua cac v~t kh6ng nr do dua tren tien de sau day : moi vt;it khong tu: do
co the xem nhu: vt;it tu: do neu vu-t bo lien kh biing cdc phan lire lien keto va thay tdc dung cUa chung

Thi du, co the xem dam AB trong hrong P (hinh 12a) voi cac lien ket la m~t phang DE, goi tua D va day KD nhir v~t nr do (hinh 12b) dang can bang diro'i tac dung cua lire chu dong P va cac phan lire lien ket N N va 8 ~ T. Co the xac dinh tri so cua cac phan lire lien ket thea cac K A dieu kieucan bang cua cac fJ .E p lire tac dung len v~t tl! do. P OJ IJI Day la phirong phap co- ban de giai cac bai toan tinh hoc,
A' D ~.

Vi~c xac dinh phan hrc lien ket con co y nghia thirc te (y chO, neu biet diroc chung, thi thea tien de 4 ta co the biet diroc ca ap lire len lien ket, tire la nhfrng so lieu ban dau can thiet de tinh d9 ben cua cac cau kien c6ng trinh.

Hinh 12.

HO'P U,rC BANG PH UUNG PHAp HiNH HOC

29

CHUO'NG II

HO~P LVC.

HE:

LVC DONG QUY

§ 6. Hep Ilfc bing phu-eng pbap biob hec, Hq-p IIfC cUa cac IlfC d6ng quy. Trong nhieu bai toan co hQC ta phai cong cac vecto, thi du cong cac hrc. Chung tanghien ciru tinh hQCMt dau bang bai toan cong luc ') theo phirong phap hinh hoc, Sau nay se quy iroc goi vecto bang t6ng hinh hoc cac hrc cua mot h~ hrc nao do la vee/a- ehinh cua h~ lire ay. Nhir dii noi (y § 3 (xem hinh 6), khong nen Ian IQn khai niem t6ng hinh hoc cua cac lire voi khai niem hop lire. Sau nay ta se thay doi v61 h~ lire nao ciing co the tim diroc t6ng hinh hoc (vecto- chinh), nhirng co nhieu h~ lire noi chung khong co hop lire. I) Hep hai IlfC. Co the xac dinh tong hinh hoc RClla hai lire F 1 va F 2 thea quy tAc hinh binh hanh (hinh 13a) hoac bang tam giac lire (hinh 13b) la mQt mra cua hinh binh hanh do. De dung tam giac hrc, nr diem tuy <; A, ta dung vecto hrc thir nhat, roi tir diem mut cua vecto nay dung vecto lire thir hai. Nci diem dau cua vecto thir nhat voi diem mut cua vecto tlnr hai, ta se diroc vecto lire R.
Tri so R bang canh A, C, cua tam giac A, B 1 C, diroc xac dinh thea cong thirc R2 Fi + F~ - 2F, F2 cos (180° -- ex),
trong do ex 1<\ goc giira hai Il!c. Do do : R
=

v' F~ +
0 --

F~ -

2F,F2

cose

(I)

dinh

G6c ~ lOa y do hrc R I~p voi cac lire thanh phan diroc xac dinh thea 1<; sin. Biet rang sin (180 ex) = sin ex, ta co
R siny sin~ sin ex' (2)

~l.~
F
2

I-~
7J
A, 0)

b) Hinh 13. Hinh 14.

I) Ciing eo nghia hi hop cac lire (N. D.)

30 2) Hop ba I,!,c khOng nim tren cling

Ch. II

mqt m~t phAng. T6ng hinh h oc

R cua ba lire Fp F2, F3 kh6ng n~m tren cung mQt mi;tt phang chinh la dirong cheo cua hinh h9P dirng 4U'Q'c tll' cac 11;I'c (quy tdc hinh h9p). do Ta co th~ clnrng minh di~u nay bing each Slr dung lien tiep quy tac hinh binh hanh (hlnh 14). 3) HQ'p h~ I,!,C. Co th~ xac dinh tong hinh h9C (hay vecto' chinh) cua moi h~ lire tuy y bang.each cong lien tiep cac 11;I'c theo quy tic hinh binh hanh hoac bang da giac 11;I'c. huong phap thir hai don gian va ti~n P IQi hon. Muon tim tong cac hrc F h F 2, F 3' ... F n (hinh 15a) bang phirong phap nay.tir di~m tuy y 0 ta dirng mQt vecto Oa bib di~n 11;I'c 1 (hinh F 15b ), til' di~m a dung vecto' ab bieu di~n 11;I'c 2' til' b dung vecto- be bien F di~n 11;I'c 3 , v. v. ", til' mut nt cua vecto' gan cuoi cung ta dung vectoF mn bieu di~n lire Fn' Noi 0 voi mut cua vecto cuoi cung, ta dU'9'C vecto On = R bieu di~n t6ng hinh h9C hay vecto chinh cua cac hrc thanh phan:

F 1 +F2 + ... + F n hay R

LF

(3)

Tri va chieu cua R kh6ng phu thuec vao thir t1;l'cac vecto 11;I'c iroc d cong, D~ dang th1iy la phirong phap dung vira riSi chinh la ket qua ap dung lien tiep quy tic tam giac b 11;I'c. Hinh ve tren hinh 15b dU"Q'C c goi la da giac ltrc (hay goi chung. la da giac vecttri. Do do, t6ng hinn hoc hay vecttr chinh

so

cl~a cdc lu-c ia canh khep kin cUo


da gidc lu« du-ng tren cac lire aJ 11 do (quy tdc da giac iwc). Khi Hinh 15. dung, da giac vecto can chu y rang t1it ca cac miii ten cua cac vecto thanh ph an phai du-oc dirng cung mQt chieu theo chu vi cua da giac, con vecto R thi co chieu ngU"Q'Clai.
0)

HQ'P lire eua h~ lire dong quy. Chung ta se nghien ciru tinh hoc mQt each tudn t1;l'til' M 11;I'C don gian den h~ 11;I'c phirc tap hon, Hay bit dau tll' hf hrc dfJng quy iiI hf lu:« rna dtrong tac dung cua chung edt nhau 0- cung mot didm (hinh 15a). Theo h~ qua cua hai tien d~ dan thi h~ 11;I'c d5ng quy tac dung len v~t rin tuyet doi se nrong duong voi h~ 11;I'c ~t tai cung d mot di~m (diem A tren hinh 15a). Neu ap dung lien tiep tien de binh hanh Il;1'C,ta se thay hf iu-c ddng quy co hop lire bl:mg t6ng hinh h9C (bting vecto: chinh) cua cac ift-c do va d(it tai didm dong quy. Bei vay, neu cac 11;I'c 1, F 2, ... , F n d5ng quy tai di~m F

PHAN Life

31

A (hinh 15a), thi vecto h,c R diroc dung theo phirong phap da giac luc va d~t tai diem A se la hop I\1'c cua h~ I\1'c do. -

N~u cac I\l'C d6ng quy tai mot diem nam ngoai hinh ve thi co the tim diem d~t hop I\l'C bang phuong phap de> thi trinh bay 6- § 30.

§ 7. Phin brc. Phan mot I\l'C thanh nhieu I\1'c thanh phan, co nghia la tim mQt h~ I\l'C rna I\1'C do Ia hop I\l'C cua no. Bai toan nay bat dinh, no chi co Uri giai duy nhat n~u co them cac di~u kien, Ta xet hai tnrong hop rieng quan trong nhllt.
I) Philn l\J'c theo hai htrlrng da cho. Gia su- phan I\1'c F (hinh 16) theo cac hirong song song voi cac dirong thang AD va AD da cho (l\1'Cva cac dirong thAng d~u n~ tren mot m~t phang), Bai toan nay tUO'Dg t\1' bai toan O\1'ng mQt hinh binh

hanh rna dirong cheo Ia I\1'c F, con cac canh thi song song voi cac dUCrng thAng
AB va AD. Muon v~y, tir diem dau va diem mut cua I\1'c F, ta dung hai dUCrng thang song song vm AB va AD. L\1'C P va Q chinh Ia cac I\1'c thanh phan can tim

- CPQ
p,

M!!:"'::';..+-,-"".J--7J
0)

b)
[J

viP+Q=F.

Hinh 16.

Ta cling co the phan hrc bang each dung tam giac I\1'c (hinh 16b). Muon v~y, til' diem tuy ya ta dung I\1'c F va qua hai d1iu cua hrc F dung cac dirong thang song song AD-va AD cho toi khi chung elit nhau. Cac 1\l'CP va Q tim duoc thay th~ ch"o l\1'c F neu d~t chung vao diem A hoac vao bat cir diem nao do tren dirong tac dung cua hrc F. 2) Phin

l\J'c thee ba

btro-ng

da cho. Neu

cac hirong nay

khong

n~ tren mot m~t phang thi day la hili toan tinh dinh rna nQi dung la dung ,mQt hinh hQp dirong cheo la luc F, con cac canh thi song song V<Yl cac hirong da cho (xem hinh 14).

rna

Dum day ban dQC hay t\1' xet cac tnrong hop phan mot hrc F da cho thanh hai hrc P va Q nAm tren cung mot m~t phang voi hrc F neu biet cac tri so cua hrc P va Q sao cho P Q ~ F. Bai toan co hai each giai.

each gilii blii toano Ta co th~ dung phirong phap phan l\1'Cde xac dinh ap hrc cua hrc da cho len lien ket. Nhir vay, muon tim ap hrc lien ket gifr chat v~t the, ta phan hrc do theo cac hirong cua phan hrc lien ket,

32

HO'P

LVC.

Hl). L\fC

DONG

QUY

eh. II

(vi theo tien de 4 ap h,rc lien k~t va phan hrc lien k~t cung doc thea mot dirong thang), Do do, de ap dung phirong phap nay ta can xac dinh tnroc htrong cua phan hrc lien k~t nrong irng,
Bai t~p 1. Xich dong g5m cac thanh AC va BC lien ket veri nro ng va veri nhau bang cac khop sao choL BAC= 90°,.c:: ABC = II (hinh 17). Tai khop C treo v~t co trong hro-ng P. B6 qua trong ltro'ng cua cac thanh, hay xac dinh lire nen len thahh Be. Bai gili. Ca hai thanh d~u bi hrc P tac dung. Ph an hrc cna chung trong tnr<'rng hop nay hiro-ng dC)Ctheo cac thanh. Do do, de tim lire nen, ta dij.t h,rc P vao diem C, r6i phan P thanh cac thanh phltn theo cac htrong AC va Be. Phan hrc 81 ehinh la lire nen can tim. Theo tam giac CDE, ta co : SI ~- -pcos II Ciing theo tam giac do, ta thay thanh AC bi keo boi hrc : S2
00

P tg s.

Khi goc II tang len, cac Ivc tac dung len ca hai thanh ciing tang len, khi II gan bang 90° chung se rat lon. Thi du, khi P= 100 kG, II = 85", ta co S 1"" 1150 k.G, SF" 1140 kG. De giam 1l)"C tac dung ta can giam goc II.

Hinh 17.

Hlnh 18.

Theo ket qua tren ta thay, doi khi co the xAy ra tnrong hop h,rc be nhimg lai tao ra ap hrc rat Ion len cac bQ phan rieng re cua cliu true (xem them bai t~p 2). Dlly la vi cac lire dlTQ'C hQ'P va phan theo quy tAc hinh binh hanh, rna trong hinh binh himh thi dU'IYng cheo co th~ ngan hon cac canh. Do v~y, neu khi giai bai toan tinh dU'Q'c nhimg phan hrc lon so vo; cac h,rc tac dung, thi dieu do khong co nghia la giai sai.

rat

Cuai cung ta chirng minh vi sao khi giai cac bai toan tuong I\l", ta nhllt thiet phai phan l\I'c rae dung theo phuong cac phan hrc lien k~t. Trong bai tij.p.vira. khao sat, ta phai xac dinh IVC & thanh BC. Ta hay dij.t Ivc P vao diem C (hinh 18) r51 phan P theo phiro'ng dC)C thanh BC va phirong vuong goc vo; BC. Ta duoc Ql = P cos
II,

Q2

P s in e.

Cach phan hrc nay tuy dung, nlnrng Q 1 kh6ng phai la lire can tim 6- trong thanh BC, vi thanh AC khong chiu tac dung toan bQ 1l)"C Q2' Do do lire Q2 tac dung len ca hai thanh va b&i v~y se tao ra hrc nen them len thanh BC rna v~n chira diroc tinh den trong Q,.

PHAN i.trc

33

Thi du nay chrrng to, neu phan lire kh6ng thee phirong ella phan hrc lien kilt, thi khong thu <1U"Q'e giAi elin tim. lai Hai t,p 2. Chup den trong hro-ng P = 20 kG (hinh 19) <1U"Q'C treo vao hai day AC va BC I:j.p voi phirong nAm ngang nhirng goc nhtr nhau bang II = 5°. Hay xac dinh sire ding 6- die day. Hai giAi. T<.Ii<1i~m C ta dimg luc P r5i phan hrc nay thee cac phu-o-ng doc thee day. Ta se <1U"Q'e inh binh hanh l\1"e co dang hinh thoi. Cac <1U"ang cheo cua hinh thoi vu6ng h goc vCri nhau tai diem gitra. Theo tarn giac aCb, ta co: p
= TJ

2 T

sin

II.

•.

Do do:

1=

= ---

P
II

2 sin

"" 115 kG.

Ttr e6ng thirc nay ta thay goc II cang giarn thi sire cang khi II = 1°, T"" 573 kG). Nl:u e3 cang eho day nAm ngang thi day sf bi dirt, vi khi do VCri 11--+ 0 , T --+ 00. HBi t,p 3. Hay xac dinh irng lire vong tai diem B va ap lire len true 0 6- co' diu thanh truyen tay quay ve tren hinh 20. Cho biet frng v<ri cac goc II va p co Ivc P tac dung len pittong A ; tay quay OB va thanh truyen AB co trong 1U"Q'ng kh6ng dang

6- cac

day cang tang nhanh (thi d\l

ke.

Bai giAi. bil:t hrc Q do B. De tim Q va AD ( AD d~n). Ta duoc

D~ xac dinh cac hrc din tim, ta phai thanh truyen AB tac dung len khop ta phan hrc P theo cac phtro-ng AB la phtro'ng pitt6ng A de len thanh

Hinh 19
Q
=

_!_-.
eosll

Dij.t hrc Q vao diem B r5i lai phan l\1"e nhir tren hinh 20, ta se <1U"Q'c irng hrc vong F va ap hrc R len true bang

F = Q sin y, R
F

Q cos y .

Q'
j}

Hinh.20.
:\ -731

34
G6c

HQ'P Life.

Hi;:

r.irc

DONG QUY
(X

eh

II

lit g6c ngoai cua tam giac OBA bang

+ ~. Do
R. -~

do, cuoi c~ng ta du-oc P cos


«(X

--COS-(X-'

+ ~)

Vi (X -' ~" 180 va (X< 9()O nen bao gio cling co F> 0, tire la luc F bao gio- cling co hirong nhir tren hinh ve. Con hrc R co huo-ng nr B ve 0 khi (X + ~< 9() ; khi (X +.~ >90' thi chieu cua lire R hoan toan nguoc lai. Khi (X + ~ == 9()', ta co R O. Qua vi du nay ta thay co th~ ap dung phtro-ng phap phan hrc ngay ca khi cac lire tac dung len vi:lt khong can bang nhau. Trong tnrong hop nay de xac dinh ap hrc len lien ket, ta phai phan hrc theo phirong ph an hrc lien ket va theo phiro'ng dich chuyen cua diem di,it lire (nhir d1i him 6- diem B). Ap lire len lien ket xac dinh diroc bang each nay goi la lip lire tinh, vi khi tinh ap hrc nay kh6ng tinh den khoi hro-ng, vi:ln toc vit gia ti.'lc cua cac vi:lt chuyen dQng. Trong thirc te ta chi co the su- dung cac ket qua tinh toan nay khi van toc va gia toc cua cac vi:lt deu co gia tri be. Ap hrc len lien ket co k~ den khoi hro ng, vi:ln ti.'le, gia toc cua cac v~t chuyen dQng dtroc goi la tip lu:c dong, D~ tinh ap lire nay, ta phai ap dung cac phuong phap cua dQng lire hoc (§ 169).

§ 8. Hmh chieu coa lu-c treD mi)t trIJC va treD m~t phADg. Ta hay xet each giai cac bai toan tinh hoc theo phirong phap giai tich (phiro-ng phap so). Phuong phap nay dira tren khai niem ve hinh chieu cua hrc tren cac true. Theo dinh nghia, hinh chieu coo lire tren true ld dai 1U'9'ng vo htrtrng hang chieu dai cua doan thang kep giira cac hinh chieu cua didm ddu ditm mut cua vecur lire tren true kem theo dau tuang trng, Hinh chieu co dau dirong neu chieu tir dau den mut cua no trung voi chieu diro'ng cua true va co dau am trong tnrorig hop ngiroc lai. Theo dinh nghia, ta thay hinh chieu cua lire tren cac true song song va cung chien bao giocling bang nhau. Dieu khang dinh nay rat co loi khi tim hinh chieu cua lire tren true kh6ng narn tren cung mat phang voi lire. Ta ky hieu hinh chien cua hrc F tren true Ox bang chir Fx. Khi do doi voi cac 1\l"Cvi': tren hinh 21, ta co ,) :

va

Fx =-= AB, AB, Do do: Fx

ab; Qx

= -~

ED,
=-

- ed. QCOSIX,. QCOSIX" (4)

Nhirng qua hinh vi: ta tnay : ~= F cos IX; ED,


=

Q coso Q coso

nrc la hinh chieu czia lu c tren true bling tich tri so cua hrc vo-i eosin cua goc giira phu ong cUa lire vo-i chieu diro ng cua true. Nhir v~y, hinh chieu se

F cos IX; Qx =~

=.'

,) Chien duo-rig cua true la chieu hirong tir diem 0 (goc true) ve phia co chir x (ky hi~n ten true). Cac miii ten tren hinh vi: la bien di~n cac vccto.

§9

PHUaNG

PHAp BlEU DIEN LIre

BANG GIAI j}

rtcu

35

/-

4)-----

:'~

p--r::!

/E

__ L

r' ,,-

Hinh 21. co dau dirong neu goc giira phirong cua lire va chieu dirong cua true la goc nhon va co dau am neu goc nay tu. Neu lire vuong g6c v&i true thi hinh chieu tren true bang khong, Hinh chiiu da F tren mdt pJuing Oxy IiI vecur Fx y = -0 B 1 kep giira hinh chieu cua diem ddu viI diem mill da lire F tren mdt phdng do (hinh 22). Do do, khac vo-i hinh chieu cua hrc tren true, hinh chieu cua lire z tren mat phang la dai hrong vecto-, VI 8 ! , no khong nhirng diroc th6 hien bang _ _,_ I tri so rna ell bang chieu tren m;;!t phang -'--_fJ o.n. Ve tri so thi F; y = F cos e, I I trong do e 1<\ goc giira phirong lire I I F vo-i hinh chieu F x y~~_-----TI.~~--_y Trong mot so tnro-ng h9'P, de tim hinh chieu cua 1l!C tren true, dau tien ngiroi ta tim hinh chieucua lire tren mat phang clnra true do, sau do mo-i Hinh 22. chieu hinh chieu nay len true. Thi du bang each do, til' hinh 22 ta tim diroc :

»:

F~=. Fy

-rx;

Fx Y sin q> =, F cos

r.,

c.OSq>

Fcos

e c.os~' e sin Cp.

(5) z

z'

§ 9. Phu-ong phap bi~u di~n i,!,c bAng giai tich, D~ bieu dien lire bang giai tich, ta can dung mot h~ true toa d!) Oxyz de xac dinh chieu cua lire trong khong gian. Trong co h9C, ngu oi ta dung h~ toa d!) thuan, nrc la h~ toa d!) rna neu nhin til' dau du-o-rig cua true o z ta thay de chap true Ox vo-i true Oy thi phai xoay true Ox mot goc quay be nhat nguoc chifu kim dong ho (hinh 23).
3*

:r'

O~

.r

-: Hlnh 23.

36

HQ'P i.irc.

Hi;: LIre

DONG QUY

Ch. II

Co th~ dung vecto bieu dien Il!C F neu ta biet tri so F cua Il!C va cac goc ... p, y tao boi lire veri cac true toa d9. Qua cac dai hrorig F ... ' ~, y ta xac dinh duoc hrc F. Yi tri diem d~t A phai duoc xac dinh them bang cac toa d9 x, y, z. Khi giai cac bai toan tinh h9C, ta nen bieu dien lire bang cac hinh chieu cua no. Muon v~y ta chirng minh rang, neu biet diroc cac hinh chieu Fx, Fy. Fe cua hrc tren cac true cua mot h~ toa dQ vuong goc decac thi hrc F hoan loan diroc xac dinh. Thuc vay, theo cong thirc (4) suy ra :

F;
=c

=cc

Fcos a, Fy

Fcos~,

Fz

Fcos y .

Ji2 vi cos" ~

.Binh phiro-ng cac dang thirc do roi cong lai, ta dtroc ~ + cos? ~ + cos" Y = I. Do do ta co : Fcc cos ~ YF2+F2+F2. x' Y
:t •

+ ~ + Ii.

F'

Fy

cos y

(6)

Cac cong thirc (6) cho thay neu biet cac hinh chieu cua hrc tren cac true toa dQ, thi co th~ tim duoc tri so va .cac goc cua lire voi cac true, tire lit co the xac dinh diroc hrc. Can chu y rang Cr cong thirc thir nhat,dhg tnroc dau din bao gio- cling mang dau dirong vi cong thirc nay xac dinh tri cua lire.

so

Neu phan lire F theo cac hu-orig song song veri cac tI1,1ctoa dQ (xem hinh 23), thi cac thanh phtin F x' F y' F z se co tri so bang hinh chieu cua hrc tren cac true tiro-rig irng. Til' day suy ra neu biet hinh chieu cua Il!c tren cac true toa dQ thi co th~ dung vecto' hrc bang hinh h9C theo quy tAc hinb hQP. xac Neu tat ca cac lire khao sat cung nAm tren mot mat phllngthi co the dinh nrng Il!C bang cac hinh chieu ella no tren hai true Ox va Oy.

Khi do cong thirc xac dinh luc theo cac hinh chieu se co dang :

F cos ~

= =-

YF;+F;,
Fx
F' cos
R I"

=
I
I

Trong tnrorig hop nay, neu biet diroc cac hinh chieu ta co the dimg Il!C thea quy tAc hinh binh hanh,

I I I
() I

I I
I

I I
I

._-;,----b>------'e:--·--Lc----'cf'7'"
Hinh 24.

§ 10

PHlH1NG

PHAp HQ"P t.trc BANG mAl

rtcn

37

§ 10. PhU'O'Ilg pblip hQ"p I....e bing giM deh. Ta co the chuyen tir h~ thirc giira cac vecto sang cac-he tmrc giira cac hinh chieu cua chung bang dinh Iy hinh hoc sau day: hinh chieu cua vecur uJng tren m{Jt true ndo do bang t6ng dai so cac hinh chieu cua cac vecur thanh ph/in ciing tren true do. Do do, vi hrc 1ft mot vecto ta suy ra: neu R = F I + F 2 + F 3 + F 4 (hinh 24), thi : R x = FIx + F2x + F3X + F4X , trong do :
FIx
=

ab, F2x

bc , F3X

=ccd, F4X

= - de, R; F
n-

ae.

GQi R 1ftt6ng cua h~ hrc (vecto chin h) F 10 F 2,"', theo dinh Iy tren ta co :

trong do R LFkz. (8)

s;
Neu biet Rx,

c=

LFkx,

Ry

LFIo/ '

n,

Ry, R z, theo cong thirc (6) ta dtroc :

cos a.

s, R'

R=

V R;+

R;+ R;;
R'
Ry

cos ~

cos

Rz R

}
R
=

(9)

Cac cong thirc (8), (9) cling cho phep giai bai toan cong lire bang gild 'tich. D~i voi cac lire cung narn tren mot m~t phang, cac cong thirc nrong irng co dang :

s;
R =:,VR;+

LFkx,

s,

LFky;
cos ~

}
y

RI,

cos a. =

R'

Rx

(10)

/f'

Neu cac hrc d1i diroc cho tnroc tri va cac g6c hop voi cac true, thi de ap dung phirong phap giai tich cong hrc, ta can tinh tnroc hinh chieu cua cac hrc do tren cac true toa dQ.
Bal ti.p 4. Hay xac dinh tbng cua ba hrc P, Q, F ; cho bist hinh chieu cua chung tren cac true toa dQ bang :
p. = 6N,Py

s~

= 3N,Pz
F.

= =

12N;Q. 5N, Fy

= 3N,Qy
=

~~ --7N,Qz

IN

2N, Fz

= -8N.

Bal gh\i. Theo c6ng thirc (8) ta tinh diroc : R. Ry


Rz
0=

II

6+3 3- 7

= 12

+ +

5 2

14N,
5N.

= -2N,

1-

8=

Thay cac gia tri nay vao dli.ng thire (9), ta co:
R=

VW +
14

(_2)2+ 52

O~

15N,
0)

cos

III

15' cos ~ = -15' cos y

Hlnh 25.

38

H<lP LI,rC. H~ LI,rC DONG QUY Ket qua hi: R


=

Ch.

II

15N,

II

= 21°,

~ "~97'40',

7030' (hinh 25a), neu cho: 60".

Bili t¥p 5. Tim hop hrc cua ba lire cung nam tren mot m~1 phang F =, F. 17,32kG, T
=

IOkG,

P ~~ 24kG,

«p = 30",~' ~~ -5kG; P,

.=
=

Bili gi&i. Tinh hinh chieu ~ua cac h,c do tren cac true t<;><1 d(>, La co :

= Fcos <p=

15kG,

T. =--Tcos~'
=

Fy

=-

Fsin <p = - 8,66 kG,

Ty

Tsin

'i' = 8,66 kG,

Py

= --

0; P = -- 24 kG.
24kG.

Khi do, theo cong thlre (10):

R. Nen :

= 15 -

5 = 10kG;

Ry = - 8,66 = 26kG;coslI ~=

8,66 -

24 = ~

R = ViOl
=

+
,II

(-24)2
=

= _2_;cos

13

=_

__13_.
13

Ket qua la R

26kG

67"20',

157"20'.

Muon giai bai nay tuong img VO; IOkG, kin ad cho ta tri so va ads; 2,5 em, thi R"" 25

bang phirong phap hinh hoc, ta phai chon met ty I~ xich (thi du I em r5i dung da giac IlJC bang cac IlJC P, F va T (hinh 25b). Canh khep chieu cua luc R theo ty I~ xich tirong irng. Thi du.neu khi do ta co kG. Sai so so voi l<ri giai chinh xac gan bang 4 'j; .

§ 11. Ciin bAng coa b~ I"c diSng quy. Theo cac dinh luat co hoc, ta bi~t rAng neu v~t th~ bi tac dung cua cac lire ben ngoai din bang nhau thi no co th~ dirng yen ho~c chuyen dQng « theo quan tinh ». Thi du v~t co th~ chuyen dQng tinh ti~ thang d~u.
Tir day ta co hai k~t lu{ln quan trong : 1) Cac lire tac dung len v~t dung yen cling nhir len v~t chuyen dQng « theo quan tinh » deu thoa man cac dieu kien can bang cua tinh hoc (xem bai t~p 6). 2) S\1' can bang cua cac lire tac dung len v~t rAn tv do la dieu kien din, nhimg chira do dl>i voi S\1' can bang (dirng yen) cua ban than v~t th~ do. Do do v{lt th~ se chi dirng yen neu' nhir tnroc khi chiu tac dung cua cac lire can bA.ng no da & trang thai dirng yen. D~ h~ luc dong quy tac dung len v{lt rAn diroc can bang thi dielJ kien c~n va do lit hop hrc cua cac hrc do bang kh6ng. Ta co th~ bieu dien dieu ki~n mit cac lire din bang phai thoa man duoi dang hinh hoc hoac giai tich. 1 . Di&. ki~n can bAng dU'o-i d~g binb hoc, Vi hop lire R cua h~ lire dong quy lit canh khep kin cua da giac lire dung tt'r cac lire do, nen R chi co th~ bang khong khi va chi khi nao di~m milt cua lire euoi cung trong da giac lire trung voi di~m dau cua lire dau tien, tire lit khi da giac lire tv khep kin.

dtrng du oc

Do do, disu kien din vel dli dd hf lire dong quy can Mng lit da gidc 'we tu- CGC lu:« do phtli tl,l' khep kin.

~ JJ

CAN BANG CVA He LVC DONG QUY

39

2. Di~u ki~n can bAng dO'lri d,ng giai rich. O1.1'&idang giai tich, hop hrc cua M hrc dong quy xac dinh theo cong thuc :

fR;- +

R;

R;.

Vi bieu thirc diroi dau can la t~ng cac so hang duong, nen R chi bang khong khi nao dong thoi Rx = 0, Ry = 0, R, = 0, nrc Ia khi cac lire tac dung len vat ran se thoa man cac dang thirc (suy tll' c6ng thirc (8» : (II) Cac dang thirc (II) m6 ta cac dii!u kien can bing diroi dang giai tich : df h¢ hrc d611g quy trong khong gian dtro:c can bang, thi dieu ki¢n din va dlJ Ia tlmg cdc hinh chieu cua cdc ltrc db tren ca ba true toa d{Jphai bang khong. Neu cac hrc dong quy tac dung len v~t the nim tren cung mot mat phang thi chung I~p thanh h¢ lu:c phang diJng quy. R6 rang trong tnrong hop h~ hrc ph~ng dong quy ta chi co hai dii!u kien din bing:

LFIo<

0, LFky

'c~

0.

(12)

Cac d~ng thirc (II) va (I 2) cling la cac,die'u kiin din (hay cac phuong trinh) doi voi su can bing cua v~t ra.n nr do deoi tac dung cua cac lire dong quy.

sir dung dinh Iy sau day: neu v~trdn tu: do rna can Mng diroi tdc dung
cua ba lire khong song song niim tren cung mr}t mdt phdng, thi dirtrng tdc dung ella chung edt nhau tai mr}t didm. De clnrng minh dinh Iy, dau tien ta dung hai lire nao do, thi du F 1 va F 2' Theo di~u kien cua dinh Iy, cac lire nay cung nAm tren mot mat phang va kh6ng song song, nen duong tac dung cua chung se cAt nhau tai diem A nao do (hinh 26). D1.1'a cac lire F b F 2 v~ diem A rai thay chung bing hop hrc R. V~t
$

3. Dinb It v~ ba lu-c. Nhieu khi de giai cac bai to an tinh hoc, ta nen

Hinh 26.

bay gio chiu Ute dung cua hai lire la lire R va lire F 3 d~t tai diem B nao do cua vat. Khi do neu v~t can bang thi theo tien d~ 1, cac hrc R va 1i'3 phai hirong theo cung mQt diro-ng thang doc theo AB. 00 do lire F 3 cling phai di qua diem A. Do la dieu din clnrng minh. Ta chu y rang, dinh ly nay khong co dinh ly dao, tire la du cho duong tac dung cua ba hrc co cat nhau tai mot diem, nhirng v~t duoi tac dung

40

H(JP LVC. H~ i.uc DONG QlJY

Ch. II

cua cac hrc do co the van khOng can bang. Do do dinh 1'1 nay chi 1<1 disu kien din khong ph IIi la dieu kien dli doi voi s\f can bang cua v~t dn t1)' do duoi tac dung cua ba hrc.
Thi d\l. Darn AD gan khop tai diem A va ty tren canh nro-ng D (hinh 27). Nell giai phong lien ket, thay tac dung cua chung bang cac phan lire uro-ng irng ta se duoc diim tir do. Dam din bang diroi tac dung cua ba 11!CP, N I) va R" rna dirong tac dung cua chung thea dinh ly vira chirng minh, phai cat nhau tai mot diem. Ta dii biet duong tac dung cua cac lire P va N" cat nhau tai diem K. Do do ph an 11!C R" cua khop d~t tai A cling phai di qua diem K,tirc la phai hU'o-ng doc theo dirong thang AK. Djnh ly ve ba 11!c trong trirong hQ'P nay cho phep ta xac djnh hu ong cua phan 11!c cua khop A rna ta khong bi!!t tnroc,

Hinh 27.

§ 12. Cac h~ tinh djnh va sieu tinh.Khi giai cac bai toan ve can bang cua v~t th€ khong t\f do, ta thay phan lire lien ket la nhirng dai hrong
chua biet. So cac dai IUQ'ng chua biet do phu thuoc vao so IUQ'ng va tinh chat cua cac lien ket. Bai toan tinh hoc chi co th€ giai diroc khi so phan lire lien ket chua biet khong nhieu qua so phirong trinh can bang chu-a cac phan hrc do. Nhirng bai toan nhir v~y goi La cac bai loan tinh dinh, con h~ cac v~t co tinh chat do goi La hf tinh dinh.

Bai toan trong do so cac phan hrc lien ket chua biet nhieu hon so phirong trinh can bang chu-a cac phan hrc .do
goi La bai loan sieu tinh, con h~ cac vat co tinh chat do goi 1:'1.hf sieu tinh, Thi du ve h~ sieu tinh : mot v~t nang treo tren ba day cung nam tren mot mi)t phang (hinh 28). Trong bai toan nay co ba dai hrong chua biet (la sire ding cua cac day Tb T2 va T3) con trong tnrong ho-p h~ phang dong quy ta co th~ chi I~p hai phu ong trinh can bang [cong thirc (12»). Nhirng thi du khac ve h~ sieu tlnh co th~ xem trong § 25.

Hinh 28.

R6 rang tinh chat sieu tinh lei do co cac lien ket thira, Trong thi du vira roi, d~ co can bang voi cac goc IX va ~ tuy y, ta chi din treo trong IUQ'ng tren hai day (xem bai t~p 2, hinh 19). Day thir ba & day la thira,

~ 13

GlAI cAc

HAl TOAN TINH HOC

41

Sau nay chung ta chi xet nhirng h~ tinh dinh, tire la cac h~ co so phan lire chua biet bang so phtro ng trinh din bang chira cac phan lire do. D~ tinh toan h~ sieu tinh, ta phai tinh den Sl,f bien dang cua cac vat, nrc la phai bo gia thiet ve cac v~t r~n tuyet doi. Cac bai toan 10<).i ay diroc giai n trong cac giao trinh sire ben vat lieu hay giao trinh tinh h9C cong trinh.

§ 13. Gh\i cac bal toan tinh hQC. Cac bai toan giai bang cac phiro-ng phap tinh hoc co th~ thuoc mot trong hai loai sau day: 1) bai toan dii cho tnroc (toan bQ hoac mot phan) cac lire tac dung len .v~t, yeu cau xac dinh & vi tri nao hoac voi nhirng h~ thirc nao giira cac lire tac dung thi v~t se can bang (bai t~p 6, 7) ; 2) bai toan cho bist chac chan lit vat can bang (hoac chuyen dQng « thea quan tinh »), yell cau xac dinh t;lt ca hoac mot scHl,fc tac dung len v~t do (bai tap 8, 9, 10 v.v ... ). Trong fat ca cac bai toan tinh h9C, phan lire lien ket deu la cac dai IUQ'ng chua biet.
Trong tinh toan ky thuat, ngiro-i ta giai cac bai toan tinh h9C d~ xac dinh dieu kien can bang cua cau true (neu cau true khong diroc lien ket chat che), cling nhir sire ep len goi diY, hoac cac irng lire trong cac bQ phan cau true khi cau true can bang. Vi diu true ta dang xet la t6ng hop cua nhieu v~t lien ket vo-i nhau, nen khi b~t dau giai bai toan, tnroc het din xac dinh xem nen khao sat S!J' can bang cua vt}t thd nao dd tim du o:c nhirng dai 1U'(J'ngchu-a biet. Qua trinh giai thea trinh tl,f sau day: 1. Chon v~t din khao sat can b~ng. D~ giai bai toan, ta bang cua v~t chiu tac dung cua cac lire dii cho va cac lire cac lire nro'ng diro'ng voi cac lire chua biet (thi du, muon goi diY, thi co th~ khao sat Sl,fcan bang cua v~t th6 chiu tac hrc cua goi diY co tri so bang sire ep can tim).

can xet s1)'can chua biet hoac tim sire ep len dung cua phan

Neu cac hrc dii cho tac dung len mot vat, con cac hrc chua bist lai tac dung len v~t khac, thi ta can khao sat Ian IUQ't Sl,fcan bang cua nrng vat, va doi khica Sl,f can bang cua cac v~t trung gian. 2. Giai phong v~t khoi cac lien ket, bi~u di~n cac IlfC lien ket da dll'Q'C giai phOng. Ta din ve rieng vat cac lien ket 1) (nlnr tren hinh 12b). Khi bieu dien ta phai chu y den cac dieu dii trinh bay ve chung IlfC da cho va cac phan diroc giai phong khoi cac phan hrc lien ket & § 4.

Il Khi dii quen thi co thi: tach v~t khao sat ra khoi ket diu rn<)t each tuo-ng tUQ'TIg va vi! cac II!C dii cho va phan luc lien ket tac dung len v~t 11 ngay tren hinh vi! tbng tlu~ (nhu tren hinh 31). Nhimg neu din khao sat din bang ella rn<)t v~t nao khac nira, thi nen tach va vi: v~t nay thanh m<)t hinh rieng.

42

H(TP LVC.

He LVC

D6NO

QUY

Ch. II

3. Lip cac dieu ki~n can bADg. Dang cac di~u kien din bang phu thuoc vao he hrc nao se tac dung len v{it khao sat sau khi giai phong lien ket va vao phuong phap giai nao se diroc ap dung (plnrong phap hinh hoc hay giai tich). Cac di~m c~n chu y khi thiet l~p di~u kien din bang cho cac he hrc khac nhau se diroc trinh bay 6- cac ph~ nrong irng trong cuon sach nay.
I

4. Xac djnh cac d~i hf'lfng dn tim, ki~m tra J(ri giM va khao sat cac ket qua thu dU'9'c. Dieu quan trong khi giai bai to~n Ia phai ve hinh ro rang va phai tinh toan theo trinh tl! hQ'P ly (hinh ve giup ta nhanh chong xac dinh each giai dung dtm, tranh diroc sai sot khi thiet l~p cac dieu kien din bang). Noi chung khi giai bai toan ta nen tien hanh titt cl phep tinh dum dang tbng quat (dai so). Khi do ta se diroc cac cong thirc xac dinh cac dai IUQ'ngchua biet cho phep ta co the phan tich cae ket qua thu diroc. Ngoai ra, nhieu khi tinh dum dang tbng quat con giup ta phat hien diroc cac sai sot qua vi~ kiem tra cac h~ thirc thir nguyen (thir nguyen cua m3i so hang 6- ca hai ve cua dAng thirc phai nhtr nhau), Neu giai dtroi dang tbng quat thi chl din di~n cac so vao ket qua cuoi cung. Trong muc nay ta se xet cac bai toan v~ can bang ella v~t the diroi tac dung cua cac hrc de>ngquy. D~ giai cac bai toan nay ta co the dung phtrong phap hinh hoc hoac giai tich.
a) PhU'O'Ilg phap hinh hqc. Phirong phap nay thuong tien ho-n. khi chi co ba ll!C tac dung len v~t (ke ca ll!c dii cho va ll!c c~n tim). Khi din bang, tam giac dung tren cac hrc do phai khep kin (can dung bAt dau t11'luc da cho). Sau khi giai tam giac nay ta se diroc cac dai IU'Q'ngc~n tim. ' b) PhU'O'Ilg phap gjM tich. Phuo-ng phap nay co tM su- dung v61 moi so hrong luc tac dung. De l~p cac di~u kien din bang, d~u tien phai chon cac true toa dQ. Trong tnrong hop he hrc phang de>ngquy, so di~u kien can bang Ia hai (cac cong thirc (12», con trong tnr~ng hQ'P he I\l'CkhOng gian Ia ba (cac cong thirc (11». Ta co th~ chon cac true toa dQ tuy y, nhtrng nl!u htrtrng mQt trong cac true vuong goc vtri mtJt lu-c chu-a bil!t nao do thi cac phirong trinh thu diroc se giai don gian hon. Tnroc khi I{ip cac di~u kien can bang, d~u tien nen tinh toan hinh chien cua t1it ca cac luc tren cac true toa dQ dii chon, ghi chung vao mQt bang (xem cac bai t{ip6, 10, 11).

Con mQt so chi d!n them se diroc trinh bay trong qua trinh giai cac bai toan dgm diiy .
, Hai t",6. M (It v~t co trong hrong, P nAm tren m~t phAng nghieng ohlin tao v(ri dUc7ng nAr.. ngang InQt g6c0: (hinh29a). Hay xac dinh tri s5cua I\I"CFsongsongvlri m~t phAngciin tac dl,mg
/

§ 13

GIAI cAc

BAI ToAN TINH HQC

43

vao v~t d~ gifr no cAn bAng va Ap 1\fC Q cUa v.t tac dung len m~t ph4ng.

81li glAi. CAc 1\fC c§n tim tAc dung len hai v.t khac nhau III 1\fC F tAc dung len v~t n~ng, con 1\fC Q len ~t ph4ng. ~ gihi bai toan nay, thay cho 1\fC Q ta se tim phan luc N Clla m~t phAng III 1\fC cO cung tri sll nhirng ngU"Q'Cchisu voi Q. Khi do cl IVC dii cho hl P IAn cac Iy-c F va N cAn tim di!u tac dung len cling mQt v.t. Ta xet sv can bAng caa v~t do. Muon v~y, hay nrong

p
(J

c
Hinh

29.

nrong da b6 diroc lien k~t (m~t phang), '/~t khi do IIi tv do (hinh 29b). Dung cac 1\fC tac dung len v~t, tire IIi 1\fC P, F va phan 1\l"Clien k!t N. ~ xac djnh cac 1\fC citn tim ta co th~ st'r dung cac di!u ki~ can bAng Clla v~t rAn tv do diroi dang hinh hQC ho~ giru tich. Hay xet cA hai phirong phap giAi do.

Phu-ong phap hinh. hoc. Khi ciin bAng, tam giac dung dll"Q'C tiI" cac 1\fC P, F va N phai khep kin. Ta dirng tam giac bAt ditu tir IVC da cho. Muon v~y, tiI" di~m tuy y a theo mQt t~ I~ xlch da chon, ta dvng 1\fC P (hinh 29c), roi qua di~m ditu va di~m mut caa 1\fC P dVng cac dirong thang song song voi cac IITc F va N. Giao di~m cua hai dll"l'rn~ thAng nay la dlnh thiI" ba cua tam giac IITc khep kin abc, trong do cac IITc bc va ca se la cac 1\fC citn tim (theo t~ I~ xich dll chon), Chii!u cua cac luc dll"Q'Cxac djnh theo quy tAc mill ten: vi 0- day hQ'P luc bAng khong, nen cac dliu mill ten khong g~p nhau tai mQt di~m.
Co th~ xac dinh tri so cua cac 1\fC citn tim bAng cach tinh ngay tren tam giac abc (khi do khong citn chu y d!!n t~ I~ xich). Chu y rAng..::.bca= 90° ,con ..::.abc= II, ta duoc F= Psinll , N= PCOSII Phu-ong phdp giai tich. Vi h~ Ivc tac dung III h~ phAng, nen chi citn chon hai true toa dQ, trong do d~ cho tinh toan diroc dO'D gian, ta lnreng true Ox vuong goc voi 1\fC clin tim N. Ta xac dinh hinh chieu cua cac 1\fC tren cac true x, y (xem bang) : I) Fk Flot FI\Y P Psinll - PCOSII

N 0 N trinh :

-F
0

Theo cac dii!u ki~n can bang (12) ta l~p cac phirong Psinll-

F= 0,

PCOSII

N= O.

Nho kheo chon cac true X,Y nen m3i phirong trinh chi chira mQt An so. GiAi cac phirong trinh nay ta dll"Q'c

F= Psin II,

N=

PCOSII.

1) Citn dii!n bang theo cQt, tire la ditu tien clin tim cac hinh chi~u cua hrc P tren ca hai true, roi d!!n hrc F V.v... Vi~ I~p tnroc cac bang rrhir thl! co th~ giam diroc xac suat mdc sai 111m khi I~p cac phirong trinh, d~c bi~t la 0- giai doan ban dliu khi ta con chira quen chieu cac hrc,

44

HQ"i>

t.irc.

HI;;

tire

D6NG

QUY
II

Ch. "

Ap 1l!Ccan tim rna v~t tac dung len m*t phang co tri sll bAng 1l!C N= P cos nhimg nguoc chi~u viri 111c N.

vim tim duoc,

Ta thlly rAng, d~ gifr v~t tren m~t phang nghieng, can tac dung mQt hrc F nho hon trong III'Q'ng P ella v~t. Nlnr v~y m~t pMng nghieng la mQt dung cu don gian nhllt cho phep dung 1l!C be hon d~ can bAng vo-i 1l1C Ion hon,

Nhir dii noi 0- dau § II, k~t qua thu diroc dung cho cl tnrong hop v~t dung yen IAn tnrong hop v~t chuyen dQng « theo quan tinh ». Do do, d~ day v~t chuyen dQng d~u len phia tren theo m~t phang nghieng ta chi can tac dung vao no mQt 1l!C F =.p sin II bAng h,rc citn d~ gifr v~t din bAng. Cling v~y, n~u mulln cho v~t roi di!u theo mij.t phang nghieng, ta can ham dan no bAng I\I'C F= P sin II. ~ v~t thuc hien duoc mQt trong hai chuyen dQng do, ta can truy!!n them cho no mQt v~n tile ban dau nrong irng. N~u nhtr khong co v~ tile do thi dll'<Yitac dung cua h,rc F= Psinll v~t se dung yen tai chO. Ap lee tac dung len m~t phl\ng trong moi truong hop di!u bAng P cos II.
Tir vi du filly co th~ rut ra mQt ket lu~n chung: trong cdc btli todn tinh hoc dU'(J'c giGi Mng cac phu-ong trinh can bang, ngU'ui ta thsrirng tim phan IJl'c lien k2t chu- khong tim tip IJl'c clia v(it ttic dung len lien k~t (phan 1l1Clien ket co tri sll bAng ap I\!'c, nhimg ngiroc chieu). Nhirng khi giM bai toan bang phirong phap phan I\I'C( § 7), thl trai lai ngll'Q; ta tim ap hrc tac dung lea lien k~t. Bai tip 7. Thanh AB dll'Q'C gtm vao glli t\!,a cll dinh bAng khop A (hinh 30a). Bau Bcua no mang mQt v~t nang P= 10kG va diroc gifr din bAng boi mQt soi day vAt qua rong r oc C, dliu day mang trong III'Q'ng Q= 14,1 kG. Tfl,IC cua rong r oc C va khop A cung nam tren mQt dirong thang dfrng va AC= AB. Hay tim gOc II va frng h!c 0- thanh AB khi he cfm bAng. Cho biet c6 th~ b6 qua trong III'Q'ng cua thanh va kich thirec cua rong rQC.

p/ c~ 9:(

6
R"

a
aJ

b)
Hinh 30.

C)

Ral glai. Ta hay xet 8\!, din Mng clla thanh AB dll'm tac dung clla tllt cll cac ll1C da cho va cac 1l1Ccan tim. Gilli ph6ng lien ket va xem thanh nhir la v~t t\!, do (hinh 30b), r6i dung cac luc tao dung len n6 : 1l1CP, bAng trong III'Q'Dgcua v~t treo, sire cang T clla day va phan 1\"CRA c6a khop huong dQC theo AB, vi trong tnrorig hop filly thanh chi co th~ bi keo ho~c nen (xem §4). Neu b6 qua rna sat giira day voi rong rQC thl sire cang cua day vAt qua rong fQC (khi v~t can bAng) 0- dau cling nhir nhau. Do do T= Q. Dung phirong phap hlnh hoc, ta dirng tam giac I\!,c khep kin cab bAng cac 1l1CP, T va R A (hinh 30c) bAt dliu tit I\I'CP. Theo tinh chllt d6ng dang gifra tam giac abc va ABC, ta suy ra: ca~o ab , L. cab= II. Do do R A = P • sin.!.2 = {L . 2P

* 13
vi T= Q= 2Psin
II

GlAI

cAc

BAI

ToAN TINH HOC

45

Y"

< 2P.

Til kl!t
Thanh

qua

vira thu dllQ'C suy ra : khi II < 180~ thi din bAng chi co th~ xAy ra neu Q < luc do bi nen voi I\I'C bang P, khong phu thuQc vao Q va g6c II.

Truo-ng hQ'P 11= 1800 din dllQ'C khao slit rieng. ~ dang thay rang trong tnro-ng hQ'P nay can Mng co th~ xAy ra vo; moi gia t.rj coa P va Q, trong do neu P> Q thi thanh bi keo b6-i I\I'C bang P- Q, con neu P< Q thi thanh bi nen b6-i I\I'C Mng Q - P. Veri cac so Ii~u dii cho trong bai toan, ngang). ta tinh dllQ'C R. = 10 kG,
11=

90° (thanh

nam

Ta chu y rang trong 1\I'CQ khong tham gia tree tiep vao di~u ki~n can Mng (vao tam giac hrc), vi I\I'C nay d~t vao v~t chii' khong d~t vao thanh AB rna ta din xet s" can bang, Sau nay, d~ giam bert hinh ve, ta se khong ve rieng v~t din khAa sat din Mng (nhe v~t t" do), nhirng khi dimg cac I\I'C tac dung vao no thi bao gier cling phiti tirong tirong la v~t da dllQ'c giAi phong, tire la b- dang nhir (rcn hinh 29b, 30b hay 12b. Oai tip 8. Cdn true vo; lien ket khop tru A va khop coi B mang trong luong P( hinh 31). Cho rang trong luong cua kl!t cau khong dang k~, hay xac djnh phan 11!CR.. R. cua cac goi. Cho billt clln true co titm voi Mng ! va AB=h.

Hinh 31. Rai gilii. Xet s" can Mng cua toan bl) din true dlleri tac dung cua 1\fC da cho va cac 11!Ccitn tim. Hay tll&ng nrong da giAi phong cac lien kl!t (khop A va B) va citn true da diroc t\l' do, r6i dung I\I'C tac dung P va phan 11!CR, coa khop tru co phiro-ng vuong g6c veri A B. Phan I\I'C R. cua khop co th~ co phirong tuy y tren m~t phAng hinh ve. Nhung citn true din Mng dllo; tac dr,lDg cOa ba I"c, do do cac diro-ng tac dung clla chung phai cdt nhau tai mQt di~m. D6 la di~m E - giao di~m cua cac dllerng tac dung clla I\I'C P R •. Do d6 phan 11!CR. hirong dQC theo BE.

va

se

Ap dung phuong phap hlnh hoc, ta d"ng tam giac khep kin abc Mng cac I\I'C P, R. va R ., bat ditu til I"c P da eho tnroc. Theo tinh chat d6ng'd<J.ng clla tam giac abc va ABE suy ra

R.
Do do P

h'
R.
=

R.
P

YJiT+I2
h I

R.

_!_
h

P,

JI

-Po h'

12

Theo tam giac abc ta thay chi~u cua cac phan I\I'C R , vii. R. tren hinh ve ia dung. Ap I\I'C tacdung Icn khop A va B co tri so Mng R. va R. nhuug co chi~u ngllQ'C lai. Cac ap l\I'c nay co tri so Ion neu ty so 1/ h Ion, Bai toan vira khao sat hi m{>t vi du lip dung dinh Iy ba I"c. Ta chu y ket lu~n sau day: neu trong bai todn dii cho tnrtrc kich thrro-c crUzcdc hQ phq.n co 'iu thi nen dung ding thrrc dong dang dJ giQi tam giac lu-c, con n~u cho cdc goc (bai tq.p 6) thi nsn dung cdc cong thirc hrong gidc. Oai t,p 9. Cho hrc nam ngang P tac dung vao khop A cua may ep true khuyu (hinh

46

HQ"'P i.trc. He i.trc OONG QUY

Ch. II

t36
p b;
C)

32a). 86 qua trong IITq-ng clla cac thanh va clla pittong, hay xac "inh ap Ivc clla pittong len v~t M khi cac g6c 4 va p cho tnroc. Bal gi~i. D1iu tien ta xet S\I' din bang cua khop A, chll bi l\I'c P duy nUt d!l cho tllc dung vao. N1!u xern khop true la v~t t\l' do, thi ngoai luc P,kh&p nay con chiu tac dung clla cac phan lvc clla cac thanh R, va R2 hirong dQC theo cac thanh. Dung tam giac I\I'C(hinh 32b). Cac g6c IP = 90° - « 'i' = 90' - p, y= «+ p. Theo dinh Iy sin, ta co : sinq>
--

R,

siny' sinfs+ p) 0) Bay gi<'r ta xet S\I' can bang cua pittong, N1!u xem pittong nhir v~t tlf do thi tren pittong ciing Hinh 32. co ba lire tac dung : ap l\I'c R',= - R, cua thanh AB, phan IIfC N cua thanh xilanh va ph an luc Q cua v~t bi ep. Vi co ba IIfC, nen khi din bang chung phai dong quy. Dung tam giac I\I'C tir cac I\I'C do (hinh 32c), ta tim dlTQ'C; Q= Thay R', b~ng RIta co :

= --

R,

Pcos «

R;

cos

p.
p
tg 4

P cos 4 cos ~ sin (4

+ P)

+ tg

Ap IIfC cua pittong len v~t M co gia tr] brmg Q nhirng co chieu nguoc lai, Tir cong thirc tren ta thay, voi cung I\I'C P, ap luc Q se tang len khi cac g6c 4 va p giam xuong. Neu cac thanh OA va AB co chieu dai nhir nhau, thi 4= ~ va Q= O,5PCtg4. Tir each giai tren ta rut ra kef luan : trong m{jl bdi 100innao do. hrc dii cho (ho(i.c cdc lire dii cho) co 1M uic dung ten m{jt v(i.t. con l!l'c can tim (ho(i.c cac hrc cill! tim) lai tac dung len v(i.t khdc. Khi do dii'u tien cdn xet W din Mng CUtI v(i.tthlr nhat va xac dinh .l!l'C ma v(i.tnay tac dung len v(i.1thlr hai, sau do mtri xet v(i.1thlr hai va xac dinh cdc dai 1U'(J'ngdin tim. Bal tfP 10. MQt gia diY diu tao boi cac thanh AB va BCdlTQ'c lien kilt voi nhau va viri nnmg bang cac khop true. Tai diem B cua gilt gan mQt rong roc (hinh 33a). Ngiroi ta, • vAt qua rong rQC mQt soi day, mQt d1iu buoc vao nrong, con dau kia mang trong IITQ'ng Q. 86 qua trong IITQ'ng cua cac thanh va kich thiroc cua rong roc, hay xac dinh phan hrc cua cac thanh theo cac goc « vii p cho tnroc, Bai gill. Xet SIf can bang cua rong roc voi doan day DE I). 86 cac lien ket va thay chung bang cac ph an IIfC nrong irng (hinh 33b). Khi do tren rong roc co doan day co bon

,) Trong cac tnro-ng hop urorig t\l' nen coi rong roc cung voi doan day hi mot v~t. Khi do cac ap lire tiro-ng hll chua billt trtroc giira day va rong roc phsn bi) thea cung de tao thanh he hrc can bang ben trong vii khong tham gia vao dieu kien can bang (xem §3, h~ qua tien de 4). Neu nhir xet rieng rong roc (hinh 33c ; ty I~ ve tren hinh thay dbi) thi rong roc se chiu tac dung cua cac ap IIfC phan bi) thea cung de cua day. Nhw v~y ta phai xac dinh hQ'P hrc cua chung, bang each xet them dieu kien can bang cua doan day DE (bang nguyen Iy hoa ran). Tinh toan do do se dai hon. .

~ 13

GlAI cAc BAI ToAN TINH HQC

. 47

b)
Hinh 33.

C)

hrc ngoai tac dung : sire cling cua nhanh day ben phai bAng Q, sire ding cua nhanh day ben trai T ciing co tri so bang Q (T=. Q) va cac phan hrc R, va R2 cua cac thanh co hirong doc thea cac thanh. Neu b6 qua kich thuoc cua rong roc, thi cac hrc nay d6ng quy. Vi so cac hrc lori hon ba, nen ta'se dung phirong phap giAi tlch, Dung cac true toa dQ nlur tren hlnh vi: va tinh hinh chieu cua tlit cA cac luc tren cac true nay (xem bang),

Fk

Q 0

T -~ Tcos ~ Tsin ~

R, R, sin R, cos
II II

R2 - R2 0

Flo..
1---.

hy

-Q

Sau do, ap dung cac dieu kj~n can bAng (12), ta I~p cac phiro-ng trinh nrong irng. Thay T bilng dai 1U'Q'ng Q ta dU'Q'c : - Q cos ~ + R, sin II - R2 = 0, - Q + Q sin ~ + RI cos II'~ O. Til' phuo-ng trinh thir hai, ta tim diroc : RI I-~ sin ~ "" -cos II Q.

Thay tri so nay vito phuong trinh thir nhat, sau khi bien dbi ta duoc : R 2 = Q Sill II - cos (11-' cos II ~).

Til' bi~u thire cua R I suy ra, voi bat ky goc nhon n110 cua II vii ~ thi R 1>0. NhU' v~y co nghia Iii phan lire RI bao gi<'Y ciing co chieu nhir tren hinh ve Ar hrc ella rong roc tac dung len thanh co chieu ngiroc lai (thanh BC bi nen). Dbi \'iri R, ,<> co ket qua k hac, Cho rang II vii ~ bao gio- ciing 1:1cac g6c nhon. Vi sin 11- cos (11- ~) = sin II - sin (90"_ a,+~); nen hieu nay seco gia tri dirong neu II> (900 - II + ~) hay 211>900 + ~.Til' do suy ra neu II;> (45" + ~/2) thi R2 > 0, nrc 1:1phan IVC R2 co chien nhir tren hinh ve, con neu II < (450j + j3/2 thi R2 < 0, nrc Iii phan luc R2 co chi!!u ngiroc lai (til' A den B). NhU' v~y. thanh AB

48

HO"P r.trc. H~ i.trc D6NG

QUY

Ch. II

trong tnrong hop dltu bi keo, con trong trtrong ho-p sau bi nen. Khi at= 45° + ~/2, ta co R2°~ O. Ta cltn IU'u y cac k~t lu~n sau : 1) Neu trong h~ cac rong roc co day vAt qua thi khi I~p cac dieu kien can bang ta nen coi rong roc cimg vui doan diiy lien quan la m9t v('it. Khi do, neu b6 qua rna sat gifra day voi rong rQC h()~c rna sat & true rong rQC thi su-c cang 6- cd hai dfiu day se co tri so Mng nhau va hirtrng tir rang roc ra ngotii (neu khong thi day se bi keo ve phia co sire cang Urn hon hoac rong roc se quay; xern them bai t~p 13). 2) N~u khi ve cdc phan hrc lien ket ma m9t phan lire nao do khiing co chitu nhu: trong thirc t~, thi khi gidi Mng phu o ng phap hinh hoc se tMy ngay dU'fYC diem do tren da gidc lire (quy tdc mill ten), con n~u gidi bilng phu ong phap gidi tich thi dl}{ iU'fYng da phdn lu-c do co dl1u am.

Noi chung, n~u co th8 him diroc ngay tir dltu thi nen vi! cac phan hrc lien ket theo dung chieu cua chung. Thi du ta co the xac dinh chieu cua phan lire khop true A trong, bai t~p 8 bang each I~p lu~n nhir sau : n!!u b6 khop true, thi dtroi tac dung cua hrc P clin true sil nghieng sang phai, Nen h.rc R thay cho khop true d~ gifr cltn true can bang phai htrong sang trai.
A

Bai tip 11. CQt OA chon thang dirng xuong diU dtroc gifr bang cac day chang dQC AD va AD tao thanh voi cQt cac g6c bAng nhau va bang (1= 30°. G6c gifra cac m~t phang AOB va AOD bang Ijl= 60° (hlnh 34). Nguoi ta buoc vao cQt hai SQ1day chang ngang vuong g6c voi nhau va song song v&i cac true Ox va Oy voi sire cang m6i day P= 100 kG. Hay xac dinh ap hrc thang dimg tac dung len cQt va sire cang trong cac day chang.Cho bi~t trong hrong ella chung khong dang k~. Bai giAi. Xet S\l' can Mng cua nut A, noi buoc cac day chang ngang va chang doc. Tai A co cac hrc cang P, va P2 cua day chang ngang (P,= Pz= P),pMnl\l'c Rz va R3 cua cac z day chang doc va phan I\I'C R, cua cQt. H~ hrc la h~ khong gian. Trong tnrong hop nay chi nen ap dung phuong phap giai tich. Dung M true tea dQ (xem hinh vi:) va tinh hinh ehi1~u cua cac hrc len cac true, rlli dien vao bang (hinh chieu cua lire RJ tren cac true x va y tinh theo S\l' chi dan & cuol § 8).

S& dung dii!u kien can bang (ll),


cac phirong trinh :

ta I~p diroc

_. P+ R3sin(lsin!p= 0, -- P + Rz sin (I + R3 sin (I cos !po= 0, R,-· Rz COS(I- RJ COSat = O.


Giai cac phtrong trinh

.r
Hinh Fk Fkx 34.

R3

P
sin at sin
=P
Ip ,

R,

(1 + tg If) 2
Rz
()

nay, ta tim diroc : I - ctg !p Rz = P sin at

as

at.

P,
0

Pz

R,
0 0 R,

R3 R3 sin at sin e R3 sin at cos e - R3 cos at

-P
0 0

Fky F~z

-P
0

Rz sin at
-Rzcosat

§ 14

MOMEN CVA t.trc DbI Val

MOT TAM

49

Vo-i k~t qua thu duoc, ta thay khi cp<45° thi R1<O, va phan II)'CR2 co chieu ngiroc vo-i chi~u th~ hi~n tren hinh ve. Vi day chang khong th~ chiu nen, nen day chang AD phai bo tri sao cho goc cp Ion hon 45 Thay tri so vao bai toan, ta diroc : R3 = 231 kG, Rl = 85kG, R) = 273kG.
0 •

§ 14. Momen e~a life dl)i v{ri mQt tam (bay mQt "i~m). Thirc t~ cho thliy la khi tac dung len v~t the, hrc co the lam cho v~t th~ chuyen d9ng tinh tilln, d3ng thoi lam cho no quay xung quanh mot Him nao do. Tac dung quay cua Il!c diroc d~c tnrngboi momen cua no.
Xet lire F tac dung vao diem A cua v~t the (hinh 35). Gia thir Il!C co xu hiro-ng lam v~t quay quanh Him O. GQi dirong thing goc h ha t~ Him lire F la tay don cua lire F doi voi tam O. Khi do, vi co the tuy y di chuyen diem d~t hrc doc theo dirong tac dung, nen tac dung quay cua hrc si! phu thuoc vao : 1) tri so cua lire F va chieu dai tay don h; 2) vi tri cua m~t phang quay OAB qua tam 0 va Il!C F; 3) chieu quay tren m~t phang nay.

toi duong tac dung cua

mo(F)=+FIJ
U}

b)

Tnroc h~t ta hay xet h~ cac hrc cung nAm tren mot

Hinh 35.

m~t phang, Khi do, flit ca cac lire d~u co chung mot m~t phing quay, con chieu quay co the xac dinh bang dau dirong n€!u quay theo mot chieu quy iroc nao do va dliu am n€!u quay theo chieu ngiroc lai. Khi do, de do dinh IU'Q'Ogtac dung quay, ta co dinh nghia sau day vemomen luc : momen da lu-c F doi wh tdm 0 fa m(3t dai hrong co dau tiro ng trng va Mng tich tri so da hrc vtri chieu ddi tay don. Momen cua hrc F doi vo-i tam 0 diroc ky hieu la mo (F). Do do mo(F)= ±Fh. (13)

Sau nay chung ta quy iroc momen-ce dau diro-ng, n€!u.ll!c co xu hirong lam cho v~t quay xung quanh tam 0 ngiroc theo chi~u kim dong ho va co dliu am n€!u luc lam v~t quay thuan chieu kim dong ho. Thi du, doi voi lire F vi! tren hinh 35a thi momen doi voi tam 0 mang dliu c9ng, con momen cua lire vi! tren hinh 35b mang dliu tnr. N€!u tay don do bang
4--731

50
met, thi momen (kGm). Momen 1) Momen tao dung.

H<;TP

urc.

He

i.irc

DeiNG

QUY

Ch. II

hrc do bang Niuton-met

(Nm) hay bang kilogam-met

luc co cac tinh ch~t sau : lire khong phu thuoc vao dil!m d~t II!C dQC thea duong

2) Momen I\1'Cdai voi tam 0 ha~ng khong chi khi nao I\1'Cbang khong ho~c dtrong tac dung cua I\1'Cdi qua tam 0 (tay don bang khong). 3) Momen (hinh 35b) luc co tri sa bang hai Ilin dien tich cua tam giac OAB rno (F)
=

2dt 6 OAB.

(14)

Ket qua nay suy ra tll' cong thee : dt 60AB


=

'2

AB .h

'2 Fh.

§ 15. Dinb Iy VarinhOng v~ mom~ cJ\a hQ'p Ivc. Chung ta se clnrng minh dinh Iy Varinhong ') sau day: momen cua hop hrc coo h¢ ifL'c phdng d{j'ng quy doi voi bIlt ky tam mio Mng t6ng .x t dai cdc momen coo cdc luc thdnh. phifn
do; voi cung tdm do.
Gia thiet h~ hrc F 10 F2, ••• , F n dong quy tai diem A (hinh 36). Uiy mQt tam 0 b1it ky va. dung qua 0 true Ox vuong goc voi dirong thing ~A. Chi~u duong cua true Ox se dtrQ'c chon sao cho hinh chieu cua ml>i Ivc tren true do cung d1iu voi d1iu cua momen hrc doi voi tam o.

so

'i.

chjmg minh dinh Iy, ta tim nhimg bieu thirc urong irng cua cac mornen Hinh 36 rno (F I), mo (F?) v. v... Thea cong thirc (J 4) tacomo (F1) = + 2dt60ABI. Nhirng nlur tren hinh ve, 2 dt 6 DAB, = OA.Ob =, OA.Flx, trong do FIX hi hlnh chieu cua hrc F 1 tren true Ox. Nen rno (F I) = OA. Fix' (15)

De

TU'O'ng tv nhir tren, ta tinh duoc momen cua cac hrc khac, Cong thirc (15) dung doi veri ca tnrong hQ'P khi Ivc F nam ben diro-i dirong 0 A ;

,) P. Varinhong (1654-1722) Iii nha bac hoc, nha toan hoc vii nha coo hQC 16i III-cngtroi Phap. Ong dii trinh bay coo so. cua tinh hQC trong quy!n « D~ ciro'ng cua coo hQC moi » (1687).

~ 16

PHlrO"NG TRiNH MOMEN ellA

Ht; LVC neiNG QUY

51

momen cua h,c ~hi do co diiu

am

vi chinh hinh chi~u F; co diiu

am.'

, GQi R hl hQ'P lIrc cUa cac hrc F 1> F 1, ••• , F n, trong do R = Khi do, theo dinh ly ve hinh chieu cua tOng 1\I'C true, ta co Rx = tren Nhan ca hai vl! cua dang thirc nay v~ OA, ta dllQ'C:

LF

LF k.,
b.

OA . Rx hay thea cong thjrc (15)

L (OA.Fb) Lfflo

rno (R) = (F ((). Cong thirc (16) la bieu thirc toan hoc cua dinh

(16)

Iy

Varinhong.

§ 16~ PhU'O'ogtrloh mlimed ella h~ 1\I'Cding quy. Khong nhirng ta c6 th! bi~u di~n dii!u ki~n din bang (giAitich) cua M It,re d6ng quy qua cac hinh chieu cua no rna con bi~u di~n qua momen cua chung nua. Ta chirng minh rang, d~ din bang he hrc phAng d6ng quy cin va chi cin tht,re hien cac di!u kien : (F 0 = 0, (F tJ = (17) trong do B va CIa esc d'i~m tuy y khong nam tren cung dirong thftng vo; di~m A lit di~m d6ng quy efta ci.c It,re (hinh 37).

Lm.

Lmc m.

0,

Lm.

Di!u kien cAn hoan toan hi~n nhien vi chang han, neu

(F 0 thi ,til' (16) ta thily (R)7t 0, nghla lit R va do do khong th~ co S\I' din bang.

'* °

* 0,
Hinh 37.

Ta chirng minh dieu kien duo Neu cac di!u ki~n (17) dll'Q'r.:

thee hien thi theo djnh IY· arinhOng m. (R) =" va me (R) = O. Dieu nay chi co dU'Q'C V khi ho~c la R = 0, hoac la dU'CYng dung cua I\I'~R d6ng thoi di qua cac di~m B va C. .Nhimg tac trong tnrong hQ'Pdang xet, khong th~ xAyra khA nang thir.hai, vi hQ'PIt,recua M It,red6ng quy pMi di qua di~m A (hinh 37), con dll'Crng thAng BC theo dieu kiett dinh Iy khong th~ di qua A. Do d6, neu cac dieu kien (17) dll'Q'Cthuc hien thl hQ'P Ivc R = 0, tile lit he It,redin bang thee S\I', R5 rang la mulln can bang neu chi co mQt di!u kien nao d6 cua (17) dU'Q'Ctht,re hien thl vAn chua duo Neu dung cac di!u ki~D (17) d~ giAi toan, ta co th~ I~p dll'Q'Ccac phirong trinh rna mlli phirong trinh chi chea mQt An. Mulln v~y phlti lily cac tarn momen teen cac dll'Crng tac dung cua cac Ivc cin tim. Bit
momen.

t.,

12. Hay giAl bai ~p 7 Mng phll'O'Ilg trinh

Bai giM. Ky hieu AB = AC = a va lily cac di~m A va C lam tam momen (liinh 38). Til' A k~ cac dll'Crngthing goc AE va AK vlri cac dll'Crng tac dung cua hrc T va P,
a cos.!., AK = a sin IZ • <1. d" 2 ' Hinh 38. m; (T) = Ta cos..!.., m; (P) ='. - Pa sin IZ. Ngoai ra, 2 m; (R = 0. Cung nrong t\l' nhir v~y, ta tinh dll'Q'Cmomen cua cac It,re dlli vlri tam C. Thi~t l~p cac dieu kien can bang (17), ta dll'Q'C:
=
I A)

ta tim dll'Q'C: AE

4*

52

H~ LIJC SONG SONG vA Ht NGAU TI/ cos.!.. 2;mc Vi T


=

Ch. III = 0,

Pa sin Pa sin

CI

(a) (b)

(F

0 "" R,

a sin

CI -

CI =

O.

Q, nen theo phuong trinh (a) ta co :

(Q - 2P sin.!..) cos.!.. 2 2
Nhir v~y ta xac dinh dircc hai tri s3 cua g6c
CI = CI

o.

irng voi vi tri din bang hi :

180° va sin.!..

Tir phuong trinh (b) khi

CI

* 180°,

2P

Q.
= P.

ta tim dll'Q'C R,

Oai t~p 13. Tim phan II!~ R2 trong bai t~p 10 bang phuong trinh momen. Oai gia\i. Lay momen d3i voi tam C (xem hinh 33b) va ky hi~u CB = a, ta co :

lin ( (F k
Tir day,

= Ta

COS(CI-

~)

R2a

COSCl-

Qa sins = O.

vi T = Q. ta lim thay ngay :

R2

sinCl-

COS(CICOSCI

~).

Ta co th~ xac dinh phan II!C R I bang each lay momen Ta thay cilng co th~ dung cac phirong giai bang cac phuong phap khac.

doi vo; tam A.

trinh momen d~ ki~m tra kISt qua cua cac each

Ta chu y rang co th~ ki~m tra dang thirc T = Q bang phirong trinh momen d3i voi tam rong rI,)C(nhtr chirng minh b § 24, phuong trinh nay dung d3i vo; he h,rc kh6ng d6ng quy). Khi do, ta dtroc Tr - Qr = 0, trong do ria ban kinh rong roc ; nen T = Q.

se

se

ca

CHUO'NG III

H~ LVC SONG SONG vA ~ NGAU TREN MQT M!T PRANG

§ 17. Ho-p va phin cac hre song song. Ta se xac dinh h9'P lire cua hai lire song song tac dung len v~t ran. 6 day co th~ co hai tnrong hop : I) cac lire cung chieu va 2) cac hrc khac chieu.
1) Hcyp hai hre eling ehi~u. Gia thu- co hai hrc song song F 1 va F 2 tac dung len v~t th~ (hinh 39). Dung cac tien ~ 1 va 2 cua tinh hoc, ta co th~ chuyen tir h~ lire song song nay sang h~ lire dong quy tirong dirong Q 1 va Q2' Muon vay, ta d~t vao cac di~mA va B hai lire can bang PI va P2 (P I = -- P 2) hirong dQCtheo dirong thang AB, roi cong chung V<Y1 cac lire F I va F 2 theo quy tlic hinh binh hanh. Sau do r<'ricac lire Q 1 va Q2 vira thu duoc v~ di~m 0 la giao di~m cua cac duong tac dung cua chung, rai phan chung thanh cac lire thanh phan ban d~u. Khi do, tai diem. 0 se co hai lire P I va P 2 can bang nhau, nen co th~ loai bo, va hai hrc F 1 va F 2

§ 17

HQ"P vA PHA.N cAc LVC SONG SONG

53

huong theo cung mQt dirong thang, Roi cac luc nay v~ di~m C va thay chung bang hop hrc R voi tri so bang :
\

= F1

F2.

(18)

Lire R chinh la hQ'P hrc cua cac hrc song song F 1 va 1"2 d~t tai cac diem A va B. De xac dinh vi tri diem C, ta xet cac tam giac OAC, Oak va OCB,

Omb. Theo tinh chat d6ng dang cua cac tam giac do, suy ra:
AC OC

p-

(.-~~-~

-vaF
1

,BC OC

=_.
F2
=

P2

hay AC.F1
Chu

c= BC.F2

vi P1

P2

F1

ty l~ thirc va tinh BC + + F2 = R, ta duoc :

y toi tinh chat

cua cac AC = AB, R (19). Hinh 39.

V~y: hop lire cua hai lire song song cung chieu tac dung len vdt tMco tri so bang t6ng tri so cua cac lire thann phdn, co htrong song song va cung chieu vo-i cac lire thanh phdn; du-ong uic dung coo hirp lire di qua diem nam girra cac didm dilt coo cdc ltrc thdnh phdn vd chia doan do thanh cdc phdn If nghich voi cdc lire thanh phdn.

(v

2) Hep hai l\I'c khac chi~u. Gia thir


co cac hrc F 1 va F 2 tac dung len v~t the, trong do de d~ xac dinh, ta coi FI >F2 (hinh 40). Tren duong keo dai cua doan BA ta Uiy diem C va tac dung vao do cac lire R va R' can bang nhau va

song song voi cac hrc F 1 va F 2' Trong tnrong hQ'P nay ta lliy tri so cua lire va vi tri diem C sao cho thoa man cac d~ng thirc :
R Be F,
=

Hinh 40. (20) AB (21)

F1-F2; AC

54

He t.irc SONG SONG vA HI;: NGAU

Ch.

III

Rai cong F2 voi R', theo cac cong thirc (18) va (19) ta dU'Q'Chop h,c Q co tri so bang F2+ R',t(rc la bang F h va d~t tai diem A. Vi cac h,rc F 1 va Q can bang nhau nen co the loai bo, K~t qua la cac h,rc da cho hl F I va F 2 dl.t<rc thay Mng mot hrc R hl hop luc cua chung, Tri so va diem d~t C cua h9'P b,rc nay dU'Q'Cxac dinh theo cac cong thirc (20) va (21). V~y : ho-p lire cua hai ltrc song song ngtrtrc chieu tac dung len vdt tM co tri so bang hifu tri so cac lire thdnh phtin, co huurng song song Cling chitu vtri lu-c krn hon ; du irng lac dung cua hop lire nam ngoai doan tilting no; cdc d;~m d{it cua cdc lu-c thanh phdn, 0- each cdc di~m do nhirng doan tf If nghich

coo

va

voi cdc ltrc.


Neu co len v~t, thi cua chung dung lien hoac bAng & chirong nhieu luc song co the xac dinh (neu co) ho~c tiep quy tAc plnrong phap song tac dung diroc hop h.rc bAng each ap cong hai hrc, se gioi thieu

. 3) Phan ."c. Dung cac cong thirc vira thu duoc ta co the giai bai toan phan mQt h,rc da cho thanh hai h,C song song cung chitSu ho~c ngiroc chieu nhau. Bai toan se xac dinh n~u co them cac Hinh 41. di~u ki~n phu (thi du cho tnroc cac duong tac dung cua hai hrc din tim hay cho tnroc tri so va dirong tac dung cua mot hrc thanh phan).
Bai ttp 14. Nguoi ta ngam dam AB dai.I = 2.5m vao nnmg co b~ day a = O,5m (hinh 411. & dau B treo vi,it nij.ng P 3 T. B6 qua trong leong cua dam, hay xac dinh cac ap hrc 1011urong, n~u cac ap h!c nay dij.t tai cac diem A va D (dam hoi bi lech). Rai giAi. D~ xac djnh ap h,c, La phan l\l'c P thea cac hiro-ng cua phan 1\I"C& di~m nra va A thanh cac hrc Q n va Q Vill,TC P kh6ng nam gifra cac Il,Tcdin tim, nen cac hrc nay runroc chieu nhau va trj so cua hrc Q n (gdn P hon) se km hon Q va co chieu nhu P. Theo c;lL' dAng thirc QD P . va P= Q." - Q,
f)
A' A

IV.

1;1

tim diroc :

QD ~

.~

/
{/

P = 15 T, Q, = 12 T.

kiell1 tra bai giAi, ta li,ip ty .

so

thir hai:.Jh_ /- a

!...
a

§ 18. Nglu ."c. M6men nglu ."c. Ngi\u hrc hi h~ hai hrc co cung tri sa, song song va ngiroc chi~u nhau tac dung len v~t the (hlnh 42). H~ h,e

§ 18

NGAU

LIre.

M6MEN

NGAU

t.trc

55

tao thanh ngAu hrc ra rang la kh6ng can bang F nhau (xem tien di! I), trong khi do, khac voi cac h~ da khao sat & tren.ngdu hrc kh6ng co hop hrc. Thuc v~y, gia tht'r ngAu lire (F, F') co hop hrc Q =1= O. thi luc Ql = - Q phai can bang voi ngAu Il,I'C nay, nrc la h~ cac hrc F, F', Ql phai can bang, Nhung.nhir ta se chirng minh, muon can bang bat ky h~ hrc nao thi phai lam sao cho t6ng hinh h9C cua chung bang Hinh 42. kh6ng. V~y, voi gia thist vira reli, ta phai co F + F' + Q 1 = O. Nhimg dii!u nay kh6ng th~ co diroc, vi F + F' = 0, can Q 1 =1= O. Nlnr v~y khong th~ thay hoac can bang ngAu Il,I'Cbang ITI(>t luc. Vi the, ta phai nghien ciru rieng cac tinh chat cua ngAu luc nhu ITI(>t dai hrong d~c biei ve SQ' nrong tac co h9C gifra cac v~t th~.

"

M~t phang chira cac dirong' tac dung cua cac luc ella ngAu hrc goi la m(il phang uic dung cua ngdu lu c. Khoang each d giira cac dtrong tac dung cua cac hrc cua ngAu luc goi la lay don cUa ngdu lu:c. Tac dung cua ngAu hrc dai voi v~t th~ th~ hien & hi~u qua quay nao do rna hieu qua quay ay phu thuoc vao : J) tri so cua l\fc F cua ngAu luc va chieu dai tay don d; 2) yj tri cua m~t phang tac dung cua ngAu hrc ; 3) huong quay trong m~t phang do. D~ d~c tnrng hieu qua nay, ta dira ra khai niem ve momen ngAu hrc, Trang chuong nay ta se khao sat cac tinh chat cua cac ngAu luc cung nam tren mot m~t phling. Dai voi tnrong hQ'P nay, nrong t\f nhir khai niem momen lire (§ 14) ta co dinh nghia sau : momen ngdu It,rc liz dai hrirng co diiu ttro ng zrng va bang tich tri s(j cUa mf)l hrc cUa ngdu It,rc vo-i lay don cua ngdu lire I). Ta se ky hieu momen ngjiu lire bangchir m ho~c chir M. Khi do : m = ±Fd._J. (22)

Cting nhir momen lire, momen ngdu hrc se co dllu dirtrng neu ngdu lire co xu hirtrng lam vdt quay ngtrirc chieu kim dong hO viz co dliu am neu ngdu ngdu lire lam V~I quay theo chieu kim dong ho~ Momen ngAu hrc ciing co don vi do nhir momen lire. Tren hinh 42, ta thay momen ngAu hrc bang momen cua m(>t luc cua ngdu luc doi voi diem d~t cua luc kia, nrc la m

= mR (F)

m, (F').

(23)

,) Khong nen IAnIQnkhai ni~m nay vtri khai niem momen hrc, Khai ni~m momen hrc c6 lien quan vtri di~m rna ta Illy momen, con momen ngi\u h,rcchl xac djnh thea cac luc vAtay don c6a ngi\u l\I'c, khOng c6 lien quan gl vtri di~m nao tren ~t phAng. NM C<Y hoc vAnha hinh hoc n6i tii!ng ngl101 PMp L. Puanxo (1777 -1859) IAngu01 kh6-i xuong thuyet • ngi\u hrc.

56

He i.trc

SONG SONG vA

Ht

NGAU

Ch. III

Ta chimg minh dinh Iy sau day ve momen cua cac hrc ngau lire : tong dai sa momen cua Cal' lire ng.-Iu lu-c d15i vo-i btu ky tani ntio nam tren mdt phdng tac dung cua ngdu hrc khiing phu thuoc VelO vi trl cua tam do vel bdng momen ngJu lire. Thuc v~y, F neu Hly bat ky diem 0 nao hen mat phang tac dung cua ngau hrc (hinh 43) ta co m., (F) = - F.Ga; m., (F') = F'. Ob. Cong cac ding thirc do lai va chu y la F' F va Ob - Oa= d, trong do d la tay don cua ngau hrc, ta co :
cc=

F'
Hinh 43.

rno (F)

mo (F') =c m,

(24)

Dinh Iy vira chirng minh Slr dung lhu~n ti~n khi tinh mornen cua cac hrc ngllu lire doi voi bat ky tam nao,

§ 19. Sl}' tu-ong dtrO'og coa cac oglu ll}'c.


tren cling mdt phang

nrong dirong cua hai ngau lire, dl1u tien ta clnrng minh dinh ly : co thi; thay the ngJu lire uic dung ICn V{jt tM bang Mt ky ngdu lire nelO khac nclm
vel co cung ma men mel khong kim thay d6i tac dung . dlii vtri vdt thi. Gin tlnr ngjiu lire (I<', F') co tay don d, tac dung len v?,t th~. Tren m~t phang tile dung cua ngau hrc, ta dimg qua hai lJ diem D 'va E hai dirong thing 4-~song song, chung ca.t cac dirong tac dung cua cac hrc cua ngjiu p, __ lire (F, F') tai cac diem A va B .~~.~E P (hinh;)lD~t cac hrc F va,F' vao cac m do (luc dilu cac hrc F va F' co the d~t 6- bat ky diem nao tren cac dirong tac dung cua Hinh 44. chung). Goi khoang each gifra cac dirong thang AD va BE la d2• Bay gio-, phan I\l'C F thanh cac lire Q va P thea cac hirong BA va DA, va h,rc F' thanh cac hrc Q' va P' theo cac hiro-ng AB va BE. R5 rang khi do P = - P', Q c= - Q'. Cac hrc Q va Q' can Mng nhau nen co the loai boo K~t qua la ngjlu hrc (F, F') se diroc thay th~ bAng ngjiu lire (P, P') voi tay don khac va vo-i cac hrc khac rna ta co th~ d~t 6- cac di~m D, E tren cac dmrng tac dung cua cluing, 0- day vi co the chon cac diem D, Eva chieu cua cac dirong thang AD . va BE mQt each tuy y, nen co the d~t ngau hrc (P, P') 6- bat ky vi tri nao

De

thiet I~p dieu kien

L,

§ 19

SI)· Tl!UNG

Dl!UNG

CVA cAc NGAU

LI)"C

57

tren m~t phang tac dung cua no (co the dira ngau hrc v~ vi tri rna 6- do cac hrc P va P' song song voi F bing phep bi~n dbi hai Hin noi tren). Cubi cling ta clnrng minh ring mornen cua cac ngAu hrc P, P' va F, F' bing nhau. Thirc vay, vi lire F la hop lire cua cac hrc P va Q, nen thea dinh Iy Varinhong : 111 II (F) = m" (P) + III II (Q). Nhirng mIl (F) =c Fd 1, mIl (P) = Pd2, mIl (Q) = 0, do do Fd 1 = Pd2, tire la momen cua cac ngjiu II,l'C bing nhau. NhU' v~y dinh Iy da diroc chirng minh. Til" dinh Iy tren suy ra cac tinh chat sau day cua ngau hrc : I) Co the di chuyen ngau h!c d~n bat ky vi tri nao tren m~t phang tac dung rna khong lam thay dbi tac dung cua no doi voi v~t the. 2) Co the tuy

thay dbi tri so cua cac lire hoac chieu dai tay don rna

khong lam thay dbi tac dung cua ngiiu hrc doi voi v~t th~ neu giir momen ngjiu lire khong dbi. Til" day suy ra, hai ngau hrc nam tren cung m~t phang va co momen nhir nhau se tiro-rig dirong nha u, b&i vi bing cac phep bien dbi neu tren (tire la Mng each thay dbi tay don va vi tri cua ngau lire tren mat phang tac dung), chung co th~ bi~n dbi cai no sang cai kia. Dang thoi, til" tat ca cac dinh Iy vira chirng minh 6- tren, ta thay rang tac dung cua ngau hrc doi voi v~t th~ thirc su -ducc the hien bang mornen ngjiu lire.

Hinh 45.

D~ dinh ra mot ngjiu lire tren m~t phang dii. cho chi din dinh ra momen cua no can cac hrc cua ngjiu hrc hay tay don cua no bang bao nhieu va ngau lire nim 6- vi tri nao tren m~t R phang tac dung, dii!u do khong quan F trong. Cho nen, d~c biet trong ky /I thuat, ngiroi ta thuong bieu di~n ngau lire bang miii ten tron chi huong quay, chir khong bieu di~n cac hrc (thi du, cac mili ten tren hinh 45 th~ hien lire F va ngau \vc co momen E <' m tac dung len v~Q.)
Bay gio- ta chirng minh djnh Iy [hit hie ~ng coo ngilu [we diJi vui vdt 1M khong d6i neu chuyen ngi'Iu [we tir mdt phling nay blil ky mijt phling nao khdc song song vui

,.'

hai:
thay sang no.

F,'
f

Hinh 46.

58

H~ LVC SONG SONG vA H~ NGAU

Ch. III

XCI ngllu I\I'C (F, F') nAm tren m~t pha.ng I (hinh 46). Ta ve m~t phAng II song song vai m~t philng L r6i tren m~t phAng d611iy doan DE bAng va song song voi AB. Tai cac di~m D va Eta d~t bon l~rr lirng doi mQt din bAng nhau sao rho F 1 = F 2 = F; Fi = F = F'. Ta th1iy rAng ABED la hinh binh hanh, cac dirong cheo cua n6 chia thanh hai phdn bAng nhau va cat & giao di~m C. Bay gia cc;;ng cac I\I'C song song F va F 2 voi nhau. Vi chung co tri sa bAng nhau nen chung duoc thay bang hop h,c R d~t tai trung di~m cl'Ia doan tha.ngAE, tfrc hi tai di~m C, trong de R = 2 F. Cac 1\I'C F' va F' 2 sau khi cong lai ciing dlTQ'Cthay bAng hop 1\I'C R' d~t tai trung 6i~m cl'Ia doan thang DB, tfrc la ciing tai di~m C, trong d6 R' = 2F' = R. Nhu v~y cac I\I'C R va R' bang nhau, nen co th~ triet tieu. Ket qua la ngllu I\I'C (F, F') thuc S\I' dlTQ'Cthay bang ngllu h,rc ( F .. F'I) nAm tren m~t phing II.

Tir cac dinh Iy tren ta suy ra : hai ngllu I\I'C HAm tren cac m~t phAng song song va co cung mo~en thl ciing tirong duong nhau.

Ta chu y d~n diem nrong tl,l' sau day. L\l'C hie dung len v~t the dU'Q'C xac dinh b6i tri so, dirong tac dung va hirong tac dung CGa no ; diem d~t 1\l'Cco the thay d6i tuy Y tren dirong tac dung, NgAu hrc tac dung len v~t the dU'Q'Cxac dinh boi tri so' cua momen, m~t phang tac dung va chieu quay cua no. Vi tri ngAu hrc co b the thay d6i tuy Y tren m~t phang
p

tac dung.
V~n de v~t the se chuyen d9ng

p'

Q'

nhir tM nao diroi tac dung cua ngAu luc la thu9c ve linh Vl,l'C nghien ciru cua d9ng luc hoc. Dua vao cac dinh ly d9ng 1\l'C hoc,' ta

co the chirng minh rang blit ky ngjiu hrc nao tac dung len v~t ran Hinh 47. tl,l' do se lam v~t do quay xung quanh trong tam (xem § 136). Con n~u v~t co' true quay co dinh thi ngAu 1\l'CnAm 6- b~t ky vi tri nao tren m~t phang vuong goc veri true se hun v~t quay xung quanh true do .voi cung momen quay. Dieu nay co the suy ra tir ddng thirc (24).
BM tOp 15. Cho don bAy cong ABCD (hinh 47) can bang dlTo; tac dung caa hai hrc song song P va P' tao thanh mQt ngllu hrc. Hay xac dinh ap I\I'C tren cac gai nra, neu AB = = a= IScm.

BC= b= JOcm,

CD= c= 20cm,

P= P'= JOkG.

Bai giAI. Thay ngllu lvc (P, P') bang ngAu h,rc nrong dirong (Q, Q') voi cac h,rc hirong theo chieu cac phan 11,1'1; cac gai. Trong tnreng hop nay, mornen cua cac ngllu l\I'c se clla bang nhau, tfrc la P (c - a) = Q h.

Nhir v~y, ap h,rc len cac gai c6 tri sa bang Q


=

Q'

=~

S kG

va c6 chi~u nhir th~ hien tren hlnh ve.

20

HQ"P cAc NGAU CUNG NAM TR~N

M(n

M~T PHANG

59

§ 20. Hep cac nglu cung nAm tren mi}t m~t phAng~ Di~u ki~n din bAng cac nglu. Ta chirng minh dinh Iy sau day ve hop cac ngAu : h¢ ngdu tren cung m(J1 m(il phdng nrtrng dtrong v&i m(Jt ngdu ciing trin m{il phdng do vtH momen bang 16ng dai so cdc momen cua cac ngifu thanh ph/in.
De clnrng minh. ta gia thO. co ba ngau voi cac momen mi' 1n2' m3 tac dung len vi,it (hinh 48). Theo dinh Iy ve S1! nrong dirong ella cac ngAu, ta co the thay cac ngAu do bang ba ngiiu (P" P'I), (P2, P;), (P3, P3) co cung tay don d va co cac momen : PI d= ml, -P2 d=
m2,

P3 d=

1n3'

Bay gio' cong rieng cac hrc d~t tai cac diem A va B voi nhau, ta diroc hrc R tai diem B, 11,l'C tai diem A chung ~o trj s'o R' bang nhau R
=

R'

PI -

P2+

P3•

Hinh 48. mQt ngau (R, R') voi momen

M = Rd = P.d + (- P2d) + P3d = m. + m2 + m3' Ta dii chirng minh dinh Iy doi voi tnrorig h9'P ba ngau. Hien nhien, ta cling co ket qua nhir v~y doi voi Mt ky so ngAu nao. H~ g5m n ngau 11!c voi ~ac momen m l' m2, ••• , mn co the dU'9'C thay bang mQt ngAu vo-i momen Tir dinh Iy vira chirng minh, ta suy ra : muon can bang M ngau phang thi dieu kien can va du la t6ng dai so cac momen cua cac ngau do phai

Ket qua la ca

ngjiu duoc thay bhg

,=

mk'

(25)

bang khong :
(26)
881 t,p 16. M<)tngAuI\!,cvoi momen ml tac dung len banh rang I ban kinh rl (hinh 49a). Hay xac dinn momen m2 cua ngAu citn phai tac dung vao banh rang 2 ban kinh r2 d~ gifr din bAng. Bai gi'i. Tnroc Mt My xet di~u kien din bAng cua banh rang l. NgAu IVCco momen ' ml tac dung V30 banh rang nay chi co the dtrQ'Ccan bAng b6i tac dung cua m<)t ngAu I\!,c khac rna trong mrong hQ'P nay la ngAu I\!,c (Q" RI). <7 day, QI la phan IVCvuong goo vo-i ban kinh la ap IVCdo banh rang 2 de len cac rang, con RI la phan IVCvuong goc V01 ban klnh la phan I\!'c true A (lip hrc de len rlingva phan Ivc cua true A con c6 cac phan I\!,c dQCthea ban kinh. Nhfrng phan luc nay can bAng nhau va khong tham gia vao di~u kien din bang). Khi do, theo di~u ki~n can bAng (26), ta c6 ml + (- QI rl)= 0, hay QI= nll/ri. Bay gier xet di~u ki~n can bAng cua banh rang 2. Theo tien d~ 4, banh rang nay se chiu tac dung cua h,c Q2= -QI tlr phia banh rang I (hinh 49b). Lee nay cling v6i philn h,c vuong g6c vo-i ban kinh cua phan luc true B tao thanh ngAu I\!,c (Q2' R2) cO momen bAng .. Q2 r2' NgAu h,c nay ding ~ din bAng v6i ngAu IVCco mornen m2 tac dung V30 banh rang

60

Hl;. LI,fC PHAN BO BAT KY TRtN

M';'T PHANG

Ch.

IV

b}

Hinh 49.
2. Do do, theo dieu kien din bAng (26) m2 dtrQ'c:

+ ('2
I

Q2 rz)= O. Tir day, vi Q2= Q"

ta tim

m2=o-ml'
r

OJ nhien Iii cac ng~u I\I'Cco cac mornen m, vii m2 khong thoa man dieu ki~n can bang (26) vi chung d~t vao cac v~t khac nhau.

Lire Q I (hay Q2) vira giai IT tren goi la irng 1\fCv6ng tac dung len banh rang. Nhir ta thay, irng hrc vong bAng mornen cua ngll.u 1\fCquay chia cho ban kinh banh rang Ql= ~
r
I

m2.
'2

CHUO'NG

IV

Ht

LVC PHAN BO BAT KY TREN M~T PHANG

§ 21. Bjnh Iy ve chuy~n dich song song eua life. HQ'P 11fC cua h~ hrc dong guy diroc xac dinh tnrc tiep bang tien de hinh binh hanh hrc, Doi F voi hai hrc song song, bai toan diroc giai bang each quy chung ve h~ hrc dong quy (xem hinh
39).

a)

b)

Ro rang Iii. ciing co the giai de dang bai toan nrong tl! doi voi b~t ky h~ hrc nao, n~u tim
diroc plnrong phap thu chung

Hinh SO.

Iy sau day cho ta phirong phap do : co tM di chuyen song song m(>t l!J'c uu: dung len v(it thA uri bat kj di~m nao cUa vdt tM ~ma khong lam thay d6i uic dung eua no nl?u ta them vao db mo: ngJu IWe co momen Mng momen cUa lu-c hi di chuyen doi vo-j didm rna h,!c dl chuyen ttri.
ve h~ hrc d~t tai mot diem. Dinh

§ 22

THU GQN

H~ U)'C

PHANG

VE MOT TAM

61

Gia th& hrc F tac dung cua lire van kh6ng d6i neu d~t hai hrc can bang F' va tao thanh mot h~ gom lire

co

len v~t nhir tai F", sao F' bang

th~ t~i di~m A (hinh SO a). Tac dung diem B bat ky nao do cua v~t th~ ta cho F'= F, F"= ~ F. H~ ba lire nay F d~t tai di~m B va ngjiu hrc (F, F") (27)

momen m= m 8 (F).

Ding thirc (27) suy tir cong thtrc (23). Nhir v~y ta dii chtrng minh dinh IY. Ta con co the mo ta ket luan cua dinh Iy nhu th~ hien tren hinh SOb (lY day, lire F xem nhir bi loai b6). Ta xet mot so vi du ap dung dinh Iy nay.
Thi d\l I, De gift din bang thanh d6ng chat AB co chieu dai 2 a va trong hro-ng P, din phai tac dung vao trung diem C mQI h.rc Q huang len phia tren co 111so bang P (hlnh 51 a). Theo dinh ly vira chung minh, co the thay II!C Q bang lire Q' dij.1 tai dllu A cua thanh va ng~u hrc co momen m= Q. a. Neu giam tay don cua ng~u lire nay to-i dai hrong h (hinh 51 b) thi phai lang cac hrc Y, F" sao cho Fh= Qa. Do do de gift thanh & dllu A thi ngoai hrc Q' ta phai lac dung them ngjiu lire (F, F'). Ta siS hinh dung thay ngay kel luan nay, ne~ thu- gift & gifra thanh hoij.c & diIu thanh (hinh 51 a, b).

f~~l--~t
f

..,f

1/

OJ

b)

Hinh 51.

Hioh 52.

Till d\l 2. Ngtroi ta qulin vao true tang trong I, ban kinh r hai sqi day thea huo-ng ngiroc nhau, & hai dllu day dij.t cac hrc F va F' ~~ - F (hinh 52) ; con tren true tang 2 co cling ban kinh chi qulin mot day va dij.t mQt lire bang 2 F. Hay xem tac dung cua cac Il!c d6 co khac nhau khong ? Ta thliy true tang trong I chi chju tac dung cua ng~u lire (F, F') co mornen quay bang 2 Fr. Con II!C tac dung len true tang trong 2 co the thay bang lire 2 F' = 2 F dij.t vito true va ngliu Il!c (2 F, 2 F'). v~y, true 2 chiu tac dung cua : I) ngAu hrc co momen quay 2 F r nhir & true I ; 2) II!C 2 F" tao ra ap h,rc tren true tang trong.

Boo

Do do, hai true se quay nhtr nhau, nhung th{" nhat khong co ap hrc nao tac dung,

true thir hai chiu ap hrc bang 2 F, con true

§ 22. Thu gQn M life phing ve m9t tam. Gia th& M luc F 1> F 2 •.. , F n nao do narn tren mot ~t phang tac dung len v~t the. Ta lay bat ky diem Onao tren m~t phang lam tam thu roi ap dung dinh Iy da chirng minh

62

Hf

LI[e

PHAN 80 BAT KY TRtN

r:»
~

M~T PHANG

Ch. IV

Hinh 53,

& § 21, do; tlit ca cac hrc ve Him 0 (hinh 53 a). Nhir vay, v~t se chill tac dung cua M lvc F\=c= Fl, F;,=F2,
.'"

F'n= Fn

(28)

d~t tai Him 0 va h~ ngjiu hrc voi cac momen xac dinh theo cong thjrc (27) bang: ml= mo (FI), m2'= m ; (F2), .." mn= mo (Fn). (28') Co the thay cac h!c d~t tai him O' bang met Il!c R cling d~t tai tam do, sao cho R = Fie hay theo dang thirc (28), bang

R= Cling vay, theo djnh Iy hop ngau, tat

LF

ca

(29) cac ngilll hrc co the thay bang

mQt ngAll hrc nAm tren cung m~t phang voi momen bang Mo hay theo dAng tlnrc (28') :

==

Lm

u; L mo
=

(F0.

(30)

Dai hro-ng R bang t6ng hinh hoc ella tlH ca cac hrc thuoc h~, nlnr ta dii bist, goi la vecur chinh cua hf ; dai hrong M 0 bang t6ng momen cua tat ca cac lire doi voi tam 0 diroc goi la momen chlnh cUa hf doi vifi tam O. Bang each do, ta dii. chirng minh diroc dinh Iy sau : m(J1 hf lu-c phdng tac dung len v{it the khi thu g(Jn ve' bat ky tam 0 mio deu dU'9'C thay Mng lu-c R bang vecto: chinh cUa hf va d(it tai tam thu 0 va ngJu l~c co .momen M 0 bting momen chinh cua hf dol v/ri tam 0 do (hinh 53 c). Ta chu y rang lire R khong phai la hop hrc cua h~ hrc vi no khong the m9t minh rna phai cung voi ngiiu hrc moi thay the duoc cho h~ hrc, Til' dinh Iy vira chirng minh, ta thay rAng hai h~ lire co cling vecto chinh va momen chinh la hai h~ nrong diro-ng nhau. B6i vay, de xac dinh

~ 23

cAe

TRtrd'NG

IH,TP THU H~ U)'C

PHANG

vE

D~NG

TOI GIAN 0

63 cua no

h¢ lu-c phang chi din xac dinh vecto chinh R doi v&i tam 0 ndo do.

va

momen chinh M

Ta co the xac dinh vecto R hoac bang phirong phap du-ng da giac 1l!C (hinh ·53 b) hoac bang phirong phap giai tich thea cac cong thirc (10) (xem § 10). R6 rang la tri so R khong pnu thUQC vao vi tri Him O. f)~i hrong M 0 diroc xac dinh tneo cong thtrc (30), nen khi thay d6i vi tri Him 0 thi noi chung momen M 0 ciing thay d6i (vi cac momen cua cac lire thanh phan thay dbi). BOi v~y, khi ra momen chinh can phai noi ro Him cua no.

Ji

=:»

Hinh 54.
30 kG,

Bai t~p 17. Hay thu h~ lire P, F" "',, F 3 tren hinh 54 vI: tam O. Cho biet pc

F,=' F2", F3, F 20kG;o<= O,Jm; b =O,5mi at= 60". Oai giAi. Bai toan dira vi! tim vecto chinh R cua h~ lire va momen chinh M; cua no dlli voi tam O. Ta co the X3C djnh R thea cac hlnh chieu R" Ry• Muon v~y, ta du-ng cac true
Oxy nhtr tren hinh ve, roi tinh hinh chieu cua til-ng l\I'ctren cac true do va momen cua chung dlli vo-i tam 0 (xern bang). r---

Fk
Fkx

P
0

F)

F2 - Fcosa. Fsin« 2bF sine

F3 -F
0
of

fky
ma
c__.

-p --- bP

--

Fr;osa.

-- Fsin«
of

(F kl

cose

Sau do, dien so li~u bai roan vao, ta dlrgc

R,

ee

LF~,

~- -. 40 kG, Ry

sza

LFky

~~_. 30 kG, Ma "" Lmo (F kl

= 11,3

kGm.

Nhir v~y, khi diri ve tam 0, h~ Il,I'Cdll'Q'C thay bang Il,I'CR d~t tai 0 v(ri cac hinh chieu R, - 40 kG, Ry - 30 kG (R =,. 50 kG) va ngllu hrc co mamenMo = 11,3 kGm.

§ 23. Cac tnrong hQ'p thu h~ IIFC phAng v~ d~ng toi gil~n. Dinh Iy chtrng minh trong § 22 cho phep ta xac dinh co th~ thu h~ hrc phang ved~ng toi gian nao. Ket qua se phu thuQc vao tri so cua vecto chinh R va momen chinh M 0 cua h~ do.
1) Neu R = 0 va Mo = 0 thi h~ lire can bang. bang se duoc khao sat ky & muc sau.

Tnro-ng hop din

64

H~ Life

"HAN

2) Neu R cc' 0, Mo =_~ Lmo (F 0. vi tri Him D, vi neu khac nhau, khong

3) Neu R

hai truong hop :

*' 0 thi

M0 Trong khong tiro'ng

*' 0 thi

BO BAT KY TRIoN MAT "HANG

Ch. I V

h~ luc thu ve mot ngau hrc co momen triro-ng hop nay, Mo se khong phu thuoc vao thi M lire do duoc thay the bang cac ngjiu Ivc duong nhau. Do la dieu khong th~ xAy ra.

h, lire thu ve m9t hop hrc,

a day

co th~ xAy ra

a) R *- 0, Mo = O.Trong tnrong hop nay hrc, tire la hop lire R di qua tam O.

diroc thay bang m9t hop nay co th~ bieu R" sao cho R' == R (hinh 55b). B6i vay, la tay don cua ngt\u (31)

b) R *- 0, Mo -1= 0 (hinh 55(/). Trong tnrong di~n ng1\u hrc co momen Mo bang hai lire R' va , va R" = -R neu d= DC hrc thi

Rd =, 1Mo I.

R' = R di qua di~m C. Vi tri cua diem C duoc xac dinh thea hai dieu Hinh 55. kien sau : I) khoang each DC = ~~d ( pC -1_ R) phai thoa man dang tlnrc (31) ; 2) dau cua momen hrc R' d~t tai diem C doi voi tam D, tire la dau cua M a (R ') phai trung voi dau ella Mo. Vi du ve each tinh toan nay cho trong bai ti[tp 20. Nhirng tnro-ng hop khao sat tren cho thay ra.ng h~ hrc phang neu khong can banz thi co th~ thu ve hoac m9t hop hrc (khi R OF 0) hoac m9t ngau luc (khi R ,= 0).
Bai t#p 18. Hay thu h~ nrc Ph P2, P3 tac dung len dam AB (hinh 56) ve dang toi gian va tim ap lire tac dung len cac goi A va B, neu PI = P2 ,~ P3 =c P. Bai gia\i. Da giac lire tao thanh boi cac hrc Ph P2, P3 la da giac khep kin, nen R
T6ng rnornen cua tat

aJ

b)

Cac hrc R va R': can ba.ng nhau nen co th~ IO<;Li Ta thay boo h~ hrc diroc thay bang m9t hop lire

ca

cA

cac hrc dlli voi moi diem (thi du diem C) bAng -

= O. Pa. Boi v~y, b~

hrc nay thu v~ mot ngau hrc c6 momen m = - Pa. Dung ngiiu hrc nay diroc thb hien bAng dirong cham cham nhir tren hlnh ve. ta thay rAng cac hrc Ph P2, 1'3 tac dung 160 cac goi .. . . cac ap IIJC Q I va Q 2 co tn ."Su b' ang Pa "t:

Bai t~p 19. Hay thu M lire Fh F2, F3 tac dung len gian AB (hinh 57) vi! dang toi gian va tim ap hrc tac dung len cac goi A va B, n~u FI = F2 = F3 = F.
Bai gia\i. Chu Y rAng cac luc F'2 va F 3 tao thanh met ngiiu hFC, n~u chuyen ngiiu hrc do ve vi tri nhtr diroc the hi~n bang dtrl'mg chlim treo hinh ve, thi cac lVC F b F'3 can bAng nhau va ca he hFC thu vi! met hop hFC R = F~.

<..._____

~ 24

65

IF;
I

Hinh 56.

Hinh 57.

Til do ta kN lu~n rdng tic dung ella die lire F 1. F 2, F 3 quy v~ m(it ap luc th~ng dung len goi A ; con goi B khong chju ap luc 01\0.
Q.
0=

Bai t~p 20. Hay tim hop h,re ella cac hrc tac dung len dlim AB (hinh 58) nllu P = 3 T, Q2 Q = 4T, khoang each OB= a O,8m.
=0 Co

Bai giAi. Dung cac lire P, Q. , Q2 dung da giac h7C, ta thay h,re R (vecto chinh ella h~ hrc ) co tri so bAng R = 5 T, vi trong da giac h,re be. ,= 2Q cos 60° 4 T; ab = 3 T. Tiep d6 l1iy diem 0 hl. giao diem ella cac hrc P va Q2 lam tarn mom en, ta tinh dlrQ'C mornen
0=

ehinh

ella h~ Mng Mo = mo (Q.) = aQ cos 30° ~= - 1,6 Y3 Tm. Khi do, theo cong thire (31), ta co :
= ~-

R TIr 0 k6 dirong thing vuong goc veri hlro-ng ell'a R, tren direng nay lay doan d. BAng each do ta xac,dinh dlrQ'C duong tac dung ella hop lire, Vi MG < 0, nen hop lire nAm ~ ben phai di~m 0 (rnomen h,re R doi vo-i diem 0 co dau am).

dcc

J~I=

0,32

V3 "'" 0,55m.

Hinh 58.

§ 24. Dj~u kj~D can bAng dta h~ I....c phAng tuy y. TrU'cYng bQ1l cae
song. ~ can bang b~t ky h~ hrc phang Olio thi din va chi din thuc hien dong thoi cac dieu kien :
SOD2

I.... c

R = 0 , Mo = 0,

(32)

trong do 0 la diem bat ky nao do tren m~t phang, vi khi R c~, 0 thi dai hro-ng momen Mo khong phu thuoc vao vi tri Him 0 (xem § 23, diem 2.) Cac h~ thirc (32) la dieu kien din vi neu mot di~u kien nao do khong

diroc thirc hien thi h~ lire tac dung len v~t s15nro-ng dirong voi hoac mot h9'P hrc (nett R "4= 0) hoac mot ngau 1l!C (khi Mo =F 0), va b&i v~y
kh6ng din bang. Dong thoi cac dAng thirc (32) con la dieu ki~n dl\, vi neu R '-~ 0 thi h? chi co the nrong dirong voi ngau luc co m6men M 0, rna M 0 = 0, nen no can bang,
5-731

66

H~' Life

PHAN

BO BAT

KY TREN

Mi~T PHANG

cs.

I\,

Ta hay tim cac dang giai tich cua cac dieu kien din bang suy tir cac dang thirc (32). Chung co th~ co ba dang khac nhau sau day: I) D~mg Co' ban coa cae dj~u ki~n din bAng. Trj so cua R va M 0 diroc xac dinh thea cac dang thirc :

R V R; + u; m., (F 0, trong do Rx ,= LFxk, Ry = yk. Nhimg R co th~ bang khong chi khi nao R; == 0 va Ry = O. BM vay, cac dieu kien (32) se diroc thuc ,
-iz:

iii ' LF

hien, neu :

LF

kx=

'LFky

0,

Llno (F 0

O.

(33)

Cac da.ng thirc (33) th~ hien cac dieu kien can bAng dU'iYi dang giai tich sa u day : dieu kifn can va dl~ d; hf lire phang tuy .v can bitng lit tdng hinh chieu clia tat cd cac lice len mot trong hai true toa d{J va t6ng cdc momen da chung dOi vtri btu ky tam nao nam tren miu phdng tac dung da cdc lire, bang khong, Cac phu-ong trinh (33) dong thai th~ hien str can bang cua v~t ran tl,l' do diroi tac dung C(Y hoc, hai dieu ki~n dau hi nhirng dieu kien dQng d9C theo cac true toa de>, con dieu kien khong quay tren m~t phang Oxy. cac cua din thir dieu kien can doi vo i he lire pha.ng. Ve mat de v~t khong chuyen ba la dieu kien db v~t

2*) Dang thn- hai coa cac dieu ki~n can bAng: diiu kifn cdn va du hf I1I'C ph/mg HlY }' can btlng lti tAng momen Cl}a tat cae lire d;Ji wh hoi lam A wi B ndc do vel t(5ng hinh chieu da chung len true Ox khong vuong goc v&i du ong thang AB, bang khong :

ca

de

Lm

(F

0=

0,

Lm

(F

==

0,

LF kx

O.

(34)

Dieu kien can cua chung Ili hi~n nhien, VI n~u bat k5r dieu kien nao khong duoc thuc hien, thi hoac R =1= 0 hoac M" =1= 0 (M II =1= 0), nen khong co Sl,l' can bang, Ta chlrng rninh dieu kien duo Gift tht'r doi vo-i h~ lire chi co hai dieu kien dau cua (34) diroc thuc hien, thi ta co M ,,= 0 vii M R = O. NhU' da chirng minh trong § 23, h~ hrc nay co th~ khong can bang, rna co hop hrc R di qua dong thai ca diem A va B 1) (hlnh 59). Nhimg theo dieu ki~n thir ba thi R; = = O. Vi true Ox khong vuong g6c voi AB, nen dieu ki~n cuoi cling nay chi co th~ thirc hi~n diroc khi hop hrc R = 0, Hinh 59. nrc iii khi co sir can bang.

LFb

I)

Di!u nay suy tir tnro-ng hop 3, diem a & § 23.

D1EU KI(:N

CAN BANG CllA

HF.l L~rC PHANG TOy Y

67

diflu kien dill I'd

3*) Dang thu' ba clla ciic di~1lI ki~n can bang (phu ong trinh ba moment : du dd hf lu c phang Illy ,1:' din bang Id t(ing momencua tal ((I cdc lu:c ay doi vO'i ba tam bat ky A, B vc/ C ruio do khong nam tren cung m9t dirong thang, bang khong :

Lm~

(F0

0,

Lm

(Fk)

==

0,

Lmc

(F0

0,

(35)

Ciing nhtr (y tnrong hop tnro-c, dleu kierr din la hien nhien, Dieu kien diJ cua (35) the hien o' chb, neu cac dieu kien do diroc thirc hien dong tho; rna M lire van chira din bang, thi no phai nrong du ong voi mot hop lire di qua dong thoi ca ba diem A, B va C. Nhir v~y khong th~ xAy ra diroc vi cac di~m nay kh6ng nam tren cung mot dirong thang. Boi vay, neu co cac dieu kien (35) thi phai co din bang. Ta thay trong tilt ca cac tnro ng hop h~ lire phang duoc khao sat co ba dieu ki?n can bang, Dieu kien (33) diroc xem la dieu kien CO' ban, .vi khi sir dung n6 khong co gioi han nao doi voi SI! hra chon cac true toa dQ va tam momen.
Neu ngoai hi) 1l!Cphang F" F 2,
.,.,

F n' v~t con chiu tac dung cua h~ ngliu !l,I'C co cac

mornen m" ms, . ." m, ciing nam tren mat phang d6, thi cac ng~u II!Ckhcng tham gia vao

phiro-ng trinh hinh chisu khi I~p cac di~u ki~n din bAng, vi tbng hinh chien cua cac Il,I'C cua ngliu tren bat ky true nao d1!u bAng khong. Nhirng cling trong cac phuong trinh mornen ay, ngoai rnomen h!c ta phai cong them (cong dai so) cac momen ngliu hrc, vi tOng mornen cua cac lire cua ngliu doi veri b1it ky tiim nao bao gio-ciing bang momen ngliu 1l!C(cong thirc (24), § 18). Do do, dieu kien din Mng (33) cua he hrc va h~ ng~u 1l!Ctac dung len ,,~t co dang : LFb ,= 0, LFky
= O.

Lillo

(Fk)

+ Lm;

= O. tir.

(36)

Trong tnro-ng hop nay cac dieu ki~n (34) va (35')dI!u co S\I' bi!!n (fOi nrong

Can bing cua h~ lu-c phAng song song. Khi tat ca cac hrc tac dung len v~t the song song nhau, thi ta the lnro-ng true Ox thang goc vo-i cac luc, con true Oy song song voi chung (hinh 60). Khi do hinh chieu cua m5i hrc tren true Ox se bang khong va dilng tlurc thir nhlH trong cac dang thirc (33) bien thanh dong nhat tlnrc 0= O. Do do, doi voi cac lire song song chi con hai dieu kien can

co

bang :

2:F

ky

==

0,

Lmo (F'~)

~~ 0,

(37)

tro_ng do true Oy song song voi cac lire. Dang thir hai cua dieu ki~!1 din bhg doi voi h~ hrc song song duoc suy nr cac d~ng tlnrc (34), nhir v~y ta co :

Lm"

(F

,= 0,

Lm"

(I<'

=c O.

(38) Blnh 60.

Do do, cac diem A va B khong rhrcc nAm tren duo-rig thang song song voi cac lire.

68
chu

HE LV'C

PHAN

80

BAT KY TREN

Mtl,.T PHANG

Ch. IV

§ 25. Cach gi:~1cac bai toano Khi giai cac bai toan cua muc nay can

den tat ca nhirng

dieu chl dAn chung & § 13.

MQt Ian nira can nhan manh la khi giai, tnroc het, phai xac dinh v~t nao can khao sat din bang, Sau do tach v~t do ra, xem no nhir v~ t nr do, r6i bieu dien tat ca' cac lire da: cho va phan lire lien ket tac dung len v~t do. Cuoi cung la l~p cac dieu kien din bang theo mot trong ba dang dieu kien noi tren, sao cho thu diroc h~ phiro-ng trinh don gian nhat (h~ phirong trinh don gian nhat la h~ trong do ml)i phiro-ng trinh co met An so). De co cac roi qua trinh du-ng true toa phtro ng trinh hiet di qua. phtrorig trinh don gian hon (neu dieu nay khong lam rk tinh toan) thi : a) khi l(ip cdc phtrong trinh hinh chieu ta d{i vuong goc v&i mQt lu-c chu-a biet nao do ; b) khi l(ip cac momen, ta lay tam momen tai diem md co nhieu lu-c chu-a

Doi ilk khi tinh mornen nen phan 11,l'cda:cho thanh hai phan lire, r6i ap dung dinh ly Varinhong tinh mornen lire bang i6ng mornen cua cac phan lire do. Trong nhieu bai toan tinh h9C tlnro-ng thiro-ng phai xac dinh phan hrc cua cac goi nra dung de gin cac dam, gian v. v ... Trong ky thuat thirong g~p ba loai goi tira sau day (ngoai cac goi nra da: khao sat & § 4) : 1) Goi t\l'a khlrp di dQng (hinh 61, goi ura A). Phan hrc N A cua goi nra nay hiro-ng theo phap tuyen cua mat phang d& cac con Ian cua goi nra di dong. ' !I 2) Gl)i t\l'a khlrp dinh (hinh 61, goi tira B). Phan ltrc R II cua goi nra nay di qua true khop va co the co bat ky huong nao tren m~t phang hinh ve. Khi giai bai toan ta se bieu dien phan 11,l'C B thanh cac phan R lire X B va Y B cua no theo hirorig cac true toa d9. Khi giai xong neu ta tim diroc X B va Y II thi phan luc R B cling se diroc xac diIn h , v", tri. so :(; "R B = 1/ X2 -t - y2B' rB

co

Hinh 61.

Cach lien ket nhir. tren hinh 61 la de trong dam AB khong xu at hien irng suat them khi no thay d6i chieu dai do nhiet ,d9 hoac d9 yang gay nen. -Ta thay rl1ng, neu goi tira A cling dtroc lam co dinh (hinh 61), thi diroi tac dung cua bat ky h~ 11,l'C phang nao, dam se sieu tinh boi vi trong tnro-ng hop nay bon phan hrc can xac dinh X" Y A' X"' Y" (xem § 12) nam trong ba plnrorig trinh can bang (33).

§ 25

eACH GlAI cAc BAI TOAN

69

3) Gl'ii bra ngam cl'i djnh hay ngam co-ng 'A (hinh 62). Trong tnrong h9'P nay h~ phan lire phan bo tac dung til" phi a cac mat phang goi tira len dau ngam cua dam.Coicac lire nay dll thu ve tam A, ta co the thay chung bang mQt luc an R A d~t tai Him do va ngliu luc voi momen M A chua biet. Luc R A co the dtroc bieu di~n thanh hai phan hrc Hinh 62. X A va Y A' Nlnr v~y dt tim phan luc cua goi nra ngam co dinh, ta can xac dinh ba dai hrong chua bist 1ft X A' Y A va M A' Neu tai mot diem B nao do ta cho them mQt goi ura nfra thi dam se tr& thanh sieu tinh.
Ve hiro-ng cua cac phan lire lien ket dum khao sat trong § 4. cac dang khac dll diroc

Bai ttp 21. Hay xac dinh cac l\!"Ccua cac banh xe A va B cua c~n true dlfQ'C rno til bAng sa do tren hinh 63 de len ray. Trong hrong citn true P ,,0 4 T, trong tam nam tren diro-ng DE. Trong hrongvgt nang Q = 1 T, utm vai cua c~n true b 0= 3,5m, khoang each AB = = 2a ,= 2,5m. Bai gi!\i. Ta hay xet sv can bang' cua din true. Neu xern can true nhu v~t tv do, thi no se chiu tac dung ella cac luc P va Q da cho va cac phan hrc N, va N.. Nhimg dii!u ki~n can bAng (37) cua h~ 1\1"Csong song nay, neu Jay di~m A lam tam momen va chieu cac 1\1"Clen true thang dirng, se co dlJng :

- Pa

N.

Ns . 2a·N. -

P-

Q (a + b) = 0, Q = O.
nay, ta dlfQ"C :

Gilli cac phuong

trinh

N.

P 2 P 2

Q
2

( (

b a b _+ a

) )
+

= 1,1

T,

N. =_+
~

Q 2

= 3,9T.

Hinh 63.
doi vm di~m B:
=

ki~m tra ket 'qull, ta I~p phuong

trinh momen

- N.2a

Pa ~ Q (b - a)

O.

Di~n tri so cua N. vira tim dlfQ'C vito day, til. diroc dong nhlit thirc. Ap 1\1"C cua banh xe de len ray co tri so bAng N, va N., nhirng co chii!u hirong xuong dlfm. Tir each giai tren cho thay, khi:
= _a_ P = 2,22 T

b-a

thi phan 1\1"C N, bang khong, tu-c lil banh xe ben trai khong con de len ray nii:a. Neu tang trong hrcng Q len nira, thi c§n true sf hi l~t. Trong hrong Q Ian nhat rna can tT\1CvAn gifr

70
d troc can bang, theo dieu ki¢n

Hi; de-oc

t.trc

PHAN BO BAT KY TREN M~T PUANG

eh. IV

Lm. (F k)

xac djnh = O.

Bili t¥p 22. M91 dam dong chat AB, trong hrong P, ell dau A ti lell mij.1 phang ngang nhiin va gO- D. dilu B lila len mat ph~ng nghieng 1(10 voi dlro-ng nam ngang m91 goc <I (hinh 64). Cho biel dam nghieng so vo'i dtro-ng n!lm ngang m9t g6c ~. Hay xac djnh ap 11I"cella dam de len hai mij.1 phang va gil D.

Hinh 64.

Bai gia\i. Xet S\I" can bAng ella dllm AD neu b6 cac lien ket va coi no la v~t t\l' do.Khi do d1im se chiu tac dung ella cile 11I"e: Il}'c P dii eho d~t & gifra dllm va cac phan h,rc lien kct R, N" Nz hll{mg vuong g6c VO; cac m~t pha.ng ttrong frng. Dung cac true toa 119 (xem hinh 64) I~p di~u ki~n ciin bAng (33) v6i lam momen tai giao di~m A cua hai I\I'C An. Ta tinh so bl) cac hlnh chieu cua tung h,e tren true toa (II) va momen cua no doi vo; tam A roi dien cac dai hrong d6 vao bang I) . 6' day ta ky hi~u AD o~ 2a va.c. KA 8 = Y (AX la tay don cua luc R doi voi tam A).

ya

Fk

r.,
F ky

---.--0

NI

-----r---··--·---+--------I r--.--.--t--------I--------.--() -P
-- Pa cos B N~

N2

R .-- R sin
<I

~.---.-

MA (F 0

-------+------0 o

------N,

. -------R
2(1

Rccrs a

cos

Bay gio ta !~p di<!u kien can bang N2--Rsin<l O,N1--P-fReosrJ'. O,--Pacos~+-2Raeosy O. Tii phiro-ng trinh cuoi cung ta tinh diroc R ~~ ~ cos 2 cos Y

P.

y=

Vi dtrong thang <1- ~. Vij.y

AK.

song song voi

mij.t philng

nghieng,

nen L:.. KAx

<I;

do d6

P cos ~ 2 cos (<<- ~)

,) Ve each Ii}p bang, xem ghi chu 11101 & bai t~p 6.

, 25

CACH GIAI cAe Giai hai phirong N, trinh dau, ta diro-c :.


=P
[ COS

sAl ToAN

71

1-

a; cos ~ ] 2 cos (a;--~) ,

N2

= P ------

sin a; cos ~ ~) ngwoc chieu.

2 cos (a;irng, nhung

Ap hrc de len cac mat phang co

[rl so

bang cac phan hrc urong

De kiem tra ket qua tinh cac dai hro ng NJ V,) N" ta lap phiro-ng trinh momen doi veri cac giao diem cua cac du-o-rig tac dung cua cac luc R, N, va R, N l : Tir each giai tren ta rut ra kef ludn : Khi muon xac dinh cac hinh chieu hoac momen cua cac lire rna doi hoi phai hiet mut dai IUQ'ng mID do (chieu dai hay goc) trong hai toan, thi din ky hieu dai hro-ng ~ly bang chir nao do roi I~p phu-o-ng trinh hang, Neu dai hrong ay khong hi trong qua trinh giai hili toan, thi bieu dien no qua cac dai IUQ'ng dii melt can loai phai biet.

Bai t,p 23, Vom doi xirng (hinh 65) chiu tac dung cua h~ hrc diro'c thu gon thanh ",cQ co 4 T d(it tai diem D vii ngau hrc veri mornen M 0= 12 I'm, Trong hro ng cua vom p~. 8 T, cac Hinh 65, kich thiroc bang : A B a -,10m, b = 2m, h = 3m, a; 60°, Hay xac djnh cac phan lire ella goi nra khop ella goi tva di dQng A,

co dinh

B v;\

Rai gi&i. Xet sv din bang ella roan he> vorn neu bo cac lien ket va eoi no lil V~l ur phan lire N ," x., Y" cua cac goi tva (phan Iuc cua goi tva khop co dinh duoc bieu dicn thanh hai phan lire nhir tren hinh 65), Trong bai toan nay nen J(ip cac dicu kien Gin bang du oi dang (34) bang each lay mornen doi veri cac tam A vii B vii cac hinh chieu tren true Ax. Khi do trong mOi phirrmg trinh eo mQIIvc An tham gia, Tinh hlnh chieu cua tirng lire V,) momcn roi I~p hang, Trong tnro-ng hop nay khi tinh momen cua hrc Q ta phan no thanh hai phan hrc Q,.Q y va ap dung dinh ly Varinhong l(ip cac clieu ki~n can bang trong do IQ, I Qcos a;, IQy I Qsin a;, ta co: ...,._______ x. + Q cos a;o 0, (a) do, Khi do vom se chju tac dung cua cac h,rc P va Q d1i biet, cua ngllu IJ!C co mornen M [) va eric

Y"a -- P!!_ ,2 2

h Q cos a; -- b Q sin a;f-M

0,

(b)

- N.,a + P!!_ -- h Q cos a;

(a -

h) Q sin a;

+
Q

Mo

o
_, Mo 0

(c)

Fk

N" 0 0

P 0 -p!!_ 2 p!!_

X.
X"

Y.
0

F""
rnA (FkJ

Qcosa;

() -,-_.,-

YI~

IQx1h-

IQy Ib

MD
M"
-'

mil

(FkJ

NAa

0
I

0
1

- IQ. Iht l_QyI(CI--b)

72
Giai cac phtro'ng trinh tren, ta co :

Ch. IV

x.
Y.

- Q cos a;

2T,
m

-+
2
p

b sin

+
a

cos

~I!_""
a
at

4,09 T,

N'=2+Qa

(a-b)sinat-hcos

MD "'" 7,37 T.

Dai itro'ng X. eo dau am, nen thanh philn hrc X" co chieu ngu oc v{yi chieu chi tren hinh ve, dieu rna ta co the thay tnroc. Phan l irc loan phan cua goi Iva B bang tbng hinh hoc cac hrc X" va y. va co tri so bang:

R.

VX;-::-~-Y;-=

4,55 T

Neu ngAu 1\fC tac dung len vom co chieu quay ngU'Q'Cvoi chieu ve tren hinh 65, thi M[) ~• .- 12 Tm. Trong truong van khong thay dbi. hop nay ta co Y. " 6,49 T; N., trinh hinh chien tren true Ay : P.4,97 T, con dai luong X•

De

kiem Ira, la lij.p phuo'ng N, l Y. -

Q sin

a; ."

O.

(d)

Thay cac dai hrong NA va Y. da lim dU'Q'Cvao phirong trinh nay, ta thay chung thea man no (nen dien duoi dang long quat d~ kiem Ira cac cong thirc va duoi dang so de kiem tra cac con tinh).

cac hinh chieu va mornen cua cac 1\fC thhg

rang each kiern tra nay co the khong phat hien dU'Q'Csai sot do xac dinh sai goc veri true Ay. B6-i v~y, can ki~m tra them phan tinh toan nay hoac I~p them mot phiro ng trinh nira, thi d\l phiro-ng trinh momen doi vai tam D, kiem tra lai.

Can chu

de

Ciing can chu

II
/

them, nhir ta dii biet, khi lap cac dieu kien (34) phai Jay htro-ng cua true chieu khong vuong goc voi du o-ng AB, nrc hi khong hirrrng doc theo Ay nhtr trong bai toan khao silt. Tuy nhien, neu nhir ta I~p phirong trinh thir ba trong cac hinh chieu tren true Ay, thi ta ' se dU'Q'C he phiro-ng trinh (b), (c), (d) chi co hai an N, va Y" (vi trong h~ nay mQI phtro'ng trinh lit he qua cua hai phuo-ng trinh kia). Do do, ta khong the xac dinh dU'Q'C phan lire x nBal t~p 24. Dam dong chat AB duoc ngam V<'IO ruo-ng va tao voi thitnh nrc-rig mQt goc a; 60° (hinh 660). Phan darn 6- ngoai uro ng co chieu dai b =, O,8m vii co trong ltro'ng pc- __ JOOkG. Ben trong goc DAB di!.t mQI khoi tru trong 1U'Q'ng Qc. 180kG, tiep xuc voi darn tai diem E, trong do AE a O,3m. Hay xac dinh pnan lire cua ngam.

a)

Hinh 66.

1
eACH (dA, cAc BAI ToAN

73

Bai giai. Xet SI! din bang ella dam n!!u bo cac lien kel va coi no la V~I II! do.Nhir v~y iren dam co cac hrc lac dung : hrc P d<)1 6- giira dam, ap lire F cua khoi Ir\1 d<)1tai diem E vuong goc voi dam (nhtrng du the nao cling khong phai la khong co hrc Q tac dung vao khoi tru, rna kh6ng lac dung vao dam) va phan II!C cua ngarn diroc bieu dien bang cac phan ltrc X" Y, va ngi!tu 1\fC voi mornen M, (xem hinh 62). De l~p cac dieu kien can bang (33), ta tinh hinh chieu cua tat ca cac lire tren cac true toa dQ va mornen cua chung doi voi lam A (xem bang).

X., X, 0
k)

Y,
0 YA 0

F Fcos ex -- Fsin ex _- Fa
-_.

P
0

M,
0 0 sin ex

Fh F

ky

-p p!.
2

m, (F

!vi,

De xac dinh ap hrc F ta phan luc Q d<)t tai tam khoi tru thanh cac phan lire F va N vuong goc voi dam va voi nrong (hinh 66b). Tir hinh binh hanh lire ta tim dtroc : F

_g__.

sin ex

Bay gia l~p cac dieu kien can bang va dong thai thay vao F tri .so vira lim diroc, ta co :

x,

Qctgex

0,

Y, -p' b sin ex

Q __ P

o,

M, Giai cac phirong XA trinh

_.11_

sm e

O.

nay, ta co: _-_103,8kG.; Y, = P

= - Q ctg ex=
M, = Q

Q = 280kG.,

_.11_

sin ex

+ p!.
2

sin ex .~ 96,9kG.rn.

Phan 1\I'C cua ngarn gom hrc RA

o=YX!+l';

va ngau tuc voi m6men

MA•

Cuoi cung, ta nhan manh mQt Illn nira ve ket lu~n C<Y bin rut ra tir qua trinh giai bai loan: chi co cac hrc d(it twe tiep vao v~1 rna ta khao sat can bang la tham gia vao cac dieu kien can bang,
Bai t.P 25. Tren gia (hinh 67) co hai rong roc C va D, nguoi ta vat qua cac rong roc mQI S9'i day, dan tren mang trong luo-ng Q .~ 240kG, dau duoi buoc vao di~m B. Gia dtroc gi& can bang bOi day chang EE,. B6 qua trong Itr9'ng cua gia va ma sat 6- rang

74

HI; LI,fC PHAN Bi') BAT KY TREN M~T PHANG

en.

IV

roc, hay xac dinh sUe ding tt day chang vii phan hrc cua ngam A nllu xem no nhir mQI khop (IUe la ngarn khong cirng, cho phep cQI quay quanh di~m A). Khoang each lic cQI toi rong rQC C bAng 1m, cac kich IhlJ'(YC khac cho tren hinh ve. Doi gi!\i. D6, cac lien kill va xet S\l' can bAng cua loan bQ k~1 diu gom gia, cac rong rQC va doan day KnCM vat qua cac rong rQC (xem bai I~p 10; ket diu thuoc loai lien ket cirng nen co the xern uhir mQt ca the). Ket cllu chiu lac dung elm cac h,rc ngoai sau day: Q va F la SlrC cang & cac doan day tai cac di~m M va K va phan Ivc lien ket T, X., Y., Cac 1'!C trong can bAng nhau, nen ta khong ve. Vi khong co ina sat 6rong rQC nen sUe ding tren SU~I SITi day nhtr nhau va co trj sll F ,= Q. voi tam A.

Hinb 67.

Tinh hinh chieu ct'Ja tllt cA cac I'!C tren true toa de) va mornen cua chung'dlli bang each Illy cac gOc 1.1 va ~ (xem bang) :

Fk Fb. FIcj

Q
0 -Q -

F Fcost.l Fsint.l
1.1

T Tcos ~ Tsin ~

X.
X. 0 0

Y.
0

r,
0

m. (F0 - Q.I,O

- F. 0,9 sin

T. 1,2 sin ~

Til cac tam giac vuong AEEI va ADB ta co EEl

= 2,Om, DB

= I,Sm.

sin ~ = 0,8; cos 1.1 = cos ~ c.~ 0,6. Nen trong tnrong hop nay 1.1 L~p cac dil!u kien can bang, dong thm thay cac ham sa 1U'Q'ng giac bang cac trj sa vila tim dU'Q'Cva coi F = Q, ta co : O,6Q Giai cac phirong trinh T= I,OQ O,6T

= ~.

Do do sin

1.1 =

+
+

X.

0,

Q - 0,8 Q - O,~ T
O,72Q nay, cuai cung 430kG, X. = ta 'dU'Q'c :

+ 0,96T

Y. = 0,
= O.

24 Q =

43

40

19

114 kG , Y. ""

30 Q

97

776 kG.

Ta chu y d~n cac ket lu<,in sau day: I) Khi I~p cac dil!u kien can bAng moi ket cllu rna sau khi giAl phong cac lien ket vAn ctmg, thl co th~ xem no la mQt ca th~ ; 2) cac h,rc trong lac dung len cac b(> ph"n ket cllu (trong bai ~p vila khao sat la sire cang ct'Ja doan day DC

§ 26

CAN BANG CVA H~ V~T

75

iac dung len cac Tong TQC C, va D) khong tham gia vao cac diC!u ki~n din bA.ng, vi chung 1:\ cac lI,c can bAng nhau.

§ 26. c§n bAng ca\a h~ v,t. Trong nhieu tnrong hop, viec tinh toan tinh cac cong trlnh kien true tlnrong dU'Q'c qui v~ khao sat' cac dieu ki~n can bang clla ket diu gelm nhieu v~t diroc gift voi nhau M,ng cac lien ket nao do. Cac lien ket ket gift cac bQ phan cua ket diu do goi la lien k~f trong, khac voi cac lien kef ngoai lit lien ket gAn ket diu voi cac v~t khong tham gia vao ket diu do (thi du nhir voi cac goi dC'r).
Neu sau khi giai phong cac lien ket ngoai (cac goi ura), ket diu van lien ket cimg thi bai toan tinh hoc dU'Q'Cgiai nhtr doi voi mQt v~tth~. Nhirng vi du nhir v~y dll xet trong cac bai t~p 23 va 25 (xem hinh 65 va 67). Tuy nhien, co the g~p cac ket diu cong trinh sau khi giai phong cac lien ket ngoai vAn kh6ng the xem lit ket diu cirng diroc, Thi du ve loai ket diu nay lit vom ba khop (hinh 68). Neu bo cac goi ttra A va B thi vom se khong lien ket cirng : cac bQ phan cua no co the quay quanh khop C. Can cir vao nguyen ly hoa rAn thi khi can bang, h~ luc tac dung len ket diu do pMi thoa man cac dieu kien can bang cua v~t the. Nhirng cac dieu kien nay, nhir dff Hinh 68. noi, la dieu kien din, chir khong phai la dieu ki~n duo Vi the tir cac dieu kien do, khong the xac dinh diroc Hit cac daihrong eUrm biet, Muon giai bai toan, ta phai khao sat them S\1'can bang ella mQh.: ';~ mQt so bQ phan nao do cua ket diu. ,~ Chang han, kh i~p cac dieu kien can bang ella cac lire tac dung len vom ba khop (xerri 'binh 68), ta se dU'Q'Cba phtrong trinh voi bon An so XA, YA, XR, YR' Xet them cac dieu kien can Mng ella nu-a vom ben trai (hoac ben phai), ta se them ba phuong trinh nira ehira hai An men la Xc> Yc' rna ta khong ve tren hinh 68. Giai M sau phuong trinh nay ta se tim dU'Q'Cea situ An (xem bai t~p 26).

ca

Phuo-ng phap thir hai co th~ ap dung d~ giai cac bai roan nrong t\1' la ta tach ngay ket diu thanh cac v~t rieng re va l~p cac dieu kien can Mng ella tirng v~t mot, xem no nhu la v~t t\1' do (xem bai t~p 27). Khi do .cac phan II!<:lien ket trong se nrng doi mQt hang nhau-va ngU'Q'Cchieu. Bang each do, dc>i voi ket cau g6m n v~t,m3i v~t chiu tole dung cua mQt h~ 1\1',e

.'

76

HI;. tire PHAN

BO BAT

KY TRIl.N

M~T

PHANG

Ch. IV

phang, thi ta se dU'Q'c 3n phirong trinh du d~ xac dinh 3n an so (doi voi cac h~ II!C khac, so phuong trinh se khac hAn). N~u doi voi k~t diu rna so phan II!C lien ket Ion hon so phtrong trinh co cac phan lire lien ket do tham gia, thi ket diu la sieu tlnh,
Oai tip 26. M(>t gia g6m thanh ngang AD (hinh 69a) co trong hrong PI = 15 kG dU'Q'Clien kl!t voi urong bang khop va thanh chong CB co trong hrong P2 = 12kG ciing dU'Q'C lien kl!t vm thanh ngang AD va vm nrong bang kh<'rp (cac kich thuoc cho tren hinh ve). Tai dllu D cna thanh ngang treo trong IU'Q'ng Q = 30 kG. Hay X3c dinh phan hrc cua cac khop A va C, gi!\ thil!t rang cA hai thanh deu d6ng chill.

Hinh 69.

Oai giii, GiAi phong cac lien kl!t ngoai , va xet S\I' can bang CD" toan b(> gia noi chung. Cac l\i'C tac dung dii bil!t la PI' Pl, Q va cac phan I\I'C lien kl!t XA, Y .. Xc, Y c. Sau khi giai phong cac lien kl!t ngoai, gia khong co kl!t cilu lien kl!t cirng (cac thanh co the quay quanh khop B). nhtrng theo. nguyen Iy hoa rAn, khi can bang cac h,rc tac dung len gia phai th6a man cac dieu ki~n can bang tinh hQC. 4p cac dieu ki~n do, ta duoc : XA YA

+ +-

X,

= 0,
PI P2
-

Y, -

Xc,4a·-Y,a--

Pla-

Q = 0, PI.2a-Q.4a

o.

Ta thliy ba phuong trinh tren chira bon anla XA, YA' X" Yc' giAi hai toan nay, ta xet them cac dieu ki~n can bang cua thanh ngang AD (hinh 69b), Thar'.ay chiu tac dung cDa cac I\I'C P" Q va cac phan hrc X•• Y A' X" Y n - Lliy momen c~' !_ll~cli cac lee do dlli vm tam B ta se I~p dU'Q'C phirong trinh thir nr (trong phuong td,y'nay khOng co cac an mo-i X., Y. tham gia). Ta co : "i ~.-

one

Z:m.
I YA = - (PI3 214

(F0

'"'

YA• 3a

Pia -

Qa

= O.

Gilii he bOn phuong trinh vira I~p tren (bat dllu tir phuong trinh culli cung), ta tim dU'Q'C: Q) = -- 5kG, 2 4 Yc = - P1+-Pl+-Q = 62kG, 3 3

Xc

= --

323

P,+- -

P2

+-

Q = 56 kG,

= - Xc =-

56 kG.

Tir cac ket qua nay ta th!iy cac 'h,rc Y A va XA co chieu ngU'Q'C voi chieu chl trsn hinh ve. Neu can xac dinh cac phan I\I'C cua khop B, thi ta xet cac phirong trinh hinh chil!u cua cac 1\I'Ctac dung vao thanh AD tren cac true x va y. Ta tim dU'Q'C: X, -I- Q - YA = 50 kG,
=-

XA,

Y. = PI -I-

§ 26

77

Ta chu y rAng khi giai h~ phirong trinh, phai dien Irj so cua tirng dai II1Q'ng vao phtrong trinh ti~p sau viri dAu rna d""i hro ng do thu dl1Q'C khi giai phirong trinh tnroc, Thi du, trong tnrong hQ'P vim khao sat, ta phai thay vao dang thfrc elloi cung YA bAng- 5kG, chii' khang phai bang 5kG. Dieu do co nghia la du co tim thay YA = - 5kG, cling khong dl1q'C dOi chieu cua 1l!C tren hinh vi! va xern YA = 5kG nhir doi khi ngl1<'ri ta vAn lam, vi lam nhir th~ co the dAn d~n sai Him khi giai cac phirong trinh din bang ti~p sau. Nhir ta thay, khi giai cac bai toan tinh hQC khang ph!! luc nao cling clin I~p tlit co cac di~u ki~n din bang cua v~t kh30 sat. N2u bdi toan kh6ng yeu ciiu xdc dinh phOn IlI'c CUDcdc lien k2t nao db. (hi nen gang I~p ngay cae E phll'(mg Ir"nh khong chu-a cal' phOn IlI'c chsra II bi2t do. Nhir ta da lam trong bai toan vim rlSi khi xet S\f din bang cua thanh A D bing each chll"p mQt phirong trinh momen doi v6i tam B.

co

A va ty I~n gbi t\l'll C (hinh 70a). Dliu B cua dlim nai khop v6i thanh BE trong 1l1Q'Og P = 40 kG tY len gir D. Cho bi& CB =
0"

Bil t,p 27. Dlim ngang A B trong II1Q'ng Q = 20 kG dl1q'C lien k1!t v6i nrong bing khop

+.AB
Blii

va DE

BE, goc

(I

45°. Hay

xac dinh phan 1l!C /y cac gai t\f8, bi~t rAng dlim va thanh d~u dlSng chill.

giM.

Tach

thanh

hai bQ ph"n

vd

Hinh

70.

khao sat rieng S\f din blng cua thanh BE va dlim AB. N~u xem thanh liE nhir v~t t\f do (hinh 70a), thi no chiu tac dung CUa 1l!C P va esc phAn l\fc lien k~t N", X ., y •. Ky hi~u BE = a va I"p cac dieu ki~n can bAng (33) cho M 1l!C d6, ta duoc :

L
~

LF .. "" X.
(FiJ ...

F"" ... Y • - P

+ No
2

No sin (I = 0,
COS(l = 0, P'2COS(l

£."m.

N"3a-=

o.
2

Giai cac phuong trinh nay, ta tim dl1q'C : N"


= 4'Pcos« = P(I

= 21,2kG, X.

3 . gPsm

(I

15kG,

3' 2 ~ -cos «) = 25kG. 4 N~u xern dam AB nlnr v~t t\f do, no chiu tac dung cua 1l!C Q, cac phan II,I'C lien ket ngoai NCo X" Y A va cac ap 1l!C X:. va y~ cua thanh BE dl1Q'c truy~n qua khop B (hinh 70b).

Y.

Nhir v~y, theo tien d~ 4, cac l\fc X~ va Y~ phai nguoc chieu vai cac 1\fC X.

va Y., co tri sa

X;

•. Ky

X., Y;
hieu AB

Y..
= b va I~p cac di~u ki~n din

bAng (34) cho cac l\fc tac dung len dlim, ta

dl1Q'C:

Lm, (F Ii - Lm< (F.) -

LFb '" X,

-- 1", Y. b

0,

Y.: h

N,

. ~ ht

Q~

Q~ -

y;:

0,

r/>~ ~1 "fffJ
"

~J-.)OW

~~

78
Thay X'.
=

Hi; LI,)"C PHAN BO BAT KY TREN M~T PHANG X


A R

Ch. IV

va Y'

= Y. vao cac phuong

trinh nay r6i giAi, ta co :

X.

ISkG, N

I
A coo -

Q--

Y.

= --

7,5 k G,

=~

4 Q+ ~ Y 2

52 , 5 kG.

Tir cile ket qua tren, ta th1iy tlit cA cac phan lI,re, trir Y AO di!u c6 chii!u nhu tren hinh 70, chi ct'q;~in I\I'C 'I A co chii!u thuc te htrlYng xubng dU'lYi.

Khi giai cac bai toan thea phuong phap nay, din chu ) r~ng: nro tip ntlo dO len v~t khdc du-oc ky hifU bang I!l'c R hay cdc phdn I!l'c X va Y, thi dp ill'c cOO v~t thu- hai ten VI;itthrr nhat se ld irrc R' co cung tri sf; R' = chilu 116'i R hoy la cac phdn Irrc X', Y' co chilu ngtroc 1Ier; X va Y, inhtrng Y' = Y).
Chi! Y r~ng khi dung phuong phap gilti nay thuong hay mAc 111msau: I) Cac 1\I'C X' va Y' kh6ng nguoc chii!u vlri X va Y, nen 2) Cac h.rc X', Y' co chii!u dung (tfrc nguec chii!u vlri X va Y) nhimg lai coi X' = - X, Y' 0= - Y. ket quA thu duoc ding sai, vi thuc sai 111mnhir & truong hop thfr nh1it. D~ tranh cac sai 111mnhu the. thong thuong nen ap bal t~p 26 doi v<ri cac ket diu g5m hai v~t the. Ngoai ra, cac M phirong trinh do-n gian hon vi khong co cac luc can bAng cua toan bQ k~t c1iu n6i chung (xem bai giM bai

I!l'c coo m{it .{it theo tien de 4. R nhtrng' ngu-(J'c co tr] sf; X' = X,

ap

se thu

vao mQt trong hai sai dU'Q'Cket qua sai : khi giai phuong trinh ch1it & day ciing mac

dung phtrcng phap giai nhtr trong phtrong phap nay thlro-ng dlln t<ri trong tham gia vao cac dieu kien t~p 26).

Bili tip 28. Cho lI!c F n~m ngang tac dung len vom ba kh(yp (hinh 71), My cheng minh r~ng khi xac djnh phan h.rc cua cac goi t\!'a A va B kh6ng the di chuyen diem d~t cua hrc F ve djtm E doc theo dU'o-ng tac dung cua no. Sai giii. Neu giai ph6ng cac lien kct ngoai (cac goi tl!a A va B), ta diroc kl;t ciiu bien hinh kh6ng the xem nhir v~t rt.n. BCri vay, khong the di chuyen diem d~t cua I\!,c tac dung len ket c1iu nay doc theo dtrong DE ngay cA khi X3C djnh cac dieu ki~n din bAng cua ket diu (xem § 3, h~ qua cua tiep dt! 1 va 2). De khAng dinh dii!u do, ta giai tnrc tiep bai toan (b6 qua trong hro'ng cua vom). Villi tien ta xet m'ra vom ben phai, xem nhtr vat nr do. Nl'ra vom nay chi chiu tac dung cua hai I\!,c la cac pMn 1\I'C R. va R c cua cac khop B va C (I\I'C R l kh6ng ve tren hinh), Khi can bang, hai I\!,c nay pMi lurong thea mot dirong thang, tlh: la thea dU'o-ng BC. V~y phan I\!,c R. hll'cYng theo BC. Bay gicYxet s\!' can bang ct\a toan bQ vom noi chung, ta th1iy co ba 1lJ'CHIC dung len no : II,J'CF dll cho va cac phan luc cua goi t\l1l R • (co huong dii xac dinh) va R •. Theo dinh Iy ba 1\I'C, dtro-ng tac dung cua chung khi can bAng phAi ct.t nhau tai mQt diem. Do do, ta xac

Hinh 71.

07

xAe

DINH U"NG

urc

TRONG
H

79
deoc xac dinh theo

dinh d!lQ'c chil!u coa phAn h,c R ,. Tr] so coa cac phan luc R , va R tam giac h,re nrong irng.

N~u nhu d~t I\!,c F tai diem Eva suy lu~n nro-ng t\!, nhu v~y, dtrng nhirng hinh vi: can thi~t (hinh 7th), thi ta 51: thliy phan hrc cua cac goi tua R A va R H trong trtrong hQ'P nay khac cA vI! trj so IAn ve chien,
".,

§ 27*. Xac djnh o-ng lITe trong. Ung luc trong cua mQt v~t hay ket
diu (dam, vom v.v...) tai mQt m~t ca.t nao do la cac lire rna cac phan cua v~t do m~t cat do chia ra tac dung len nhau. PhU'O'Ilgphap xac dinh cac irng h!c trong ciing nrong t\1'nhir phirong phap d§:ap dung de nghien ciru SI!can bAng cua h~ v~t. Dau tien ta khao sat S\1' an bAng cua toan bQ v~t c (ket diu) noi chung va xac dinh cac phan 1l!Clien ket ngoai. Sau do dung m~t ca.t (chi) can xac djnh cac irng 1l!Ctrong) phan v~t thanh hai phan, roi xet S\1'can bAng cua nrng phan. Khi do, neu h~ 1l!Cngoai tac dung len v~t la h~ lire phang, thi tac dung cua ph~n bi loai bo, trong tnrong hop t6ng quat, se dU'Q'C thay bAng hf cdc lu-c phdng phiin bO theo m{11 cdt. Cfing nhir tnro-ng hQ'P ngam cirng (xem hinh 62) cac 1l!Cnay diroc thu v~ mQt lire d~t /y tam m~t cat voi hai phan lire x, Y chira biet va mot ng~u 1l!Cvo-i momen M cling clnra biet. Thi du tinh toan co the xem trong bill t~p 29.
6- m~t
8al t4P 29. Cho dAm AB c6 chil!u dai ) m (bai t,p 27, hinh 70), hAy xac dinh irng I\!,c cat ngang cua dam 6- each dAu A mQt doan AE = O,6m.

8ai giaii. Cac lien ket ngoai doi v(ridllm iii goi t\l'll C va cac khop A va B. Ta da tim duoc phan IVC cua cac lien k~t nay khi giAi bai t~p 27. CAt dAm theo m~t cAt ab va xet s\r din bAng cua philn dl1m ben trai (hinh 72).

YE
,ME

e
A
XA Hinh 72.

) XE
Q

Nhir dii n6i 6- tren, ta thay tac dung cua philn b6 di bang hai IVC X. va Y. d~t tai tam E cua m~t cAt va ngAu I\!'c c6 momen M, . L~p cac di!u kiel! can bang (36) cho cac I\!,c X" Y ,. Q, X EYE va ngAu tvc v(ri momen M E tac dung len philn dAm ben trai nay, ta co : ,.

LF""
L
s;

.,.X,+X,
A E-

=0,
= O.

~ F ky "" Y + Y Q = 0, m, (F0 M, -+ 0,6 Y.- 0,5 Q

80

HI;: ttrc PHAN BO BAT KY TREN M~T PHANG

Ch.

IV

In

51> nay

Theo cac sllli~u c~a bai t~p 27 thi Q = 20 kG, X., = 15 kG, Y. = - 7,5 kG. Di!!n cac vao cac phuong trinh tren, ta co:

Xt=~-15KG,

Y.=27,5kG,

M =-6,5kGm.
E

Nhu v~y 6- philn ben trai cua diim tai mij.t cat ab co cac h!c : 1) Iwe doc X • co rae dung nen diim ; 2) hrc ngang Y. co xu guong xe dich pMn diim ti~p giap voi mij.t c!t dQC theo dtr<'rng ab ; 3) ngAu h!c v<'ri momen M. la miimen uon co xu hirong keo dan cac tho- tren va nen cac tho- dll'<'ri cUa dilm.

§ 28*. Cae b,e phin booTrong tinh toan ky thuat ta thirong g~p cac luc phan b3 tren be m~t thea m(>t quy lu~t nao do. Ta hay xet m(>t so vi dl,ldon gian nhat v~ cac hrc phan b() tren mQt m~t ph!ng.
H~ luc phan bo ph!ng diroc the hi~n bang cirong d9 q, tirc la dai IU'Q'ngcua IVCtac dung len mQt don vi chi~u dai, Don vi cuong d9 hrc la Niuton [met hoij.c kiliigam [met (N [m, kG 1m). 1) Cae J"c pbilo 1M> d& dqc thee dolpl thlng (hlnh 73 a). DjI)i voi M hrc nay, cirong d(> q Ia dai 1U'Q'Ilgkhong dOi. Trong tinh toan tinh hoc co th~ thay h~ hrc nay bang hop 1\l'C Q. Tri so cua Q ba.ng Q = aq. (39) Lire Q d~t tai trung diem Clla doan AB.
2) Cae ly-c phin 1M> d9C theo dolpl thing thee quy lu,t tuyen tioh (hinh 73b). Thi du v~ loai IlfC nay nhir ap IlfC Clla flU'O-C len d~p co gia tri Io-n nhat & day va giam xuong dl!n khong & tren mat mro'c. Doi voi cac luc nay ciro-ng d(> q la dai hrong bien doi tang tir khong toi gia tri C\l'C dai qrn.
i----q-C

0)

II

c)

Hinh 73. H9'P lire Q duoc xac dinh nrong ur nhir hop I\l'c cua cac trong hrc tac dung len mQt 11im tam giac dong ch~tABC. Vi trong 1U'Q'ngcua tam dong chat ty I~ v61 di~n tich cua no, nen v~ tri so

'2

aqrn'

(40)

28

cAc urc

PHAN

80

81

LI!C Q d~t tai mQt di~m tren canh AB cua tam giac ABC each canh BC mQt doan
a

ba.ng 3

(xem § 57, diem 2).

4) Cae 1\f'C phin bli dm tboo eung trim (hinh 74). Thi du ve cac 11!Cnay nhu ap lire thuy tinh len cac thanh ben cua mQt binh hinh tru, Theo tinh chat doi xjrng, ta thay t6ng hinh chieu cua cac hrc nay tren true

3) Cae It,.rc pllAn bli dqc tboo do,n thAng tbeo quy lu,t tOy y (hinh 73 c). Giong nhir trong lire, hQ'P 11!CQ cua cac 11!Cnay cO tri so Mng di~ ttic~Eet di~n ABD_§(dll'Q'C do thea mQt ty· ~ t ich hop) va di qua trong tam cua tiet dien do (van de xac dinh trong tam cua cac dien tich se diroc khao sat l:t § 55).

g6c voi true doi xirng Ox bang khong. BlYi v~y hQ'P h!c Q hirong dQC so Q = Qx = trong do qllh lit hrc tac dung len mQt phan to cua cung co chieu dill Il I k; <P k la goc tao b6i IlJC nay voi true Ox. Tren hinh ve ta thay A1k COS<P k = Ily k • Nen neu dira thira so chung If ra ngoai d1iu t6ng, ta diroc : theo true Ox.

Oy thang

se

Hinh 74.

Ve tri

L(q Il/.j

COS<Pk,

Lq6Yk

=q LdYk==

qAB.

Q ~~ qh, (41) trong do h la chieu dai cat tuyen tnrong cung AB.
Bai tf, 30. Dam congxon A B co kich thllcYc nhu tren hinh 75 chju tac dung eOa mQI h~ 1\l'C ph an bo deu vai cireng dQ q 0 k G 1m

Nlur vay :

Hinh 75.

B6 qua trong hrong eua dlim va eho rang ap luc len dliu ngam phan bO theo quy lu~t tuy!!n tjnh. Hiiy xac d;nh ellOong dQ IiTn nhlit qm 'va q'm eua cac l\!,e do, ni!u b = 20 (so sanh vo-i so dIS bai t~p 14 § 17). Bal gh\i. Ta thay cac h.re phan bo bAng hop h.re Q, R va R' cac eong thfrc (39) va (40) : eW! chung, trong do theo

6-731

S2

Hf;: i.trc PHAN BaBAT

KY

TRf:N

MAT PIIANO

Ch. IS

R=-

2q

m'

R'

1 .0 "2 q , ma,

va I~p cac dieu kien din bang (37) cho cac luc song song tac dung len d1im :

LF

kY

...

Q + R ...

R'

"

0,

Lm c (F.l

- R;-

Q( ~

+~)
ta duoc

00

o.

Dien cac tri sll cua Q, R vii R', reli giai cac phuo-ng trinh tren, cuoi cung

N!!u b = 2a, ta se diroc qm = 16qo, q'm = 20qo.

Bai.t,p 31. MQt hinh tru co chii!u cao H , du6'ng kinh trong d, duVC born d1iy khi voi ap IVC p kG / mi, Be dAy cua thanh binh Ill. a. Hay xac dinh irng suat keo 6- thanh binh theo cac huong : I) dQC va 2) ngangnrng sullt bang ty so luc keo chia cho dien tich m~t
cAt ngang).

Bai gi&i. I) CAt binh thea m~t phAng vuong g6c voi true cua no thanh hai ph1in va xet S\l' can bang clla mot ph1in db (hinh 76 a). Tren phAn 1iy, thea huong true binh co cac hrc : ap I\I'C len day F
=

Tt:2 p va cac

I\I'C phan bll tren ti!!t dien clla m~t cAt(tac dung cua

mra ph1in bi be) c6 hop I\I'C Q. Khi din bang ta co Q = F = rr:~ p. Coi dien tich c~a mat cAt
ngang ang chung blng nda ta c6 img sullt keo
0, 0,

bang :

xda 4a 2) Bay gio- cat binh thanh hai ph1in nira thea m~t phang di qua true binh va khao sat S\l' can bAng cua mQt ph1in' do, cho rang tllt ca cac I\I'C tac dung len philn nay & I~t phang cua ti!!t dien giil'a (hlnh 76b). Tren m'ra pMn blnh nay c6 cac hrc : a) cac ap I\I'C phan be deu thea cung R nu-a vong tron voi ctr6'ng dQ q = pH; thea cong thire (41), hQ'P IVC clla cac h,rc nay la R = qd=» pHd; b) cac lire ph an blI tren cac m~t cAt A va B (tir phi a phan binh bi b6 di) ; chung co hQ'P luc 8, va 82, trong d6, do tinh dlli ximg

_g_

="!"''!!_p

kG/m2•

SI b) SI

82

S.
can bang, ta co

Theo

bang

2 tich cua m~t cAt tren do luc S phan be aH(b6 qua dien tich m~t cAt cua day binh), nen ta tinh diroc irng suat keo 02 bang :

cac dieu kien S2 = R, nen S =

2... pdH.

Vi dien

Hinh 76.

02

= --

Sid aR

--p 2a

k G / It! 2 .
keo theo lnrong doc.

Ta thliy irng sullt keo thea heong ngang Ion glip doi t'rng.sulit

~ 29

DA mAc Ll)"C vA DA GIAc DAy CHU(TNG V

83

CO'

sa

CVA

TiNH HQC

no THJ

§ 29. Da glac hre va da ghic day. Thu h~ hrc phAngve hai I\"c. Trong tinh toan ky thuat ngiroi ta thuong dung phiro-ng phap do thi. Phuong phap nay tuy khong chinh xac bang phuorig phap giai tieh, nhirng lai cho cac ket qua nhanh hon \va thay ra hon. Doi voi M hrc phang, phuong phap giai cac bai toan tinh hoc bang do thi dira vao phep dirng da giac lire va da giac dayGia tht'r h~ ba hrc F b F 2, F 3 tac dung len v~t the (hinh 77 a). Nhu ta da biet, hinh abed dung tren cac lire d6 (hinh 77 b) goi hi da gidc 'we 1). Ta nho r:1ng khi mut cua hrc cuoi cling trung voi dau cua lire thir nhat thi da giac lire goi hi da giac kin, neu kh6ng thi goi la da giac hCr.

L
Oc

tr"'C -""'-

b7J

o) Hinh 77.

IJ)

Tren mat phang da giac hrc, ta lay mot diem 0 nao d6 (goi la diem eire) kh6ng nam tren cac canh cua da giac hay tren dirong keo dai cua chung va noi n6 voi cac dinh da giac bang nhirng doan cham cham Oa, a b, Oe, ad diroc ky hieu bang cac chii' so 01, 12, 23, 30 (doc la « kh6ngmot », « mot --- hai » v.v..., cac chir so nay chi ra so tlnr nr cua nhirng hrc dong quy tai dlnh da giac rna mra duong thing di qua). Bay gio tren hinh ve chinh (hinh 77a), ta lay mot diem A nao d6, til' A ke dirong thing song song val mra dirong thang 01 den di€m B la giao diem cua n6 voi' dtrong tac dung cua lire Fl' Til' B ta ke dirong thang song song voi mra dirong t~ng 12 den diem Cia giao di~m cua dirong nay voi diro ng tac dung Cll~I!C F2 v.v... va cir nlnr the ta se duoc mot

d Ta ky hieu cac luc trong da giac hrc bAng cac' stl ; thay cho F 2 viet 2 V.v...

so

C1,l

the thay cho F 1 ta viet so -1,

6*

84 hinh ABCDE

co

sa

eVA TINH Hoe D6 TH!

Ch.

goi la da giac day. SlY di goi nhir v~y Itt vi neu co dinh day

6- cac di~m A va Eva tac dung len day nhirng hrc F b F 2, F 3 tai cac di~m
B, C va D thi khi can bang day se co dang cua dirong gap khuc A BCDE. Da giac day diroc goi la kin neu cac canh ngoai cung cua n6(trong tnro-ng hQ'P nay la AB va DE) chap vao nhau, nrc la se hirong theo mot dirong thang. NgU'Q'c lai, ta co da gidc ho: BAng phep dung nlnr the, co th~ thay bat ky he hrc phang nao bang hai hrc hirong theo cac canh ngoai cung ( AB va DE ) cua da giac day. D~ chirng minh dieu nay, tnroc het ta xet mot hrc nao do tac dung len v~t, thi du hrc F 1 (hinh 77 a). Khi bi~u di~n rieng 11!Cnay diroi dang vecto ab (hinh 77b), roi noi cac diem a va b voi diem 0 nao do, thi cling co nghia la ta phan 11!CF 1 thanh hai hrc aD va Db ,vi til' tam giac hrc aOb ta thay rang F 1 = ab = aD + Db (hinh 77 b). Nhirng theo tien de hinh binh hanh Il!C, neuh = aO + Ob , thi co th~ thay 11!CF 1 tac dung len v~t bang cac hrc aO va Db d(tt tai di~m bat ky nao do tren dirong tac dung CUahrc F b thi du nhir tai di~m B (hinh 77 a). Trong tnrong hQ'P nay vi canh AB cua da giac day song song VM o O, con BC song song VM bO, nen 11!C00 se hirong doc theo canh AB cua da giac, con 11!CDb se hirong . theo Be. D3i VM tat cA cac i1!C khac ta cling co ket qua nrong tu. NhU' v~y, bang each dung cac mra dirong thang 00, Ob v.v ... , tren hinh 77b, ta thay m3i hrc F b F 2, F 3 phan thanh hai thanh phfin la

Fl =ab=aO+Ob,

F2=bc=bO+Oc,

F3 =cd==cO+Od.

Cac 11!Cbang aO va Ob thay cho 11!CF, neu chung d~t tai diem B cua da giac day (hinh 79a). Cling vay, ta co th~ thay cac hrc F2 va F3 bang cac hrc bO va Oc va cO va Od d~t tai cac diem C va D nrong irng. Can chu y rAng cac 11!COb, bO va Oc, cO hirong theo cac dirong thang BC va CD til'n_g_doi m.2!__can bang nhau, vi nhir-r ren hinh 79b, ta thay Ob == - bO va Oc = - cO, do do co th~ loai b6 cac 11!CnaLKet qua la h~ 11!CF 1, F 2, F 3 thuc Sl! diroc thay the bang hai hrc aO va Od hirong theo cac canh ngoai cung AB va DE cua da giac day.
Ket qua nay se thu diroc voi moi h~ co so 11!Cbat kyo

§ 30. Xac dinh hc;rpI.,.c bAng do th]. Neu da giac hrc cua M hrc phang khong kin (vecto- chinh R #0 0), thi theo § 23 h~ 11!Cnay qui ve mot hop
11!C. .
Ta co the dung d6 thi de xac dinh hQ'P hrc bAng each ap dung lien tiep quy tAc hinh binh hanh, nlnrng VM so IU'Q'ng lire Io-n thi phirong phap nay rat cong kenh. Giai bai t011n nay bang phirong phap da giac luc va da giac day thi dan gian han.

po

xAc OINH HQ'P LV'C BANG 06 THI

85

Kn
a)

..
Hinh 78.

Gia thu- h~ lire F b F 2> F'3, F4 tac dung len v~tthe (hinh 78 a). Ta chon mot ty I~ xich nao do, rai til' cac hrc nay dung da giac hrc abcde (hinh 78 b). Canh khep kin ae cua da giac I\I'c cho ta tri sl) va chieu cua h9'P Il!C R. De xac dinh diem d~t h9'P I\I'c, ta n3i cac dlnh a. b. c, d, e v61 eire o nao do va dung da giac day ABCDEF sao cho AB II aO, BC II bO v.v... (hinh 78a). Nhir da chirng minh, co the thay h~ lire tren bang hai Il!C hirong theo cac dirong thang AB va EF. Bo-i vay h9'P lire cua cac hrc do (tire la F b F 2, F 3, F4) se di qua giao diem ella cac dtrong thang AB va EF. Nhir vay, sau khi dung da giac day va keo dai cac canh ngoai cung cua no cho t61 khi chung g~p nhau, ta se diroc di~m K rna h9'P hrc di qua. Bay gio qua K ta ke duong thang mn song song voi ae va d~t hrc R van mot diem bat ky nao do cua mn, ta se diroc h9'P I\I'c can tim. Cach dirng nrong t\l' d3i voi tnro-ng h9'P cac lire song song duoc the hien tren hinh 79. Da giac lire trong tnrong h9'P nay bien thanh mot doan thang, con h9'P luc R=, ae. .

R
Hinh 79.

f'

R6

co"

sa

eLlA

TiNI-I

noc DO Tl-ll

eh. v

, § 31. Xac dinh ngftu t6ng Mng do thi. Neu da giac luc cua h~ hrc phang la da giac kin, con da giac day la da giac h& thi h~ IV'c nay tuo-ng dirong voi mot ngau tbng. Thuc vay, khi da giac hrc abcde cua cac lire F l e F 2, .'J, F4 la da giac kin (hinh 80), thi cac mra diro-ng thang a 0 va e se chap nhau I). Khi do cac canh ngoai cung AB va EF cua da giac day neu da giac nay khong kin thi se song song nhau.

Nhir da chung rninh trong § 29, ta co th~ thay h~ lire da cho bang hai lire bang aO va Oa (vi Oe= Oa) va hiro-ng theo cac diro-ng thang AB va EF. NhU' vay, h~ lire F b F Z, .'3, F 4 tlurc s\f dtro'c thay bang ngjlu hrc (aO, Oa) voi tay don d. Momen cua ngiiu hrc nay bang Oa. d, trong do Oa diroc do theo ty I~ hrc dtroc chon de dung da giac hrc, con d do theo ty l~ chieu dai diroc chon de ve ban vi! chinh.

Hinh 80.

§ 32. Dieu ki~n can bang h¢ Il!'c phang b~ng do thi, Qua cac ket q wi da thu diroc & cac muc tnroc, ta thay die'u kien din vd dl~ dd can bang h¢ hrc phdng ruio dotac dung len' vt;it1M la da gidc lire va da giac day da cdc I!I'C do phai khep kin. Day 1<1 cac dieu kien can b~ng duoi dang d6 thi.
Thirc vay, neu co mot da giac nao do ke tren lire nay se nrong dirong voi hoac mot hop lire, do do se khong co sv can bang. Neu nhu- ca hai kin thi cac hrc tac dung len v~t ro rang se tirong khong khep kin thi h~ hoac mot ngau hrc va da giac nay deu khep dirong voi hai hrc co

I) Trong tnro-ng hQ'P nay ky hieu cac nu-a dirong thang Oa, Ob, ... bang cac 23, 34, 41, vi trong da giac hrc kin 6- mlli dinh co hai hrc d6ng quy.

so

12,

§ 33

xAe

D!NH

cAe

PHAN

i.o-c eVA GO! TI,J"A

87

tri so bang nhau, va htro'ng thea mot dirong thang nhtrng ngiroc chieu (hlnh 80 khi dc~ 0) va do do v~t se din bang.

§ 33. Xac djnh cac phan hre cua gl')i bra. Ta se ap dung plurong phap do thi de xac dinh cac phan hrc cua cac goi nra A va K cua chiec gian the hien tren hinh 81 a. D~u tien, ta chon ty I~ chieu dai thich irng (thi du : 0,4fn irng voi J em) roi the hien tren hinh ve chiec gian va cac lire da cho F b F 2, F 3 tac dung len no. G9i R4 va R, la phan II,rC (y cac goi nra. Trong tnrong ho-p nay ta da biet hirong cua R4, con hirong cua R, chua biet. Bay gio-ta chon ty I~ ve cac lire (thi du 0,5 T irng voi I em), r5i nr cac lire tac dung len gian, ta dung da giac lire (hlnh 81 b) bAt d~u nr lire I, 2, 3 (Fl =c ab, F2 =c be, F 3= ed). Qua trinh dirng hinh phai dirng lai khi ve htrong cua hrc 14, vi ta chira biet tri so cua no (kh6ng biet diem e). Ta chi biet diem mut cua lire R, se (y diem a vi khi can bang da giac lire phai khep kin.

Hinh 81.

De tiep tuc giai, ta chon eire 0 va ve cac mra dirong thang 12, 23, 34, 51. Ta kh6ng biet huong cua mra dirong thang 45 vi chua biet vi tri cua dlnh e cua da giac lire, De xac dinh mra diro-ng thang nay, ta dung da giac day (hinh 81 a) biit dtiu tir di€m A la di€m d(it da Iwe R, chtra biet htrirng (trong tnrong hQ'P ngiroc lai, da giac nay kh6ng the khep kin dtro'c, vi ta kh6ng biet tnroc met diem nao khac tren duong tac dung cua II,rC Rs). Vi tai diem A co lire R, tac dung, nen mra duong thang 51 di nr diem A den chb cAt voi dirong tac dung cua lire F 1 tai diem B. Sau do ta dung mra dtrorig thang 12 toi cM cAt voi duong tac dung cua F 2 tai diem C, mra dirong thang 23 den chb cAt voi dt.rO-ngtic dung cna F3 tai diem D va mra dirong thang 34 den chb cAt voi dtrong tac dung ella hrc R4 tai diem E. Bay gio thea cac dieu kien din bang, ve dirong EA khep kin da giac day, ta se tim dtroc hiro-ng cua mra diro-ng thang 45.

88

cAe

TiNH

cAc

GlAN

Ch.

Vi

Hinh 82.

Sau do nr eire 0 (hinh 81 b) ta dirng mra dirong thang 45 song song v61 EA. Giao diem cua mra diro'ng thang nay v61 hirong cua hrc 4 cho ta dlnh e cua da giac lire din tim. Ket qua la vecto de se bieu dien hrc I4 can tim (theo mot ty I~da chon), con vectobieu dien hrc Rs. Bai toan da giai xong. Thi du ve each xac dinh phan 1l)'C cua cac goi tua bang do thi trong tnrong hop cac hrc song song da dU'Q'Cth~ hien tren hinh 82. Trong tnrong hop nay ta co th~ dung da giac day bcit dau nr diem bat ky nao do vi hu-ong cua ca hai phan lire da diroc biet tnroc, Nita dirong thang 45 can tim dU'Q'C hien tren cac hinh vi! bang nhirng dirong vach kep. the

eo

CHU(yNG

VI

CACH TiNH cAc

GIAN

§ 34. Khai ni~m ve gian. Tinh gian phAng bAng phu-ong phap giai tich. Gian k) diu true ctrng lam bang cdc thanh lIning lien J{er vtri nhau bang cdc khtrp 6, hai ddu. Neu tat ca cac thanh gian nam tren mot mat phang thi gian do goi la gian p/uing. Nhirng chb lien ket cac thanh gian goi la
mit. Tat ca cac tai trong ngoai tac dung len gian chi d~t tai cac nut. Khi tinh gian, ngiroi ta bo qua lire rna sat & cac nut va trong 1U'Q'ng cua cac thanh (vi no kh6ng dang k~ so voi tai trong ngoai) hoac phan bo trong 1U'Q'ng cua cac thanh & cac nut. Khi do, tren mbi thanh gianco hai lire tac dung d~t tai hai dau rna khi can being thi chung phai huong d9C theo thanh. B&i v~y, co the khang dinh : cdc thanh gidn chi bi keo hodc bi nen. DU'Criday ta han cM chi xet cac gian phang cirng kh6ng co cac thanh thira diroc lien ket voi nhau thanh cac tam giac. Trong cac gian nhir . v~y so thanh k va so nut n co lien quan voi nhau theo M thirc

TtNH GIAN PHANG BANG PHtJaNG

PHAp GlAI rtca

89
(42)

k=

2n - 3.

Tlnrc vay, trong tam giac cimg gam ba thanh se co ba nut (xem tam giac ABD tren hinh 84 duoc tao thanh boi cac thanh I, 2, 3). Muon tao them mot nut nira, phai them hai thanh (thi du nut C tren hinh 84 do cac thanh 4, 5 tao thanh, nut E do cac thanh 6, 7 tao thanh, v.v ...). Boi v~y, de tao n - 3 nut con lai, cttn phai co 2 (n - 3) thanh. Ket qua la sa thanh trong gian bang k= 3 + 2 (n - 3)== 2 n - 3. Neu sa thanh it han, gian se khong cimg, con neu nhieu hon gian se la sieu tinh. Vi~c tinh gian nru trung la xac dinh phan hrc & cac goi hrc & cac thanh gian.

nra

va irng

De xac dinh phan lire & cac goi nra, ta co the su- dung cac phuong phap thong thuong trong tinh hoc (§ 25) chi cttn xem gian nhir mQt ca the. Duoi day, ta xet cac phirong phap xac dinh irng hrc trong cac thanh. Phu-ong phap tach nut. Phuo-ng phap nay su- dung ti~n khi cttn xac dinh irng hrc & ca cac thanh gian, NQi dung cua phirong phap hl l~n hrot xet cac dieu kien can bang cua cac hrc d3ng quy tai tirng nut gian. Trinh tir tinh se d11Q'Ctrinh bay trong vi du cu the.

au

Xet gian ve tren ninh 83 a d11Q'Ctao thanh boi cac tam giac vuong can bang nhau. Cac luc tac dung len gian song song voi true x va bang F 1 === F 2 == F 3 ,== F~~~ T. 2 y
n~= 6, so thanh

gian nay la lak= 9. Do do, h~ thirc (42) d11Q'Cthoa man va gian thuoc loai cirng, khong co cac thanh thira. L~p cac phirongtrinh can bang (33) cho toan gian, ta thay phan hrc cua cac goi tua co chieu nhir tren hinh ve va co tri so bang :

So nut trong

Hinh 83.

De xac dinh cac irng luc trong c~c thanh, ta danh sa cac nut gian bang sa La-ma, va cac thanh bang so A-r~p. U'ng hrc can tim trong thanh 1 d11Q'Cky hieu la S" trong thanh 2 lit S 2 V.V ... Bay gio ta nrorig nrong da: cAt roo cac nut cung voi cac thanh d3ng guy tai do ra khoi h~ gian.

90

eACH TjNH cAe GlAN

cu.

VI

Thay tac dung cua cac bi? phan thanh bi bo bang cac hrc hirong doc thea cac thanh nrong irng va co tri so bang cac img hrc S I, S 2, ... can tim. Ta ve ngay Hit ca cac hrc do len hinh thea chien tit' cdc nut ra ngoai, nghla la coi ti1t ca cac thanh deu bi keo (hinh 83 a ; can phai hinh dung hinh ve cua m3i nut nhir tren hinh 83 b doi voi nut III). Neu khi tinh thay irng ll;l'c 6- mot thanh nao do co dl'lu am, thi nghia la thanh nay kh6ng bi keo rna bi nen I). Ta khong can ky hieu bang chii' cho cac ll;l'c tac dung len cac thanh ve tren hinh 83, vi ro rang cac hrc tac dung dQC thea thanh I phai bang S I> con doc theo thanh 2 phai bang S 2 v.v ... Bay gio- ta l~p tuan ttr cac phurrng hQi tu 6- m6i nut trinh din bang (12) cho cac hrc

F kx = 0, F ky = 0, Mt dllu tir nut I, chO hai thanh g(ip nhau, boi vi tir hai phtrong bang ta chi co the xac dinh dU'<?,C hai irng luc An.
L~p cac phuong trinh din bAng cho nut I, ta drroc

trinh din

PI
S2
=-

+ S 2 cos
-

45° = 0,
1=

Tir day, suy ra: F

Y2 =
I>

2,82 T; S

+ S 1 + S 2 sin 45° = Y2 F -- N - S 2 2" = -2" =


N

0.
-

.l T.

Sau khi biet S do do

l~p plnrong trinh can bang cho nut II" ta co

S3+P2= S3= -P=

0,

S4-SI=

0,

-2T,

S4= SI= -IT.

Biet S4, ta l~p cac phirong trinh can bang urong tl;l' nhir v~y cho nut III reli nut IV. Tu cac phtro-ng trinh nay, ta tim diroc S5
=-

S4

V2=

Cuoi cung, de tinh S 9 guy tai nut V bang each chieu tren true By ta diroc = 0, do do S 9 = .- 3 Y2 = - 4,23 T.

1,41 T, S6= S8= - 3T, S7= 0. ta l~p phirong trinh can bang cua cac luc d5ng
YA

S9 cos 45°

=;

Co th~ l~p phuong trinh can bang thjr hai cho nut V va hai phuong trinh cho nut VI de kiem tra ket qua tinh toano Cac phirong trinh do khOng can dung d~ tinh toan irng IlJ'C trong cac thanh, vi thay cho chung ta dii sir dung ba phuo-ng trinh din bang cua d gian d~ xac dinh N, X A va Y A (xem § 26),

1)

Vi Iy do do, trong cac phtro-ng phap tinh ngu-oi ta ghi them cho h,c keo dau
n,

va cho hrc nen dau « -

~ 34

TiNH GIAN PHANG BANG PHUO'NG

PHAp GIAI

rtca

91

Cac ket qua tinh toan c6 th~ dim vao bang diroi day: Thanh U"ng hrc (tinh bang T) 1 -1 2 -- 2,82 3 -2 4 -1 5 +1,41 6 -3

9 -4,23

0 -3

Xet dau cua cac irng hrc, ta thay thanh 5 bi keo, con cac thanh khac bi nen, thanh 7 khong c6 irng hrc (thanh khOng). Vi~c xuat hien cac thanh khong (nhir thanh 7) & tren gian co th~ phat hien dU'Q'Cngay til' dau, boi vi neu tai nut khong co cdc Z¥'c ngoai rna co ba thanh g(ip nhau, trong dO hai thanh htrong theo m(Jt du-ong thang thi lire 6- thanh thir ba bang khong. Ket qua nay thu diroc til' phirong trinh din bang chien tren true vuong g6c voi hai thanh noi tren, Thi du.tren gian hinh 84, neu khong c6 lire P 4 thi thanh 15 se hi thanh khong, va do do, thanh °13 cling the. Khi co hrc P 4 thi ca hai thanh nay deu khong the la thanh khong, Neu khi tinh ta gi;ip nut co so an 1&0 hO'O hai thi co the dung phirong

phap mat c~t. Phu-ong phap mij.tdt (Phuong phap Rito). Phuong phap nay ti~n dung khi can xac dinh nhirng irng hrc trong cac thanh gian rieng re nao do va d~c biet dung d~ kiem tra tinh toano N(_)idung cua phirong phap 1a chia gian thanh hai phfin bang mi;it c~t cat qua ba thanh rna ta can xac dinh irng lire (6o ca ba hoac & m(_)tthanh nao d6), sau do xet SI! din bang cua mot phdn gian do. Muon v~y, thay tac dung cua phan gian bi dt bo bang cac hrc nrong img hircng thea cac thanh bi c~t thea chieu til' cac nut ra ngoai, tire hl coi cac thanh bi keo (nhir phuong phap tach nut). Sau do, l~p cac phtrong trinh din bang & dang (35) hoac (34) bang each chon cac Him momen (hoac true chieu) sao cho trong mJi phirong trinn chi chira m(Jt trng lire tin.

Hinh 84.

92
Thi tac dung la N 1 = gian ben

CACH TiNH cAc GIAN d.... Gia thir can xac dinh irng hrc trong thanh 6 cua theo chieu thang dung la PI = P 2 = P 3 = P 4 = N 2 = 4 T. Ta dirng m~t cM ab cAt cac thanh 4, 5, 6 trai, Bang each thay tac dung cua phan gian ben phai

Ch.

VI

gian ve tren hinh 84. Cac hrc 2 T, phan 1\lC & cac goi nra va xet su din bang cua philn bang cac hrc huang dQC theo

cac thanh 4, 5, 6. DC tim S 6, ta I~p phiro-ng trinh momen dlli voi diem C hi giao diem cua cac thanh 4 va 5. Cho ring AD = DC a va BC.l BE, ta diroc N
I.

2a

-+

P Ia

+ S 6'

CB

O.

Tir do, ta tim du oc S 6' Theo cac kich tlnroc hinh hoc cua gian ta tinh tay d6n CB (neu dung pluro ng phap nay d~ kiern tra ang chirng dung ket qua tinh toan bang do thi (§ 35) WI gi,\n diroc ve tneo ty l~ xich thi co the lily tri so cua CB tren hinh ve), Trong vi du nay L A'BC =
=

90°, CB=

irng lire trong cac thanh 4 va 5, ta co th~ l~p cac phuong Him B (giao diCm cua thanh 5 va 6) va tam A (giao diem cua thanh
S\l can bang cua phan

De tim

aY2. Do d6, S6=

3Y2=

4,23T; thanh bj keo.


trinh mornen doi vo-i 4 va 6).

D~ xac dinh irng lire trong thanh 9, ta dung mi,it cit de qua cac thanh 8. 9, 10 va xet gian ben phai. Lap phuorig trinh cac hinh chieu tren true vuong g6c voi cac thanh 8 va 10, ta diroc
S9COSQt
-

P3

P4

N2=

O.

Tir day suy ra S 9. Ta c6 the tim img lijc lrong cac thanh 8 va 10 bang each I~p plnrong trinh mornen doi vo-i cac tam K va C.

§ 35;* Tinh gian phang bAng do thi. Co th~ tinh gian thea phuong phap tach nut bing do thi .. Muon v~y, dau tien ta xac dinh phan hrc cac goi tva nhtrcach da trinh bay trong § 33. Sau do, Ian hrot cat cac nut ra khoi gian, roi dung cac da giac lire khep kin nrong irng d~ tim irng lire trong cac thanh dong quy (y cac nut do. Tat ca cac phep dung hinh deu tien hanh thea ty l~ xich da chon tnroc (xem § 33). Ba.t dau tinh tir nut co hai thanh dong quy (neu khong, ta khong th~ xac dinh diroc cac irng lire chua biet),
D~ lay lam thi du, ta xet gian vi! tren hinh 85 a. Gian co so nut n = 6, so thanh k = 9. Do do, h~ thirc (42) diroc thoa man va gian thuoc loai cirng khong co cac thanh tlnra, Phan lire R4 va R~ (y cac goi tva da diroc xac dinh trong § 33. Nen ta vi! chung cung voi nhirng hrc da biet

Fl, F2, F3•


D~ xac dinh irng lire trong cac thanh, ta ba.t dau xet tir cac thanh dong quy tai nut I

Hinh 85.

§ 36

BII':U

DO

MACXOEN

CREM{)NA

93

(cac nut diroc danh so La-ma, cac thanh so A-r~p). Hay tirong nrong da dt bo phan gian con lai khoi cac thanh nay 1), thay tac dung cua phan gian bi loai bo bAng cac lire Siva S2 huong doc thea cac thanh 1 va 2. Dung tam giac khep kin cho h~ lire Rs, SI va S2 dong quy tai nut I (hinh 85b). Muon vay, dau tien chon ty 1~xich nhat dinh, roi dung hrc R, da biet, sau do qua diem dau va diem cuoi cua no ke cac dtro'ng thang song song voi cac thanh 1 va 2. BAng each do, ta xac dinh diroc cac lire SI va S2 tac dung len cac thanh 1 va 2. Sau do, xet su can bang cua cac thanh dong quy tai nut II, thay tac dung cua phfin gian bi virt bo bang cac luc S'h S2 va S4 hiro-ng doc thea cac thanh do. Trong so cac lire nay ta dil biet S'1 = - S 1 thea tien de bang nhau giira tac dung va phan tac dung. Dung tam giac khep kin cho h~ hrc dong quy tai nut II (bat dau til' lire S't) ta se tim du-oc S3 va S4 (0- day S4 = 0). BAng plnrong phap tiro-ng tu-, ta xac dinh cac irng hrc 0- tat d. cac thanh con lai, Cac da giac ll!c tiro'ng irng voi cac nut ve tren hinh 85b. Da giac cuoi cung (0- nut VI) dung de kiem tra,)vi tat ca cac lire dong quy 0- day da du-oc xac dinh. Tir cac da giac vira du-ng, theo l~ .xich d1i chon, ta xac dinh d~Q'C tat ca cac img lire. Dau cua irng hrc trong m3i thanh diroc X{lC dinh nhtr sau: hay urong nrong da dl roi mot nut nao do (thi du nut lll) co J's cac thanh dong quy, roi d~t cac hrc vira tim duoc vao chb cac thanh bi dt (hinh Hinh 86. 86). Khi do hrc co chieu hirong tir nut ra ngoai (nlnr S5 tren hinh 86) la lire keo thanh, con lire hirong vao nut (nhtr s; va S6 tren hinh 86) la lire nen thanh. Theo quy tro'c (xem § 34) cac hrc keo mang dau « + », cac il!c nen mang dau « - ». Trong thi du khao sat 0- hinh 85, cac thanh 1, 2, 3,6, 7, 9 bi nen, con cac thanh 5, 8 bi keo, § 36: Bi~u diS MACXOEN - CREMONA. Co th~ tinh giau bAng do thi nhanh ho-n va bieu di~n k@t qua tinh toan ngan gon han, neu trong qua trinh dtrng hinh ta g9P tlit cit cac da giac lire 6- cac nut gian vao mot bieu do irng hrc goi Iii bit!u dri Macxoen - Cremona.
De: dung bieu do nay, nhlit thiet phai tuan theo trinh nr nh1it dinh nhir sau :

tv

I) Tim phan lire 6- cac goi nra eua gian.

bing.

I) Tren hinh 86 bieu dien cac thanh bi cat khoi gian Doi voi cac nut khac cling diroc bieu dien nrong

6-

nut 111 la nut dang khao sat din nr.

1)4

CACH TiNH

cAe

GIAN

Ch.

VI

2) Bieu di1!n tal d. cac hrc da cho va phan lire cua cac goi tva tac dung len gian sao cho chung deu narn (~,ngodi bien e,;o gian (hinh 870). Ky hieu cac khu virc narn giira cac lire nay va bien ella gian va gifra cac thanh gian voi nhau bang cac chir A, B, C, ... , K. 3) Thea ty I¢ xich da chon, dimg da giac lire khcp kin cho h~ hrc ngoai tac dung len gian (tire la cac hrc da biet vii cac phan lire & goi tva) bang each d~t cac hrc thco thu: IW gflP chung (I- ngoai bien gian theo chiiu kim ddng h6 (cac diro-ng darn net tren hinh 87 h). De cho tien, ngiroi ta ky hieu diem (Hiu va di6111rnut cua Ivc bang cac chir nho ttro'ng irng voi ten cac khu vue ngoai bien narn ben trai va ben phai cua lire (lire J.' 1 ky hieu bang ab, hrc F 2 bang be, v. v ... ). Nhir vav, da giac elm cac hrc ngoai cua gian se la da giac abcdea, (Tren bieu do khong vi:' cac miii ten hrc). 4) U.n luo t dirng them vao cia giac cua cac hrc ngoai cac da giac hrc cho tat c~ cac nut gian, bi'lt dau til" nut dong quy cua hai thanh ; khi du-ng tung do giac, nen bdl diiu tir cdc lu:c dD biet ; ,.fii den cdc hrc khac thea thtr tl!' g(ip chung khi vong quanh nut do thea chie'u kim d{ing hrJ'(ta ciiug ky hicu cac frng hrc trong thanh nhu cac hrc ngoai : irng lire trong thanh 1 ky hieu bang at, trong thanh 2 bilng fe, v.v ... ).

Hinh 87.
Vi du, tai nut VI (hinh 87a) cac doan cd va de (hinh 87h). Til" diem diem {' du-ng duo-rig thang song song k cua da giac lire khep kin cdekc cua tv nlnr vay. lire e ta voi nut F 3 va R4 da biet, dtroc ky hi~u tren bieu do bang dung dtrong thang song song vo-i thanh 8, va til" thanh 9. 'Giao diem cua cac duo-ng d6 cho ta dlnh VI. Da giac luc cua cac nut khac ding dung tuo-ng .

Bieu do cac img Ivc cua ell gian dung brlllg each do diroc the hien tren hinh 87 h. De tim irng lire & thanh I, ta nro-ng nro'ng cat nut I, roi di vong thco chieu kim dong ho va doc ten hrc : Tim tren bi6u do vecto' -;;; roi xac dinh gia tr] cua no. D(it vecto nay van thanh I (hinh 86) ta thay thanh b] ncn. Ta ciing thu du-oc ket qua nhtr vay, neu cilt nut II roi di vong theo chieu kim dong ho sc thay lire din tim co vecto Nhir vay, ta thay rAng phiro-ng phap ky hi~u lire nhir th~ da tv 1116til dinh luat bang nhau giira lire tac dung va lire phan tac dung,

at:

;-0.

GIn chu y toi nhimg

tnrong

hop rieng sau day:

I) Neu tai nut khong d~t cac luc ngoai rna co ba thanh dong quy, trong do hal thanh huang thco mQt dirong thang, thi nhu da noi & § 34, irng hrc & thanh th(r ba (thanh 4 tren hinh 870) se bang khong (thanh kh6ng). Vi v~y, cac di~m g va/tren bi~u do (hinh 87b) sC trung nhau (g[= 0). 2) Neu trong gian co cac thanh cheo nhau (hinh 880, thanh 1 va 5), thi co the dung bieu do ung hrc cua gian bang phuo-ng phap thong thuo'ng xem giao diem cua cacthanh nhir mot nul. Khi do, irng lire & cac thanh I va 5 se diroc bieu di~n hai Hin tren bieu do, chung co

~ 37

cAc DjNH LU~T VE MA SAT rmro-r

95

Hinh bi~u di~n irng lire & thanh

88 . irng diroc the hien tren hinh 88b, trong 5. I, con ef va gd 13 irng hrc & thanh

.cling gia trj tuyet doi va cung dau, Bibu do luang do cac doan de vsfg 3) Neu khi dimg bieu

do g~p

phai nut co so fin Io-n han hai, thi din phai du-ng thil' bieu del

& ca hai dau gian (ueu gian kh6ng doi xirng) hoac xac dinh frng hrc trong mot so thanh
bang phuo ng phap miit ctu (§ 34). Bang phiro-ng phap nay, ta co the kiern tra ket qua va d(> chinh xac cua phep tinh bilng do thi.

CIlUO'NG

VII

MA

SAT

1)

§ 37. Cac djnh lu~t v~ rna sat tru-o-t. Thirc nghiem clnrng t6 rang; khi chuyendich mot vij.t tren be mat cua mot vij.t.khac, thi tren rn(tt tiep xuc cua cac vij.t do xuat hien hrc can doi vo-i chuyen dQng tnrot, hrc can do goi la hrc rna sat tnrot.
Nguyen nhan lam xuat hien rna sat, trtroc het la do be rn~t tiep xuc go ghe him can tro- str di chuyen va do hrc dinh ket cac vij.t vao nhau, Vi~c nghien ciru tilt ca cic d~c tinh cua hien nrong rna sat hI mot van de co- ly kha phirc tap virot ra ngoai pham vi cua giao trinh co- h9C ly thuyet, Trong tinh roan ky thuat, .ngiroi ta thirong xuat phat tir mot so dinh luij.t tbng quat diroc xac dinh bang thirc nghiem, chung phan anh cac d(tc tinh CO' ban cua hien nrong rna sat voi dQ do chinh xac dai v&i thirc tien. Cac dinh luij.t .do goi la dinh luij.t rna sat tnrot tlnh, Ta co the dien dat chung nhir sau: 1. Khi chuyen dich mot vij.t tren be rn~t cua vij.t khac se xuat hien 11!crna sat (hay hrc dinh ket) 0- mat phang tiep xuc giira cac vij.t. Lire nay co th~ co tri so tu khong den Fgh goi la lire ma sat gitri han.

') Trong

chuang

nay, chir F thuong

diroc dung de ky hieu lire rna sat (N.D.).

96

MA

SAT

en. VII

Lire rna sat co huong ngiroc voi chieu chuyen dQng cua v~t rna hrc tac dung co xu huong gay ra. 2. Tri sa lire rna sat gioi han bing tich h~ so rna sat tinh voi ap lire phap tuyen hay voi phan hrc phap tuyen : Fgh = fo N. (43) H~ sa ma sat tinh fo la hir sa. No dU'Q'C xac dinh bang thirc nghiern va phu thuoc vao v~t lieu cua cac v~t tiep xuc va trang thai be m~t (dQ bong, nhiet dQ, dQ am, dQ boi tron, v.v ... ). . 3. Tri so luc rna sat gioi hantrong pharn vi kha rong khong phu thuoc vao kich thiroc cua cac be m~t tiep xuc khi bj rna sat. Ket hop dinh luat tlur nhat voi dinh luat thir hai, ta thay rang, khi can bang l\!,c ma sat tinh (I\!'C dinh ket) co gia trj F <;; F g.h hay F <;;foN. (44) Co th~ xac dinh h~ so rna sat bang thirc nghiem nho mot thiet bi don gian th~ hien tren so do hinh 89 . Nguoi 'ta d~t mQl khoi chir nhat D N lam bang cac V?t li~u c~n xac dinh h~ sa rna sat len m~t ban nam ngang F AB. Trong hrc P se tac dung len kh3i D lam can bang v(yj phan lire phap A tuyen N cua m~t ban, con lire tTU'Q't p Q khi dung yen can bang voi hrc rna sat F (hrc Q bang trong hrong cua Hlnh 89. dia din va can E). BAng. each chlit tai 'tang dan vao dia can ta se tim diroc lire Q * luc khoi D bAt dau xe dich. R5 rang la khi do lire rna sat gioi han Fgh = Q*. Vi trong tnrong hop nay N = P, nen thea cong thtrc (43) ta tinh dtroc :

qua

./, N P' Lain mQt sa thinghiern nrong tu.ta thay rang khi thay d6i trong gioi han nao do trong IU'Q'ng P thi lire Q * se tang ty I~ voi P, nhtrng gia tri fo van khong thay d6i. Cling v~y, h~ so fo van co gia trj ban dau neu cho dien tich cac m~t ben cua khoi thay d6i trong mQt gioi han nao do. Nhirng thi nghiern do khang dinh cac dinh lu~t ma sat 2 va 3 la dung. Dinh lu~t thir nhat dung (y ch6 neu bat Cll' tai trong Q nao be hori Q* thi khoi D vAn dirng yen tai ch6. Do do, hrc ma sat F can bang voi lire Q co the co gia tri tir khong (khi Q = 0) den F gh (khi Q ~ Q*). Can chu y rang khi khOi D dirng yen thi hrc ma sat F bing lire tTU'Q't Q, chir khong bing F gh = fo N. L\I'C ma sat chi co gia tri foN khi vj tri can bang (y trang thai gioi han.

(0

= Fgh

Q'"

PHAN

t.irc LIEN

KET

NHAM.

GOC

MA SAT

97

DU'm dav hi h~ so ma sat cua mot vai v~t li~u :


03 vei gli Kim loai v6i kim (01).i Thep v{Yibang

=
c~

0,4 --:- 0,7 0,027

/0

= 0,15 -r- O,~5

.f~

Nhirng so lieu chi tiet hori co th~ tim trong cac sach tra cuu

Tat nhirng dieu trinh bay tren noi ve ma sat tnrot tinh. Khi chuyen dong, lire ma sat co htro ng ngtro'c veri chieuchuyen. dong va bang tich h~ so ma sat dong veri ap hrc phap tuyen F= so va dU'<?,C xac dinh bang thirc nghiem, H~ so ma sat I khong nhirng phu thuoc vao v~t lieu va trang thai be mat tiep xuc ma trong mire d9 nao do con phu thuoc vao v~n toc chuyen dong cua vat. Trong nhieu truong hop khi cho van toc tang len, h~ so I dau tien hoi giam nhirng sau do hau nhir giii: nguyen gia trj khong d6i.

ca

ky thuat,

H~ so rna sat trirot dong

I cling hi hir

IN.

§ 38. Ph an lu-c lien Ut nham. Goc rna sat. Cho den nay, khi giai cac bai toan tinh hoc ta dii bo qua khong tinh den lire rna sat va gia thiet rang cac mat lien ket la nhan, phan hrc cua chung hiro-ng theo phap tuyen cua cac mat do. Phan lire lien kCt thirc te (lien ket nharn) co hai thanh phjin la phan lire phap tuyen N va lire rna sat F vuong goc veri N. Boi vay, phan lire toan phfin R sf: lech veri phap tuyen cua rn~t lien ket mot goc nao do. Khi cho lire rna sat thay d6i tir khong toi F gh , lire R sf: thay d6i nr N to-i R gh- con g6c ho-p veri phap tuyen sf: tang tir khong den gia tri gioi han CPo nao do (hinh 90). Goc CPo lori nhat rna phan hrc toan phan cua lien ket nham Hinh 90. tao thanh veri phap tuyen ella mat phang goi la goc ma sat.
Tir hinh 90 ta thay tg CPo Vi Fgh rna sat hi
=

-N-'

Fgh

j;)N. nen tir day suy ra h~ thirc giira goc ma sat vahe tg CPo

so

Io·

(45)

Khi din bang, tuy thea cac hrc tnrot ma phan luc toan phfin R c6 the nam 0- mot vi tri nao do ben trong .goc rna sat. Khi can bang 0- trang thai
7-7:>1

98

MA SAT

Ch.

VII

gioi han, thi phan lire se lech voi phap tuyen mot

goc bang CPo.


Neu tac dung len v~t nam tren mi;it phang nham mQt hrcP I~p voi phap tuyen mQt goc a. (hinh 91), thi v~t se chi xe dich khi lire trtrot P sine Io-n hon Fgh" fo Pcos« (ta coi N = Pcosa., bo qua trong hrong cua v~t). Nhirng, bat dang thirc P sin a. > > .fo P cos a., trong do fo = tg CPo chi diroc thirc hien Hinh 91 khi tg Ilt > tg CPo, nrc la khi (X > 'Po. B6i v~y, khong co 1\l'Cnao I~p vo-i phap tuyen goc a. be hon goc ma sat CPo lai co th~ lam cho v~t chuyen dQng doc thea mi;it nay. Dieu nay giai thich vi sao co hien nrong v~t dirng yen hay nr ham.

§ 39. Can bAng khi co rna sat. Bai to an khao sat ve S\l' can bang ella cac v~t co ke den l\l'c rna sat thirong diroc dira ve tnrong hQ'P xet vi tri can bang gioi han khi lire rna sat co gia tri 16'n nhat Fgh' Neu giai bai to an bang phirong phap giai tich, thi trong tnrong hQ'P nay ta phan phan I\l'c cua lien ket nham thanh hai hrc thanh phan la N va F gh . trong do F gh=
.fo N. Sau do, I~p cac dieu kien can bang tlnh hQCquen biet, roi thay Fgh bAng .foN va gifti cac phirong trinh do ta diroc cac dll-i hrong din tim.
=

se

Neu bai toan yeu cli~ xac djnh tat cft cac vi tri can bang co the xay ra, thi khi giai cung co the chi xet vi tri can bAng gio'i han, Cac vi tri can bAng khac tim duoc bang each giam M so rna sat til.f~ toi khong ') .

se

Ta chu

F khong bAng F gh: Gia tri F (neu din tim) phai duoc xac djnh tlr cac dieu kien can bAng diroc xem nhir la mQt An moi (xem phlin thir hai cua bai

rang, & cac vi tri can bang khong phai la giO'i han, lire rna sat

j,

"t~p 32).

Neu giai bang phuong phap hinh hoc, ta nen bieu di~n phan luc lien ket nham bang mQt lire R rna & vi tri can bang gioi hlin se lech khoi phap tuyen cua mi;it lien ket mQt goc CPo.
Bal 32. Hay xac dinh xem life Q I~p voi dlTang nam ngang mc}t g6c 11= 30° phai bang bao nhieu de khi tac dung vao v~t c6 trong IITI,Yng = 10 kG ndm tren m~t phAng P

tt,

') Thuc v~y. khi din bang tr trang thai gioi han, thi life rna sat F = Fill! = foN. 6' cac vi tri can bang khac : F < fo N. Tire la, co th~ tim cac vi tri nay bang each giAm dlj.i IITI,Yng (0 trong dang thirc F = foN. Khi /0 ~~ 0 ta co vi tri din bang frng voi trlTang hQ'P Iii:n ket nhAn (IY nrong),

ngang (hinh 92) co the him cho v~t diYi ch6. Bi~t r~ng he m~t pha.ng IA 10 = 0,6. 8al gill. .Theo di~u kien bai toan, ta clln xet vi tri can b~ng gim han cl'ia v~t. Tai vi tri nay v~t chiu tac dung cua cac IIlC P, Q, N va Fgh- L~p cac di~u kien din Mng dU"o-i dang hinh chi~u tren true x va y, ta co ;
QCOSIZ-

so rna sat tlnh giihl v~t vo-j

Fgh

= 0,

QsinIX-

P = O.

Tir phirong trinh cuoi Cling suy ra N = PNen Fgh


= IoN =

Q sine,

fo {P- Q sine).

p
Hinh 92.
tnro't sf bang lai bang Fgh = rna sat gifr cho tren true Ox va .

Thay gia tri cua F gh vao phirong trinh thir nhlit roi giaj phirong trinh nay, ta dU"Q"C

=.

COSIZ

J~ P J;

sine

= 5,2 kG.

Neu d~t vao v~t mQt h,c nho hon, chang han Q' = 4kG, thi h,c Q' cos 30° = 2 V3 = 3.,~6kG,trong khi do 1\l'Crna sat C\l'Cd;;ti co thi: xay ra = fo (PQ' sin 30°) = 4,8kG. Boi vay, v~t sf dirng yen tai ch6. Khi dolllC v~t can bang se tim diroc tir phirong trinh can bAng dU"m dang hinh chil!u sf bang luc trirot (F= Q' cos 30" ,= 3,46 kG) chir khong bang IIlC Fgh.

Clln chu y rang, trong t~t ca cac phep tinh nen xac dinh F gh theo cong thirc F gh ~'=IoN, trong do N diroc tim tir cac di~u kien can b~ng. Sai lllm thirong mAc phai khi giai nhfrng bai toan urong t\r nhir bai toan nay la coi F go = loP, trong luc do thi ap 1\l'C len m~t phang 1;;t1khong bang trong 1U"Q"ngcua v~t P. Oai t~p 33. Hay xac dinh vm goc nghieng at bao nhieu thi v~t nAm tren ~t phang nghieng sf can bang, n~u h~ so rna sat ct'ia no v6'i m~t phAng bAng fo.· 8al giM. Trong bai roan tlii ca cac vi tri dln'bAng cua . h!t. ta hay xac dinh vi tri c!n' IX bAng rJgh. vj tri nay (hinh c\\a trong h,c P, phan h,c phap nay yeu cllu xac dinh v~t. Muon v~y, truoc b~ng gioi hIP! khi goc 93), v~t chiu tac dung tuy!n N va 1\l'Crna sat

p
__

Hinh 93.
N tgatgh. Nhirng m~t khac,
(a)

gi6'i han F gh. Dung F gh = foN. Nen

tam giac 1\l'Ckhep kin, ta tim diroc Fgh tgatgh


= fo.

Trong dAngth~ tren,nllu/o giarn thi dQ Ion C"Ia rJllh"Cung giarn. Bm v~y ta kilt lu~n ; s., can bAng cling co th~ xdy ra khi rJ < IlIgh. Kilt qua hi tlit ca gia tri cua g6c rJ rna v~t sf can bAng dU"Q"cxac dinh theo bat dang thirc ; . tge <. fo. • (b) N(!u khong co rna sat (fo =, 0), thi can bang chi co th~ xa.y ra khi II = O. Yfiy, trong khi co rna sat mil s" can b~ng co th~ xAy ra khi goc II co bat k~ gia tri nao gijra khong vii IIgh, thi khi khong co rna sat, s" can bang chi co th~ xa.y ra v&i gia tri duy nhat cua goc II bang khong, Day s., khac lJi~t giii'a can bang co rna sat v&i din bang Clla cac h~ co cac lien kilt Iy tU"o-ng(khong co rna sat}.

la

Co th~.dllng kilt qua thu diroc trong bili toan dU"Q"C tht hien bAng dilng thire (a) d~ xac dinh he so lila sat bang thuc nghiern (theo goc a gh do diroc). 7;:·

100

MA sAT

Ch.

VII

Clln chu y them la vi.fo = tg!po, trong do !Pola g6c rna sat, nen IIgh = !Po, ttrc la g6c II Ion nhllt rna v~t nAm tren m~t phang nghieng vAn can bAng d\rQ'c bAng goc rna sat. 88i tip 34. MQt thanh uan vuong g6c co phlln thAng dtrng tua vao hai ga A va B 6- each nhau theo chieu thang dtrng mQt doan bAng h (hinh 940). 86 qua trong hrong coa thanh, hay xac dinh veri chi!u rQng d bAng bao nhieu 'thi thanh duoi rae dung cua tAi trong P

p Hinh 94

d~t 6- pMn nAm ngang CUll no, se can bAng khong phu thuoc vao sa rna sat gill'a thanh voi cac ga bAng 10' Bill gill. Xet vi tri can chiu tac dung cac dii!u ki~n

vi

tri cua tAi trong P. H~

QQi Pia trong hrong coa tAi tr "ng, Iia khoang each til Pta; phlln thAng dtrng. bAng gilri han coa thanh trng vlri chi!u rQng d = dgh. Tai vi tri nay thanh cua cac h,rc P, N, F, N', F', trong do F va F' hi cac hrc rna sat gieri han. L~p can bAng (33) va Illy cac m6men dai veri tam A, ta diroc : N-N'

= 0,

F+F'-P N
=

= 0:
N', P
~=

Nh-Fdgh-PI trinh dllu ta co

0,

trong d6 F = IoN,

F' = IoN. Til hai pfnrong

2foN.
=

Thay cac gia trj nay vao phirong hay

trinh thtr ba, sau khi don gian N ta dlrQ'c

h - lodgh - 2/01

0,

h dgh = - - 2/.

10

Trong dAng thtrc nay n~u giAm/o tCri khong, thi v~ phai cua no se tang t61 vo cue, Nen Sl,l'can bang c6 th~ x&y ra veri moi gia tri d >dgh. Con dgh se co gia trj Cl,l'Cd~i khi I = 0. Dii!u d6 co nghia la thanh se can bAng voi moi vi tri cua tAi trong (khi I ~ 0) n!!u h d ">'/0' He sa rna sat cang be thi d cang phAi lern hon. Khi khong c6 rna sat Sl,l'can bAng kh6ng th~ x&y ra vi khi d6 d 0= 00.

(fo

0) thi ro rang

Hay xet them each giAi bai toan bAng hinh hoc, Trong each giAi nay thay cho cac phan Il,I'Cphap tuy!!n va Il,I'Crna sat, tai cac di~m A va B ta dung cac vecto phan Il,I'Ctoan phlln R, va R. rna tai vj tri gi61 han se lech voi cac phap tuy!!n mQt goc bAng goc rna sat !Po (hinh 94b). Khi d6 thanh se chiu tac dung cua ba Il,I'c R.,R., P. Khi can bAng,cac dlrang tac dung cua cac Il,I'Cnay phai cAt tai mQt-.di~m, do la di~m K, giao di~m cua cac luc R, va R •. Do do, ta co dang thtrc hi~n nhien (xem hinh vel h = (I + 4h.) tg!po r Itg!po, hay h = (21· + dgh)Jo, vi tg !P" = [«, K~t qua la ta tim dlrq'C gia trj dgh nhtr (y bai giAi theo phirong phap giAi tleh. 8ai toan l1ay hi mQt vi du vi! cac
C<Y

cau tl,l' ham thirong

dll"Q'c 'Sir dung trong thee

te.

Bal tip 35. 86 qua trong JlrQ'ng Clla thang AB (hinh 95), hay xac dinh vai cac gia tr; nao caa gee II thi ta co th~ treo t61 dlnh thang B. Cho bi!!t g6c rna sat cua thang vai san va tll"ang la !Po.

You might also like